Pediatric Trauma + Critical Care Flashcards

1
Q

Which of the following statements concerning pediatric trauma is true?

A. Trauma is the second leading cause of death in children between 1 and 15 years of age.

B. Computed tomography (CT) is indicated when there is a painful distracting injury, significant head injury, or an unclear examination.

C. Indications for operative intervention include documentation of injury to the spleen or liver on CT.

D. Intraosseous access is the preferred means for delivering fluids or blood in a child younger than 10 years of age.

E. Surgical cricothyroidotomy is an acceptable means of airway control for a child younger than 12 years.

A

ANSWER: B
COMMENTS: Trauma is the leading cause of death in children between the ages of 1 and 15 years. Motor vehicle accidents, falls, bicycle accidents, and child abuse are the most common causes of traumatic death.

The priorities of resuscitation are airway, breathing, and circulation. Fluid resuscitation is given as 20 mL/kg boluses. Intravenous access is the preferred method of fluid administration. However, if intravenous access cannot be obtained in a timely manner, a specially designed needle can be used to deliver fluids or blood through an intraosseous route in children younger than 6 years, most commonly via the tibia. The needle is placed 1 to 2 cm below the tibial tuberosity through the anteromedial surface of the tibia under sterile conditions.

If hypovolemic shock is refractory to two crystalloid boluses, blood transfusion should be initiated as a 10 mL/ kg bolus.

CT is commonly used to evaluate pediatric trauma patients. CT, which can include chest, abdomen, and pelvis, is indicated when there is an injury elsewhere causing pain, a significant head injury precluding a reliable examination, or if there is an equivocal examination in general.

Although injuries to the liver and spleen are common, the need for operative intervention is not absolute and many patients may be managed nonoperatively if stable.

Surgical cricothyroidotomy should not be attempted in a child younger than 12 years because of the risk of inadvertent airway injury. If an airway is needed, needle cricothyroidotomy can be performed.

How well did you know this?
1
Not at all
2
3
4
5
Perfectly
2
Q

An 8-year-old female presents as the restrained passenger after a head-on collision. On the secondary survey, a seatbelt sign is noted. In the pediatric population, what is an associated injury?

A. Chance fracture
B. Duodenal hematoma
C. Colonic perforation
D. Splenic hematoma
E. Hepatic hematoma

A

ANSWER: A
COMMENTS: Seatbelts in a motor vehicle collision are lifesaving. A “seatbelt sign” is abdominal wall ecchymosis that is in the pattern of a lap belt across the abdomen.

When observed on physical examination, this has been shown to have an increased rate of both solid and hollow organ injury.

In the pediatric population, the triad of abdominal wall hematoma, chance fracture, and jejunal or ileal perforations has been well described.

A chance fracture is a flexion–distraction type injury that involves the anterior, posterior, and transverse portion of the vertebral body of the lumbar spine.

Patients with a seatbelt sign should have a thorough evaluation for intraabdominal injuries as well as vertebral injuries.

These patients should undergo initial imaging with either a CT scan or a focused assessment with sonography for trauma (FAST) examination and should be admitted for serial abdominal examinations, even with normal initial imaging studies.

How well did you know this?
1
Not at all
2
3
4
5
Perfectly
3
Q

What are the most common foreign bodies ingested by children?

A

The vast majority of ingestions in children are accidental.

The most common type of ingested FB varies by geographic region.

In the United States and Europe, coins are the most common. Other commonly ingested objects include toys, batteries, needles, straight pins, safety pins, screws, earrings, pencils, erasers, glass, fish and chicken bones, and meat.

However, in areas of the world where fish contribute a significant portion of the diet, such as in Asia, a fish bone is the most common FB ingested by children.

How well did you know this?
1
Not at all
2
3
4
5
Perfectly
4
Q

When should foreign body ingestion be suspected?

A

FB ingestions usually present after a witnessed event or disappearance of an object.

Also, there may be heightened suspicion for ingestion by a caregiver based on the child’s description.

The initial presentation can vary from the child being completely asymptomatic to a variety of symptoms including drooling, neck and throat pain, dysphagia, emesis, wheezing, respiratory distress, or abdominal pain/distention.

The majority of patients will have a normal physical exam; however, the child should be evaluated for signs of complications.

Physical exam findings that raise suspicion of potential complications include oropharyngeal abrasions, crepitus, or signs of peritonitis.

How well did you know this?
1
Not at all
2
3
4
5
Perfectly
5
Q

What are the most common sites for foreign body impaction in the Esophagus?

A

The esophagus is the narrowest portion of the alimentary tract and is thus a common site for FB impaction.

Within the esophagus itself, there are three areas of anatomical narrowing that are potential areas of impaction:

the cricopharyngeus sling (70%),

the level of the aortic arch in the mid esophagus (15%), and

the lower esophageal sphincter at the gastroesophageal junction (15%).

Other areas of potential impaction may be found in the esophagus of children who have underlying esophageal pathology (i.e., strictures or eosinophilic esophagitis) or prior esophageal surgery (i.e., esophageal atresia).

Although usually asymptomatic, sharp foreign bodies may penetrate the mucosa at any level and cause mediastinitis, aortoenteric fistula, or peritonitis.

Symptoms of esophageal FB impaction are nonspecific and include drooling, poor feeding, neck and throat pain, vomiting, or wheezing.

Radiopaque objects can be detected on anteroposterior (AP) and lateral neck and chest radiographs, while radiolucent objects may require further workup with a gastrografin esophagram or esophagoscopy depending on the symptoms and provider’s level of suspicion.

How well did you know this?
1
Not at all
2
3
4
5
Perfectly
6
Q

What are options for retrieval of ingested coins?

A

The most common round, smooth object ingested that is amenable to extraction or advancement is a coin.

The majority of esophageal coins will appear en face in the AP view, and from the side on the lateral radiograph.

On occasion, more than one coin will have been ingested, and thus completion esophagoscopy is generally recommended following removal of the first coin.

The location of the object on the radiograph is important in determining the treatment options.

Most FB impactions are located in the proximal esophagus at the level of the upper esophageal sphincter or thoracic inlet.

The majority of FB impactions found in the upper or mid esophagus will remain entrapped and require retrieval.

Options for retrieval include nonemergent endoscopy (rigid or flexible) and Foley balloon extraction with fluoroscopy.

The Foley balloon extraction technique should be limited to round, smooth objects that have been impacted for <1 week in appropriately selected children without any evidence of complications.

This technique was found to have a success rate of 80% while significantly lowering costs.

Objects that are impacted in the lower esophagus often spontaneously pass into the stomach.

For this reason, certain lower esophageal impactions can be observed for a brief duration of time, or attempted to be advanced into the stomach with bougienage or a nasogastric tube in the ED without anesthesia.

Recently, transnasal esophagoscopy has emerged as a new option. Advantages include shortened procedure time and the need for only a local anesthetic.

Rarely, a chronic esophageal coin can cause esophageal perforation, but this will usually be contained.

How well did you know this?
1
Not at all
2
3
4
5
Perfectly
7
Q

How do you manage gastrointestinal foreign bodies?

A

FB ingestions that are found to be distal to the esophagus are usually asymptomatic when discovered.

Signs and symptoms including significant abdominal pain, nausea, vomiting, fevers, abdominal distention, or peritonitis should alert the provider to potential complications including obstruction and/or perforation.

The majority of FBs that pass into the stomach will usually pass through the remainder of the gastrointestinal (GI) tract uneventfully.

These patients can be managed as an outpatient.

Occasionally, a FB will remain present in the bowel after a period of observation and serial radiographs.

Prokinetic agents and cathartics have not been found to improve gut transit time and passage of the FB.

Often parents are instructed to strain the child’s stool; however, in up to 50% of cases, the FB is not identified even with successful passage.

If the child remains asymptomatic and the FB has not been identified, a repeat abdominal radiograph can be performed at 2- to 3-week intervals.

Subsequent endoscopy is usually deferred for 4–6 weeks.

How well did you know this?
1
Not at all
2
3
4
5
Perfectly
8
Q

Why are esophageal battery ingestion cases associated with increased morbidity?

A

Battery ingestions deserve special attention due to the potential for significant morbidity associated with esophageal battery impactions.

Button batteries are more commonly ingested than cylindrical batteries in young children.

Symptoms occur in fewer than 10% of cases.

Button batteries will appear as a round, smooth object on radiographs and are often misdiagnosed as coins. However, on close inspection, some larger button batteries will demonstrate a double contour rim.

Esophageal batteries are associated with increased morbidity due to:
1) the tissue injury that can occur through pressure necrosis,
2) release of a low-voltage electric current, or
3) leakage of an alkali solution, which causes a liquefaction necrosis.

This mucosal injury may occur in as little as 1 hour of contact time and may continue even after removal.

Therefore, any suspected case of esophageal battery impaction warrants immediate removal.

Battery size appears to be important as a battery diameter >20 mm has been associated with greater risk of esophageal impaction and higher grade injury.

Following removal, an intraoperative esophagram may be helpful in identifying a full-thickness injury.

Mucosal irregularities and even contained perforations can be seen and may necessitate enteral tube feedings.

Early and late complications of esophageal battery impaction include esophageal perforation, tracheoesophageal fistula, stricture and stenosis, and death.

If the battery is confirmed to be distal to the esophagus in the GI tract and the patient is asymptomatic, then it can be observed, similar to other GI FBs.

More than 80% of batteries that are distal to the esophagus will pass uneventfully within 48 hours.

An updated management algorithm for ingested batteries was recently published.

How well did you know this?
1
Not at all
2
3
4
5
Perfectly
9
Q

How should you manage magnet ingestion?

A

Magnet ingestion can be another source of significant morbidity when multiple magnets or a single magnet and a second metallic FB are ingested simultaneously, or within a short time of each other.

At presentation, fewer than 40% of these patients are symptomatic, with the most common symptom being abdominal pain.

Plain radiographs are most commonly used to confirm the diagnosis. However, radiographs should be interpreted with caution because multiple magnets may appear to be attached at a single point in the GI lumen when, in fact, they are really attached across the bowel wall from two different intestinal lumens.

Therefore, once the ingestion is confirmed on radiographs, close observation for potential complications is important.

Single magnet ingestion distal to the esophagus can be observed in the outpatient setting similar to other GI foreign bodies.

If multiple magnets or a single magnet and a second metallic FB are identified in the esophagus or stomach, endoscopy should be performed to prevent potential subsequent complications.

Once the objects pass distal to the stomach, if separated within the GI tract, they may attach to each other and lead to obstruction, volvulus, perforation, or fistula through pressure necrosis. Therefore, these children, even if asymptomatic, should be observed as an inpatient with serial abdominal exams and radiographs.

If the child becomes symptomatic, develops signs of obstruction on the abdominal radiograph, or shows failure of the objects to progress in 48 hours, then intervention may be warranted.

A management algorithm for ingested magnets was also recently published.

How well did you know this?
1
Not at all
2
3
4
5
Perfectly
10
Q

How should you approach ingestion of sharp foreign bodies?

A

Ingestion of sharp foreign bodies can cause significant morbidity with an associated 15–35% risk of perforation.

Commonly ingested objects include nails, needles, screws, toothpicks, safety pins, and bones.

Perforation is most likely to occur in narrowed portions or areas of curvature in the alimentary tract, especially the ileocecal valve.

Smaller objects and straight pins are associated with lower rates of perforation and can be conservatively managed.

However, other objects should be retrieved endoscopically if possible or observed closely for potential development of complications.

How well did you know this?
1
Not at all
2
3
4
5
Perfectly
11
Q

What are bezoars?

A

A bezoar is a tight collection of undigested material that may often present as a gastric outlet or intestinal obstruction.

These can include lactobezoars (milk), phytobezoars (plant), or trichobezoars (hair).

Presenting symptoms can include nausea, vomiting, weight loss, and abdominal distention.

The diagnosis may be confirmed on plain radiographs, upper GI contrast studies, or endoscopy.

Often due to the size and density of the bezoar, medical management and endoscopic removal are unsuccessful, and operation is necessary.

How well did you know this?
1
Not at all
2
3
4
5
Perfectly
12
Q

What is the “push” vs “pull” method for food impaction?

A

Historically, children presenting with food impaction were taken to the OR for a piecemeal removal of the impaction.

This “pull” technique would often require multiple passes of the endoscope, thus increasing patient morbidity.

More recently, the “push” technique has been studied where a provider would slowly “push” the FB into the stomach.

With proper technique, the historical concerns over distal perforation have been ameliorated.

In a recent study, initial endoscopic disimpaction success rates were 65% for pull and 68% for push endoscopy.

Unsuccessful attempts using one technique were rescued with the other method.

How well did you know this?
1
Not at all
2
3
4
5
Perfectly
13
Q

Why are children prone to airway foreign bodies?

A

Most episodes of aspirated FBs occur while eating or playing.

Appropriate development milestones render children curious who are still in the oral exploration phase of development when everything tends to go into the mouth.

Additionally, children often will cry or run with objects in their mouth.

Overall, these young patients tend to have immature coordination of swallowing and less developed airway protection.

A high index of suspicion is required to make the diagnosis in these young children and especially in those who are debilitated.

A recent retrospective multi-institutional report on bronchoscopy for FBs from the Pediatric National Surgical Quality Program from 2014–2015 in 334 children noted the mean operative time to be 27 minutes.

As expected, the operative time was longer when foreign bodies were in the mainstem bronchus or distal to it.

There was one patient death within 2 weeks of the bronchoscopy.

The annual death rates from FB aspiration is significant with estimates ranging from 220 to 2900 in the United States.

Current data predicts one death per 100,000 children ages 0 to 4 years.

Death caused by suffocation following a FB aspiration is the leading cause of mortality from unintentional injury in children younger than 1 year, and is overall the fifth most common cause of unintentional injury mortality in U.S. children.

Boys are affected twice as often as girls.

Like esophageal FBs, geographical differences have been noted. For example, sunflower seeds are the most common seed aspirated in the United States, yet watermelon seeds are much more common internationally.

One report involving 132 cases noted a high incidence of food aspirations, especially nuts, in children from non-English-speaking backgrounds.

Therefore, there may be a role for public education in targeted communities.

Victims of child abuse represent another community that is at higher risk.

Caregivers should be on alert when tending to a young child with multiple FBs or multiple episodes of aspiration.

Unfortunately, the public health and economic burdens of airway foreign bodies is increasing. In a recent review of the KID database from 2000–2009, charges increased from $93M to $486M in the observed period with charges higher in urban and teaching hospitals.

How well did you know this?
1
Not at all
2
3
4
5
Perfectly
14
Q

What are the differences between the airway of younger versus that of older children?

A

Several anatomical differences are found in the airway of young children compared with older children.

Young children have a shorter airway that is smaller in caliber.

The anterior position of a child’s larynx can increase the difficulty with oral intubation.

Additionally, the subglottic region is the narrowest part of a child’s airway.

The proclivity for FBs to find the right main stem bronchus is well known. Not only is the diameter of the right bronchus larger than the left and airflow generally greater to the right lung, but also the right bronchus has a smaller angle of divergence from the trachea.

This important anatomical feature seems to direct the aspirated FB down into the right bronchus.

How well did you know this?
1
Not at all
2
3
4
5
Perfectly
15
Q

When should airway foreign body obstruction be suspected in a child?

A

Common presenting symptoms include respiratory distress, stridor, and/or wheezing.

Dysphonia may also be observed.

A subtle change in voice or cry may be noted, yet many children will be asymptomatic.

Many aspiration events go unwitnessed.

Laryngeal pathology usually will manifest as inspiratory stridor while tracheal FBs cause expiratory stridor.

Albeit rare, FBs may completely obstruct the larynx or trachea producing sudden death.

Chronic FBs often masquerade as respiratory illnesses with persistent cough and atelectasis, recurrent pneumonia, or hoarseness.

Other late findings include the development of granulation tissue, strictures, perforation, and bronchiectasis.

Following a detailed history, investigation usually turns to AP and lateral films of the neck and chest. If the child is cooperative, inspiratory and expiratory films may be beneficial. Review of the radiograph can reveal hyperinflation or “air trapping” in up to 60% of children as the FB is acting as a one-way valve producing obstructive emphysema.

In time, mediastinal shift may develop.

Decubitus views may also prove helpful because the obstructed lung will not deflate, even while in a dependent position.

Interestingly, in one study, 56% of patients had a normal chest film within 24 hours of aspiration.

Radiopaque FBs are easily identified, but radiolucent FBs become clinically diagnosed through indirect radiographic clues such as hyperexpansion.

Foreign bodies lodged in the larynx or trachea tend to have a higher radiographic detection rate (90%) than those in the bronchus (70%).

A multi-institutional review of 1269 FB events revealed that 85% were correctly diagnosed following a single physician encounter. Therefore, a negative bronchoscopy rate of 10–15% is considered acceptable in order to avoid a delay in treatment with subsequent morbidity.

Radiographic imaging remains helpful in children with a history of choking, yet definitive diagnosis still requires bronchoscopy.

Emergent management of airway FBs can be a dramatic experience. An accurate history remains important, yet sometimes is hard to obtain from small children who are unreliable historians.

The use of the flexible bronchoscope to diagnose a FB followed by a rigid bronchoscopy for removal is a common approach utilized by pediatric surgeons.

General anesthesia in the OR using spontaneous ventilation offers the best chance for safe and successful removal.

Direct and frequent communication with our anesthesia colleagues is imperative as many children will be at risk from bronchospasm, especially if repeated passage of the bronchoscope is needed.

Positive-pressure ventilation may be required, but this technique runs the risk for further propagating the FB into the more distal passages of the airway.

With severely ill children, where transportation presents a logistical risk, rigid bronchoscopy has been performed safely in the intensive care setting.

How well did you know this?
1
Not at all
2
3
4
5
Perfectly
16
Q

How is bronchoscopy performed for airway foreign bodies?

A

We recommend positioning the head in the “sniffing” position with a folded towel under the shoulders. The eyes are taped and protected.

Precautions to minimize secretions, laryngospasm, and hypoxia are employed.

Careful laryngoscopy may reveal a FB that can be retrieved with McGill forceps. More distal evaluation requires direct instrumentation of the airway.

Special precautions must be considered to avoid injuries to the lips, tongue, and most importantly the teeth.

Once the bronchoscopy starts, the operative team must be ready for emergent intubation, or rarely, tracheostomy.

Liberal use of lidocaine (2–4 mg/kg) applied to the glottic area may minimize laryngospasm.

There are several commercial available rigid bronchoscopes. Instruments vary in size between 2.5 cm × 20 cm and 6 × 30 cm. Length and diameter of the bronchoscope will be determined by the age and size of the child.

The Doesel–Huzly bronchoscope with a Hopkins rod-lens telescope or Holinger ventilating bronchoscope is commonly used. Exposure is excellent with both, and the caliber of the scope allows the FB to be retracted into the scope during removal, thereby decreasing the risk of inadvertently dropping the FB during extraction.

Equipment combining optics and illumination while allowing the introduction of working forceps are favored in most children’s hospitals.

These techniques allow for excellent visualization while educating all team members.

The larynx and cords are visualized, and the bronchoscope is advanced to the right of the laryngoscope and into the trachea.

Inspection of the right or left main stem bronchus can be facilitated by turning the head to the opposite side. Angled scopes are generally not needed.

The operative side channel allows passage of suction and retrieval instruments as well as instillation of fluids to help clear a bloody airway.

The ventilation side port allows continuous ventilation during the procedure.

Loose connections from any of these sites can lead to hypoventilation.

Furthermore, if ventilation is impaired, the telescope can be removed, leaving the unobstructed bronchoscope for ventilation.

In difficult cases, especially with FBs lodged distal to the main bronchus, a Fogarty catheter may be helpful to wedge the FB between the bronchoscope and Fogarty balloon.

Partial FB removal will at times be necessary, especially with chronic foreign bodies associated with significant bleeding or airway edema. Prior to the second endoscopy, the child’s condition can be optimized with inhaled epinephrine and intravenous corticosteroids. Most children can return to the OR the following day.

Flexible bronchoscopy remains an option, especially for diagnostic purposes. The standard pediatric flexible bronchoscope has a two-way deflection tip with a range between 180° and 220° and a side port to allow passage of suction catheters and working instruments.

Most newborns can breathe normally around this scope for brief periods of time. A face mask adapter can be used to reduce the risk of hypoxia.

The ultrathin scope (“noodle scope”) can be inserted through smaller caliber endotracheal or tracheostomy tubes while maintaining ventilation. However, these scopes have very limited, if any, working channels and suction capability.

Overall complications of rigid or flexible endoscopy include bleeding from local inflammation, laryngospasm, pneumothorax, and hypoxia, with the more serious complications being found in the youngest patients.

A lack of experience, poor visualization, and inadequate instrumentation contribute to failure with bronchoscopy. Many of these cases are seen at night, and OR personnel frequently struggle to locate the required instruments. The surgeon performing the procedure must ensure that all of the needed instruments are in working order before the child is brought to the operating room.

A preoperative “game plan” between nursing, anesthesia, and surgical staff is also important.

Bleeding that obscures visualization is common, and the introduction of a small suction catheter through the working channel may be helpful.

In other cases, partial FB removal is recommended with a plan to return to the OR the next day. Rarely a thoracotomy with bronchotomy or lobectomy is required.

Following successful removal, attention to proper cleaning of the instruments is important, given that inadequate cleaning and storage may predispose to cracks in the equipment that could lead to bacterial contamination.

Maintaining bronchoscopic skills remains necessary to effectively manage these difficult cases. Across undergraduate and graduate medical education, simulation continues to gain momentum. In a setting of low procedural volume, simulation with deliberate practice fosters competency. Several institutions have developed simulation courses for residents to improve psychomotor skills associated with airway FBs. Objectives include an understanding of the tracheobronchial anatomy, ability to adequately visualize the larynx with laryngoscopy, proficiency in rigid bronchoscopy, and familiarity with FB instrumentation. Of note, in one institution that held a simulation course among otolaryngology residents, success in assembling the needed instruments and completing the assigned tasks increased to 81% and 43%, respectively, at the completion of the course. These simulators have offered value and relevance to experienced and novice practitioners while still adapting for material response. One would expect further development of simulation exercises to be useful for credentialing and continued proficiency for both trainees and faculty.

How well did you know this?
1
Not at all
2
3
4
5
Perfectly
17
Q

What is most common source of bite wounds?

A

Dogs are the most common source of bite wounds, costing over $1 billion per year in the United States alone.

Bites occur most frequently in males ages 5–9 years of age when compared with bites in adults.

The increased incidence in children has been attributed to their smaller size, lack of awareness, and playful manner when interacting with pets, leading these pets to attack them, often in a playful manner.

Bites more frequently occur in the summer months by animals that are well known to the child and frequently from a pet owned by an immediate family member.

Children under 10 years of age are most frequently bitten on their head or neck while older children are more frequently bitten on their distal extremities.

Based on a 1-year telephone survey of over 5000 randomly dialed households to estimate the incidence of dog bites and those seeking treatment, there were an estimated 377,000 children who sought treatment after dog bites in the United States, which is twice the number of adults seeking treatment for dog bites.

One recent study looked at 1017 bite wounds in children and found that bites occur most frequently in children younger than 5 years old and decreased in incidence throughout adulthood. In this series, 66% of the bites were to the head and neck region. Younger children experienced more head or neck wounds, and older children and adults experienced more extremity injuries.

The majority (72%) were from dogs belonging to family or friends.

Sixty-seven percent of patients were sutured in the emergency department, 25% underwent local wound care, and 6% required operative repair.

The authors noted the stability in the epidemiology of bite wounds over the past five decades and the need for improved preventive strategies.

In another study, a 5% hospital admission rate was found among a cohort of 1347 dog bites in children and another 4% were observed for 23 hours.

The mean length of stay was 7 days for children younger than 5 years old, and 3–4 days for children 5–18 years.

On multivariate regression, risk factors for admission were a bite to the head or neck, whereas age was not a significant factor.

The median charge was $300 for those seen in the emergency room, $3600 for those admitted to an observation unit, and $5900 for those who were hospitalized.

In another study of 769 dog bite victims prospectively evaluated over a 2-year period, evidence of a superficial wound infection was present in 2.5% of patients on presentation and in 2.1% of patients at follow-up.

Predictors of developing an infection included a deep wound, female gender, and wounds requiring debridement.

A recent multicenter prospective study reviewed 495 patients with dog bites, 18 of which developed infections. They found puncture wounds and wounds closed primarily to be at increased risk of becoming infected and recommended prophylactic antibiotics for these patients.

At our institution, a level I pediatric trauma center, 69 children admitted for treatment of dog bites were reviewed for this chapter. Of these, 94% required operative debridement and laceration repair. We found that 98% of patients under 5 years of age presented with a bite to the head and neck region, compared with 48% in patients older than age 5 years.

Severe injuries, such as a tracheal laceration, penetrating cranial laceration, cervical spine injury, foot amputations, and vascular lacerations, were also seen. Although all were treated with antibiotics, 17% displayed signs of wound infection during or after their hospital stay.

Although there were no deaths in our series, dog bites in children can be fatal. Approximately 25–35 patients die each year from dog bites, the majority being children. The deaths are usually from bites to the head, neck, or vital organs.

Pitbulls in particular are a common dog breed that bite and cause fatal bites.

How well did you know this?
1
Not at all
2
3
4
5
Perfectly
18
Q

Why do cat bites manifest with infection more rapidly than dog bites?

A

Cats are the second most common source of bites, with an estimated incidence of 400,000 per year.

One study looking at 643 cat and dog bites reported to the El Paso Animal Regulation and Disease Control Center revealed that 89% of cat bites were provoked. Interestingly, in this study, only 45% of dog bites were provoked.

Females and adults seem more likely to be victims of cat bites, whereas dog bites may be more common in males and children.

One retrospective review found that 65% of cat bite wounds occurred on the upper extremities, 20% on the head and neck, 10% on the lower extremities, with fewer than 5% on the trunk.

Cats have long, narrow teeth that are more likely to penetrate bones and joints, leading to deep abscesses and osteomyelitis.

In one large retrospective study looking at 1592 patients presenting after mammalian bites, the infection rates for dog bites was 15% and for cat bites was 37%.

In this study, cat bites manifested signs of infection more rapidly when compared with dog bites (12 vs 24 hours).

How well did you know this?
1
Not at all
2
3
4
5
Perfectly
19
Q

Why do human bites have a higher complication rate than animal bites?

A

The incidence of human bites is difficult to determine because many patients do not seek medical care.

Human bites generally compress tissue but rarely avulse the skin and soft tissue like animal bites.

A “fight bite” occurs when a fist hits a tooth and is often dismissed by patients. These occur from clenched fists and frequently involve the distal phalanx of the long or index finger of the dominant hand.

However, these fight bites can cause significant morbidity due to infection.

Human bites have a higher complication and infection rate compared with animal bites. Infections from human bites frequently contain aerobic and anaerobic bacteria, and antibiotics need to cover for these anaerobes.

Coverage for Staphylococcus, Streptococcus, and Eikenella, as well as anaerobic organisms such as Fusobacterium, Prevotella, and Veillonella is needed.

In a multicenter, prospective study of 50 patients with infected human bites, the median number of bacteria isolated per wound culture was 4 (3 aerobes and 1 anaerobe).

Streptococcus anginosus was the most common (52%).

Bites to the hands are often deep and are more likely to become infected compared with bites to other areas.

How well did you know this?
1
Not at all
2
3
4
5
Perfectly
20
Q

When is secondary intention for closure of bite wounds indicated?

A

Management of bite wounds involves the basic tenets of wound care, including wound irrigation and debridement of any necrotic tissue.

A large-bore blunt needle connected to a syringe can provide adequate pressure to clean these wounds.

Radiographic studies should be obtained when there is a concern for bone or joint penetration, and the tetanus immunization status should be investigated.

Rabies prophylaxis is important when indicated.

In one study the most common complication from bites was infection: cats, 16–50%; dogs, 1–30%; and humans, 9–18%.

When the bite is from a cat, dog, or other mammal, the most common infectious organisms are Streptococcus, Staphylococcus, Actinomycetes, Pasteurella species, Capnocytophaga species, Moraxella species, Corynebacterium species, Neisseria species, Eikenella corrodens, Haemophilus species, anaerobes, Fusobacterium nucleatum, and Prevotella melaninogenica.

Human bites are a potential source not only for bacterial contamination but also for hepatitis B and, possibly, human immunodeficiency virus (HIV) infection.

Recommendations for management of a bite wound are shown in Box 12.1.

Evidence-based studies concerning whether to close wounds are not conclusive. Prospective trials, looking at primary closure of bites, have shown low rates of infection after primary closure, and two randomized controlled studies comparing primary closure to nonclosure showed no difference in infection rates after primary closure. In these studies, primary closure improved cosmesis and reduced healing times.

Wounds to the distal extremities and puncture wounds have been shown to have a higher rate of infection, and some providers recommend not closing distal puncture wounds or wounds that have been open for more than 6–12 hours before washout.

Additionally, consideration could be given to not closing wounds that are visibly infected, or closing them loosely over a Penrose drain or vessel loop.

Facial wounds have a low rate of infection due to the rich blood supply, and many surgeons advocate for closing these lacerations after irrigation and debridement.

Whether wounds at minimal risk require prophylactic antimicrobial therapy is also controversial. Antibiotics started within 8–12 hours of the bite and continued for 2–3 days may decrease the infection rate.

The oral drug of choice is amoxicillin-clavulanate. For penicillin-allergic patients, an extended-spectrum cephalosporin or trimethoprim-sulfamethoxazole plus clindamycin can be used.

It is possible to reduce the incidence of animal bites through anticipatory guidance. Guidelines are available from the Centers for Disease Control and Prevention (CDC) (https://www.cdc.gov/features/dogbite-prevention/index.html) to aid physicians in counseling parents.

How well did you know this?
1
Not at all
2
3
4
5
Perfectly
21
Q

Which injuries are most prone to tetanus infection?

A

The Gram-positive anaerobic organism Clostridium tetani is the causative agent for tetanus, a severe and often fatal disease. In 2009 there were a total of 18 cases (zero under 14 years of age) reported in the United States.

There has been a low incidence rate of tetanus since a peak of 102 cases in 1975. Mortality from tetanus is associated with comorbid conditions such as diabetes, intravenous drug use, and old age, especially when the patient’s vaccination status is unknown.

Infection can occur weeks after a break in the skin, even after a wound appears to be healed.

The anaerobic environment allows spores to germinate into mature organisms producing two neurotoxins: tetanolysin and tetanospasmin. The latter is able to enter peripheral nerves and travel to the brain, causing the clinical manifestations of uncontrolled muscle spasms and autonomic instability.

The incubation period varies from as short as 2 days to several months, with most cases occurring within 14 days.

In general, the shorter the incubation period, the more severe the disease is and the higher the risk of death.

Initially, the diagnosis is made clinically because cultures are often negative and serology for antitoxin antibodies takes a long time to process.

So-called dirty wounds (lacerations treated after 24 hours, abscesses, ulcers, gangrene, and wounds with nonviable tissue) are the most common injuries that become infected with tetanus.

However, a history of trauma is not necessary for tetanus infection.

All wounds should be cleaned and debrided. Symptomatic and supportive care includes medications such as benzodiazepines to control tetanic spasms and antimicrobials for infection.

Metronidazole (oral or intravenous, 30 mg/ kg/day, divided into four daily doses, maximum 4 g/day) is the preferred antibiotic because it decreases the number of vegetative forms of C. tetani.

An alternative choice is parenteral treatment with penicillin G (100,000 units/kg/ day every 4–6 hours, not to exceed 12 million units/day) for 7–10 days.

Human tetanus immunoglobulin (TIG) is administered to adults and adolescents as a one-time dose of 3000–6000 units intramuscularly.

Some authors recommend that children receive 500 units to decrease the discomfort from injection.

Infiltrating part of the dose directly into the wound is controversial.

Immunoglobulin intravenous (IGIV) can be used at dose 200 to 400 mg/kg if TIG is not available.

Tetanus prevention in a potentially exposed patient depends on the nature of the wound and history of immunization with tetanus toxoid.

How well did you know this?
1
Not at all
2
3
4
5
Perfectly
22
Q

What prophylactic treatment is advised for humans potentially exposed to rabies?

A

Rabies is a viral disease usually transmitted through the saliva of a sick mammal (dogs, cats, ferrets, raccoons, skunks, foxes, bats, and most other carnivores).

Small rodents such as rats, mice, squirrels, chipmunks, hamsters, guinea pigs, rabbits, and gerbils are almost never infected with rabies.

The rabies virus enters the central nervous system and causes an acute, progressive encephalomyelitis from which survival is extremely unlikely.

The human host has a wide range for the incubation period from days to years (most commonly weeks to months).

As a result of canine vaccination programs and stray animal control, there has been a marked decrease in rabies in the United States in recent decades.

In 2006, 79 cases were reported in domestic dogs and only 3 in humans, but none were transmitted from the domestic dogs.

Each year in the United States, approximately 16,00039,000 people come into contact with potentially rabid animals and receive rabies postexposure prophylaxis.

Prophylactic treatment for humans potentially exposed to rabies includes immediate and thorough wound cleansing followed by passive vaccination with human rabies immunoglobulin and cell culture rabies vaccines, either human diploid or purified chick embryo.

Many factors determine the risk assessment in deciding which patient benefits from postexposure prophylaxis and which regimen should be given.

The risk of infection depends on the type of exposure, surveillance, epidemiology of animal rabies in the region of contact, species of animal, animal behavior causing it to bite, and the availability of the animal for observation or laboratory testing for the rabies virus.

The final decision for treatment with vaccines is complex. Therefore, local, state, or CDC experts are available for assistance. There is no single effective treatment for rabies once symptoms develop.

How well did you know this?
1
Not at all
2
3
4
5
Perfectly
23
Q

Which are the two most medically important spiders in the US?

A

There are about 40,000 species of spiders that have been named and placed in about 3000 genera and 105 families.

In regard to medically relevant spiders, few are known to cause significant clinical effects. It is rare that a spider bite requires surgical care.

Few spiders have been shown to have the ability to bite humans because their fangs cannot pierce the skin.

The two most medically important spiders in the United States are Sicariidae (brown spiders) and Latrodectus (widow spiders).

How well did you know this?
1
Not at all
2
3
4
5
Perfectly
24
Q

How do you diagnose a brown recluse spider envenomation?

A

Loxoscelism is a form of cutaneous–visceral (necroticsystemic) arachnidism found throughout the world, with a predilection for North and South America.

There are four species of brown spiders within the United States known to cause necrotic skin lesions (Loxosceles deserta, L. arizonica, L. rufescens, and L. reclusa). L. deserta and L. arizonica can be found in the southwestern United States.

L. reclusa, known as the brown recluse spider, is the most common species associated with human bites. It is usually found in the south-central United States, especially Missouri, Kansas, Oklahoma, Arkansas, Tennessee, and Kentucky.

Spiders can be transported out of their natural habitat but rarely cause arachnidism in nonendemic areas.

L. reclusa is tan to brown with a characteristic dark, violin-shaped marking on its dorsal cephalothorax, giving it the nickname “fiddleback” or “violin” spider. The spider can measure up to 1 cm in total body length with a 3-cm or longer leg span. These spiders have only three pairs of eyes, whereas most spiders have four pairs.

The incidence of L. reclusa bites predominantly occurs from April through October in the United States.

The venom of the brown recluse spider contains at least 11 protein components. Most are enzymes with cytotoxic activity. Sphingomyelinase D is believed to be the enzyme responsible for dermonecrosis and activity on red blood cell membranes.

In addition to the local effects, the venom has activity against neutrophils and the complement pathway that induces an immunologic response. The resulting effect is a necrotic dermal lesion and the possibility that a systemic response from the child or adult will be life threatening.

The prevalence of brown recluse spider envenomations is unknown. The victim may not feel the bite or may feel only a mild pinprick sensation. Many victims are bitten while they sleep and may be unaware of the envenomation until a wound develops. The majority of victims do not see the spider at the time of the bite.

Typically, the bite progressively begins to itch and tingle and becomes ecchymotic, indurated, and edematous within several hours. Often within hours, a characteristic bleb or bullae will form. The tissue under the blister is likely to become necrotic, but the extent of necrosis is not predictable.

As the ischemia and inflammation progresses, the wound becomes painful and may blanch or become erythematous, forming a “target” or “halo” design. Inflammation, ischemia, and pain increase over the first few days after the bite as the toxic enzymes spread.

Over hours to weeks, an eschar forms at the site of the bite. Eventually, this eschar sloughs, revealing an underlying ulcer that may require months to heal, usually by secondary intention. On rare occasions, the ulcer does not heal and requires surgical intervention.

The need for hospitalization occurs if the patient develops systemic symptoms.

The true incidence of systemic loxoscelism is unknown because a spider is identified in a minority of cases. Thus, there is often insufficient proof that the symptoms are related to the spider bite.

However, one study showed the utility of enzyme-linked immunosorbent assay (ELISA) to confirm the Loxosceles venom in a patient with hemolytic anemia when no spider was identified.

Other common symptoms include a maculopapular rash, nausea and vomiting, headache, malaise, muscle and joint pain, hepatitis, pancreatitis, and other organ toxicity. Lifethreatening systemic effects include hemolysis (intravascular and/or extravascular), coagulopathy, and multiple organ system failure. Secondary effects include sepsis, necrotizing fasciitis, and shock.

Hemolysis usually manifests within the first 96 hours. However, late presentations also can occur. When hemolysis does develop, it can take 4–7 days (or longer) to resolve.

Complications such as cardiac dysrhythmias, coma, respiratory compromise, pulmonary edema, congestive heart failure, renal failure, and seizures can occur.

The diagnosis of a brown recluse spider envenomation is largely one of exclusion because it is rare to see or identify the spider. Although the wound can appear classic for an envenomation, other causes must be considered.

Certain laboratory findings can be consistent with a brown recluse spider envenomation but are not specific in making the diagnosis.

How well did you know this?
1
Not at all
2
3
4
5
Perfectly
25
Q

What lab findings are consistent with Loxosceles envenomations?

A
How well did you know this?
1
Not at all
2
3
4
5
Perfectly
26
Q

What are treatment options for loxoscelism?

A

Controversy surrounds the treatment of dermal and systemic symptoms of loxoscelism. Medications such as dapsone, nitroglycerin, and tetracycline have been used. In addition, hyperbaric oxygen (HBO) therapy has been advocated as has excision of the necrotic wound. However, none of these treatment options has proved effective in treating or preventing the development of an ulcer.

In South America, an antivenom has been developed and used in the treatment of Loxosceles envenomations. Unfortunately, the usual long delay in seeking medical care often leads to ineffective use of this antivenom.

An antivenom is not available in North America.

The use of dapsone, a leukocyte inhibitor, has been advocated in case reports and animal studies. However, other animal studies have shown no benefit from this treatment.

In one animal study, piglets received venom and were randomized to receive one of four treatments: no treatment, HBO, dapsone, or dapsone with HBO. Neither dapsone, HBO, nor the combination treatment reduced necrosis compared with controls. Another study compared the use of HBO, dapsone, or cyproheptadine against no treatment in decreasing the necrotic wound after envenomation with L. deserta venom. No statistical difference was seen with respect to lesion size, ulcer size, or histopathologic ranking. In addition, the use of dapsone is not without risk because hypersensitivity reactions can occur.

Therapeutic doses of dapsone are associated with hemolytic anemia, methemoglobinemia, and other hematologic effects in patients with and without glucose-6-phosphate dehydrogenase deficiency.

Topically applied nitroglycerin as a vasodilator has been advocated in the past, but is not effective in preventing necrosis.

Tetracycline was shown to be effective in a rabbit study.

Rabbits were inoculated with Loxosceles venom and randomized to receive topical doxycycline, topical tetracycline, or placebo. Those who received topical tetracycline had reduced progression of the dermal lesion. However, the treatment was started at 6 hours after envenomation, which may not be realistic after a human bite. In addition, the agents used for this research study are not commercially available in the United States. Further studies need to be performed before topical tetracycline can be recommended.

HBO has been advocated for treatment to prevent progression of the necrotic wound. The initial use of HBO was based on the belief that tissue hypoxia was partially responsible for the subsequent necrosis that occurred after a bite. As mentioned previously, no statistical differences were noted in animal studies that compared dapsone and HBO. Similar results have been seen in animal studies assessing the effect of HBO alone. A randomized, controlled trial of HBO in a rabbit model in which standard HBO was used showed a significantly reduced wound diameter at 10 days. However, there was no significant change in blood flow at the wound center or 1–2 cm from the wound center. Notably, HBO is expensive and can lead to complications.

At the present time, much of the literature contradicts the benefit of HBO for brown recluse spider envenomations. Thus, it is not currently recommended as a therapy for these bites, but may be helpful in patients with underlying or preexisting vascular compromise such as sickle cell anemia or diabetes.

Early surgical intervention is not helpful because the venom diffuses rapidly throughout the soft tissues surrounding the bite. In addition, patients may be more at risk for delayed wound healing and excessive scarring if the operation occurs within the first 72 hours of the bite.

Debridement of enlarging blebs is proposed with the theory that toxins exist within the blister fluid. However, necrosis almost always occurs beneath the blisters.

The question is whether surgical intervention should be advocated late after envenomations.

The wound from the brown recluse spider may take 2–3 months to heal. Thus, skin grafting of a nonhealing necrotic area should be delayed up to 12 weeks to allow for neovascularization of the demarcated area.

Treatment of systemic symptoms largely involves supportive care. Patients should be monitored closely for hemolysis, and children may need hospitalization if systemic symptoms such as fever and rash develop.

Systemic corticosteroids seem to suppress hemolysis and may be needed for 5–10 days with a subsequent tapering dose. Methylprednisolone can be administered as a 1–2 mg/kg intravenous loading dose (no maximum) followed by a 0.5–1 mg/ kg maintenance dose every 6 hours.

Hydration to maintain good urine output is required to prevent acute renal tubular necrosis if hemolysis or hematuria occurs.

Antibiotics are not generally required early in the care of these patients because the spider does not inoculate humans with bacteria. However, secondary infections can occur and lead to sepsis, toxic shock syndrome, and necrotizing fasciitis.

These complications require close observation and antibiotic therapy to cover anaerobic, staphylococcal, and streptococcal infections.

How well did you know this?
1
Not at all
2
3
4
5
Perfectly
27
Q

When should you suspect envenomation from black widow spiders?

A

Black widow spiders (Latrodectus mactans) are found throughout North America. They can usually be found outdoors in warm, dark places or in a garage or basement. They are web-making spiders and usually strike when their web is disturbed.

The female spider is readily recognized because she is a black spider with a red marking on her abdomen in the shape of an hourglass.

Widow spiders have a neurotoxic venom that is responsible for their clinical effects. The venom, α-latrotoxin, acts on the neuromuscular junction to cause depletion of acetylcholine at the motor endings and catecholamines at the postganglionic sympathetic synaptic sites, which is followed by complete blockade of the neuromediator release.

In the majority of cases, a pinprick sensation is felt at the time of a bite. A “halo” lesion may develop, but this tends to disappear within 12 hours of envenomation.

A few hours after the bite, the regional lymph nodes and affected extremity may become tender.

Depending on where the bite occurs, pain usually migrates to the large muscle groups in the thigh, buttock, abdomen, or chest.

The most common presenting complaint is intractable abdominal, chest, back, or leg pain, depending on the site of the bite.

The pain generally peaks at 2–3 hours, but can last up to 72 hours.

Board-like rigidity of the abdomen, shoulders, and back can develop that may lead to the misdiagnosis of a surgical abdomen or other etiology.

Because the venom affects the autonomic nervous system, patients can present with symptoms of dysautonomia that include hypertension (sometimes severe), tachycardia, weakness, ptosis, eyelid edema, pruritus, nausea and vomiting, diaphoresis, hyperreflexia, difficulty breathing, and excessive salivation.

Fatalities are rare, but can occur.

Children are more at risk for developing systemic symptoms.

How well did you know this?
1
Not at all
2
3
4
5
Perfectly
28
Q

How are black widow spider bites treated?

A

Management is largely symptomatic and supportive.

For the most part, treatment is focused on analgesia.

For those with mild pain, oral analgesics are appropriate.

Patients may present with severe pain requiring opioids and benzodiazepines as adjunctive therapy.

Calcium gluconate was advocated in the past, but is not recommended now because of the lack of consistent effects in alleviating the symptoms.

Antivenom is available and generally reserved for patients who have life-threatening symptoms or for pain that is not relieved by opioids and benzodiazepines.

How well did you know this?
1
Not at all
2
3
4
5
Perfectly
29
Q

How can you distinguish pit vipers from nonpoisonous snakes?

A

The World Health Organization (WHO) estimates there are up to 1.8 million bites worldwide from venomous snakes annually, causing 20,000–90,000 deaths. In the United States, several thousand snakebites occur every year, resulting in fewer than 10 deaths.

Despite the relatively low incidence of bites and exceptionally low mortality in the United States, snakebites generate much attention and fear.

Approximately 95% of all U.S. snake envenomations are caused by crotalids, also known as pit vipers. Pit vipers include rattlesnakes, copperheads, and water moccasins, which are also known as cottonmouths.

Pit vipers are characterized by large, triangular heads; relatively small eyes; large, retractable fangs; and a thermoreceptor “pit” located approximately halfway between the eye and the nostril.

Pit vipers also have a single row of subcaudal plates distal to the anus. In addition, rattlesnakes will have one or more keratin buttons that compose the “rattle” at the distal end.

Though some references recommend using the pupil shape as a way of distinguishing a pit viper from noncrotalids, it should be noted that all snakes can have round or elliptical pupils, depending on the amount of ambient light.

Additionally, many nonvenomous snakes can flatten their heads into a triangle shape when they feel threatened.

Coral snakes, which are not crotalids but rather elapids, are also venomous, and they lack the characteristic features of pit vipers. Coral snake envenomations, which account for a small percentage of snake bites in the United States, will be discussed in a subsequent section.

Snakebites affect all age groups. Between 2012 and 2015, approximately 24% of snakebites occurred in children, including 5% in kids younger than 6 years old, 9% in children ages 6–12 years, and 10% in teenagers.

Contrary to popular belief among herpetologists and other snake enthusiasts, most snakebites do not occur in the occupational setting or in the context of intentionally interacting with the snake. The majority of envenomations occur after the victim accidentally steps on a snake or reaches into an area where the snake is located.

However, intentionally teasing or handling a snake is a risk factor for envenomation.

How well did you know this?
1
Not at all
2
3
4
5
Perfectly
30
Q

Which components of pit viper venom cause disease?

A

Pit viper venom comprises a variety of protein and nonprotein substances.

Most crotalid venom contains a mixture of metalloproteinases, collagenase, phospholipase, and hyaluronidase that can cause myonecrosis and dermatonecrosis.

Multiple venom components (e.g., serine proteases, disintegrins, metalloproteinases, and C-type lectin-like proteins) produce a variety of hematologic effects, resulting in coagulopathy, platelet aggregation, activation or inhibition, or increased coagulation leading to thrombotic complications.

Certain crotalid species have unique toxins. Crotalocytin, found in the Timber rattlesnake (Crotalus horridus), causes platelet aggregation.

Mojave toxin, found in some populations of the Mojave rattlesnake (C. scutulatus), inhibits the presynaptic release of acetylcholine, leading to weakness and paralysis.

Pit viper venom also may include bradykinin-related peptides that can lead to angioedema and hypotension.

Pit vipers are solenoglyphous and have hollow, mobile and relatively long fangs capable of delivering venom quite efficiently. However, not all bites result in an envenomation.

Approximately 10–15% of crotalid bites are “dry,” meaning there is no envenomation.

A number of factors related to both the snake and the snakebite victim determine the severity of the envenomation. These include the species, age, size, and overall health of the snake as well as its diet and the last time it had fed or released venom.

The age and health of the patient as well as the location of the bite will also affect the severity of the bite.

How well did you know this?
1
Not at all
2
3
4
5
Perfectly
31
Q

How does crotalid envenomation present?

A

The most consistent finding in crotalid envenomation is local tissue injury, which is present in more than 90% of cases.

Fang marks are not always obvious, and often only one fang is responsible for the envenomation.

The distance between puncture wounds is not a reliable way of estimating the size of the snake because pit viper fangs are very mobile.

Swelling and ecchymosis are most commonly observed after envenomation, though it may take several hours to appear.

Hemorrhagic blebs are common following bites to the finger but also may be seen on lower extremity bites.

Systemic toxicity can present in several ways. Nausea, vomiting, tachycardia, and a sensation of impending doom are nonspecific findings and may represent nothing more than anxiety from being bitten.

However, recurrent vomiting can suggest a significant envenomation, and tachycardia may result from significant third-spacing of fluids.

Hypotension may result from fluid shifts or be one manifestation of a severe envenomation.

Cardiovascular collapse and airway swelling can be due to anaphylaxis in patients who have been previously sensitized to snake venom antigens and in patients with severe envenomations who have never been previously exposed to venom.

Hematologic toxicity is frequently reported in crotalid envenomation.

Many patients have various laboratory test abnormalities (e.g., coagulopathy, thrombocytopenia, and hypofibrinogenemia) without any overt signs and symptoms.

However, in some patients the impaired hemostasis leads to clinically significant bleeding.

Although the hematotoxicity may be noted at presentation, it also can develop at any point in the first 2 weeks after envenomation.

Thrombotic events are also possible, but true disseminated intravascular coagulation is uncommon.

Neurotoxicity is not observed in most crotalid envenomations but is common following bites from some rattlesnakes, including certain populations of the Mojave rattlesnake (C. scutulatus) and the Southern Pacific rattlesnake (C. helleri).

Mild signs can include myokymia, ptosis, and diplopia, but more severe manifestations such as peripheral muscle paralysis and respiratory failure are also possible.

Uncommon sequelae of crotalid envenomation include myocardial injury, cerebrovascular accident, rhabdomyolysis, and fasciitis.

Compartment syndrome is a much-feared but rarely observed complication of pit viper bites. Venom is typically deposited above the fascia, so the overlying skin and subcutaneous tissue may be swollen and taut while compartment pressures remain normal.

However, venom can be injected into the fascia if the overlying tissue is particularly thin.

Signs of compartment syndrome include paresthesias, paralysis, diminished distal pulses, poikilothermia, pallor, and significant pain.

Among native pit vipers, rattlesnakes typically cause the most severe envenomations, whereas copperheads are often considered the least dangerous. In fact, some physicians have suggested that copperhead bites do not require aggressive treatment.

However, the potential for serious toxicity from copperheads should not be minimized. Approximately 2.3% of copperhead bites reported to U.S. poison centers from 2012–2015 resulted in death or a “major” effect, defined as signs and symptoms that were life threatening or resulted in significant residual disability or disfigurement.

A study of copperhead envenomations in the Carolinas found that tissue necrosis was present in 8% of bites, hematologic abnormalities were observed in approximately 30% of patients for whom laboratory testing was performed, and the median duration of disability in untreated patients was 42 days, with some patients reporting disability 1 year after the envenomation.

How well did you know this?
1
Not at all
2
3
4
5
Perfectly
32
Q

What is the best treatment for snakebites prior to hospital arrival?

A

The best treatment for snakebites prior to hospital arrival is to arrange for quick, safe transport to an appropriate medical facility.

If the snake can be safely and quickly photographed, that information may help guide therapy.

However, transport should not be delayed, because ultimately a pit viper envenomation can be clinically distinguished from a coral snake envenomation and from a bite from a nonvenomous snake.

The snake should not be transported with the patient because it exposes everyone to greater risk.

Even a recently killed snake can envenomate because brain activity and bite reflexes may persist for several hours.

Other recommended interventions include good supportive care.

Airway patency must be ensured. Fluids should be given judiciously, as pit viper bites can result in significant third-spacing.

It is important to maintain euvolemia without increasing the local tissue edema.

Analgesia should also be provided. Intravenous opioids are preferred. Nonsteroidal anti-inflammatory drugs (NSAIDs) are not recommended because of their potential negative hematologic effects.

Proper positioning of the affected extremity in the prehospital setting is controversial. Previously, experts recommended keeping the extremity below heart level to minimize the spread of the venom. Unfortunately, this can exacerbate local damage, which is almost universally present after crotalid envenomations.

Elevating the extremity above the heart can reduce the swelling but can accelerate systemic absorption of the venom. In areas where bites are unlikely to cause systemic toxicity, elevation is preferred.

If there is a high likelihood of systemic toxicity, it may be prudent to keep the affected extremity at heart level.

Once the patient arrives in the hospital, however, elevation is recommended for all pit viper envenomations.

How well did you know this?
1
Not at all
2
3
4
5
Perfectly
33
Q

How are snake bites managed in-hospital?

A

Once the patient has arrived at the hospital, definitive care promptly ensues. Life-threatening airway, breathing, and circulatory issues are addressed first.

Airway patency may be compromised in severe envenomations, and intubation should be performed if the patient fails to respond to pharmacological interventions such as epinephrine.

Intubation is also necessary for patients who have impaired ventilation secondary to respiratory muscle weakness.

Intravenous fluid resuscitation may be needed to restore euvolemia in patients with significant hypovolemia secondary to gastrointestinal losses and/or third-spacing.

Aggressive fluid administration should be avoided because it can exacerbate the tissue swelling that occurs in patients with increased capillary permeability.

Analgesia is an essential component to snakebite management, as significant pain often characterizes these bites. As mentioned, NSAIDs are discouraged because of the potential hematologic effects.

Intravenous opioids are preferred initially.

Morphine appears to cause greater histamine release, which could exacerbate the swelling and mimic an allergic reaction, so hydromorphone and fentanyl are preferred.

Although the prehospital positioning of the affected extremity has been debated, there is consensus that the limb should be elevated once the patient has arrived at the hospital. This prevents the venom from accumulating in the extremity and reduces the hydrostatic pressures that can exacerbate tissue swelling. Elevation presumably increases the potential for systemic absorption, but this allows for faster recognition of toxicity.

Furthermore, if antivenom is administered, extremity elevation allows for quicker venom neutralization.

A full assessment of the snakebite patient includes laboratory studies, such as prothrombin time (PT), fibrinogen, serum electrolytes, blood urea nitrogen, creatinine, and complete blood count (CBC).

The partial thromboplastin time (PTT) and d-dimer do not provide any additional information and are not typically needed.

Rotational thromboelastometry and thromboelastography are not readily available in many health care settings but may have a greater role in evaluating patients for hematotoxicity in the future.

The routine measurement of serum creatine phosphokinase (CPK) is not recommended. However, in patients with known or suspected rhabdomyolysis, serial CPK levels should be obtained.

Radiographic studies are not necessary in most patients but may be helpful if there is suspicion for a retained foreign body or to evaluate end-organ damage, including airway injury.

The affected extremity should be monitored for progression of the swelling and tenderness.

Evaluation of neuromuscular strength is recommended for envenomations from suspected neurotoxic species.

The negative inspiratory force is a sensitive test for respiratory muscle strength and can be performed at the bedside by a respiratory therapist.

Dynamometry is an excellent test of peripheral muscle strength but may not always be available.

The specific treatment for snake envenomations is antivenom, and each patient should be assessed individually to determine whether antivenom is indicated. As of early 2018 there was only one available antivenom approved by the U.S. Food and Drug Administration (FDA) for U.S. pit vipers.

Crotalidae polyvalent immune fab ovine (CroFab) (FabAV) was first studied in 1993 and has been commercially available since 2000. It is made by immunizing different flocks of sheep with the venom of one of four crotalid species: western diamondback rattlesnake (C. atrox), eastern diamondback rattlesnake (C. adamanteus), Mojave rattlesnake (C. scutulatus), and cottonmouth (Agkistrodon piscivorus).

The antibodies collected from the sheep are then treated with papain to liberate the individual Fab fragments of the immunoglobulin molecule.

It has been shown to terminate both the local and systemic venom effects, resulting in faster and more complete recovery from envenomation when compared with placebo.

Indications for antivenom use include progression of local tissue findings and/or evidence of systemic toxicity (e.g., hematotoxicity, airway swelling, or cardiovascular collapse).

The precise definition of hematologic toxicity is unclear, but many providers use PT longer than 15 seconds, platelet count less than 150 × 103 /μL, fibrinogen less than 220 mg/dL, or a significant change from baseline.

An algorithm for CroFab administration is available in the 2004 American College of Surgeons Committee on Trauma publication.

It should be emphasized that the Snakebite Severity Score (SSS) should not be used to determine the need for treatment. It was designed as a research tool, not to guide therapy. Reliance on this scale can result in significant undertreatment.

If the swelling and tenderness are more than minimal and have extended beyond a major joint, antivenom is warranted.

If there is significant local tissue injury, such as necrosis, antivenom is also indicated, even if the swelling has not progressed across a joint.

A recent randomized clinical trial studying the effects of CroFab on copperhead bites demonstrated that even mild bites recovered better when treated with antivenom. Specifically, patients had improved limb function at 7, 10, and 14 days after envenomation compared with the placebo group, and 75% of treated patients had full recovery of limb use by day 31, whereas the control group did reach this milestone until 57 days. Additionally, patients who were treated with antivenom required less opioids. In this study, no one treated with antivenom needed opioids after 21 days, whereas some patients who did not receive antivenom required opioids for up to 90 days.

As the incidence of opioid abuse continues to rise, this is an important reason to consider early and aggressive treatment with antivenom.

Another antivenom, Crotalidae immune F(ab′)2 equine (Anavip), has been approved for the treatment of North American rattlesnake bites and is now available. In a comparison of the two antivenoms, F(ab′)2 use was associated with less late coagulopathy than the FabAV product.

Once they are both available, the decision to use one over the other will likely be based on several variables, including the likelihood of hematotoxicity following the bite, cost and availability of the respective products, and prior sensitization of the patient to equine- or ovine-derived products.

Some physicians are reluctant to treat with antivenom because of safety concerns, but the likelihood of an adverse reaction is relatively low. Unlike the antivenin Crotalidae polyvalent product that was used for decades before CroFab was introduced, CroFab has minimal amounts of the Fc portion, making it much less immunogenic than the older product that has since been discontinued.

Manifestations of adverse reactions include urticaria, bronchospasm, anaphylaxis, and serum sickness.

In a meta-analysis of 11 studies, the authors found an 8% incidence of acute adverse reactions and 13% incidence of delayed effects (e.g., serum sickness), following CroFab use.

Recent data from the North American Snakebite Registry (NASBR) indicate that only 2.3% of adults and 2.7% of children who received CroFab have acute adverse reactions, including rash (0.9%), hypotension (0.9%), and bronchospasm (0.9%).

There is also an unfounded fear of administering antivenom to patients who have previously received it. Most people who have been treated with CroFab more than once have not had an adverse reaction to the second and subsequent exposures. In one exceptional case, a gentleman who received FabAV at least 19 times had two episodes of mild acute hypersensitivity, but tolerated the antivenom the other 17 times without incident.

Antibiotics should not be routinely administered following crotalid envenomation as there is no benefit of antibody prophylaxis. In fact, infection following a snakebite is exceedingly rare. Of 276 snakebites documented in the NASBR from March 2013 to October 2014, there was only one documented infection, and this occurred in the setting of extensive necrosis and digital nerve injury.

Antihistamines and corticosteroids also confer no benefit following most pit viper bites, though they are indicated in the rare setting of an allergic reaction or anaphylaxis to the bite or to the antivenom.

How well did you know this?
1
Not at all
2
3
4
5
Perfectly
34
Q

When is surgery indicated for snakebites?

A

Surgeons are frequently primarily responsible for snakebite management in areas where envenomations are common.

For many years, an aggressive approach to crotalid envenomations was advocated, but it is becoming increasingly apparent that surgical intervention is rarely necessary.

Prophylactic fasciotomies are no longer recommended.

Animal studies prove that morbidity and mortality are increased following prophylactic fasciotomy when compared with antivenom.

In a review of 99 publications evaluating the efficacy of fasciotomy in animals and humans, the authors could not identify any situation in which surgical intervention was beneficial.

An expert panel consisting of trauma surgeons and medical toxicologists also concluded that prophylactic fasciotomy was not beneficial and was possibly harmful. The same authors concluded that, even in the exceptionally rare case of confirmed compartment syndrome, the initial treatment should be additional doses of antivenom, not fasciotomy.

The rationale, supported by animal studies, is that elevated compartment pressure represents a severe envenomation but is not the cause of the morbidity.

Rather, it is the venom that is causing the damage, and neutralizing the venom is the definitive treatment.

Surgical intervention following envenomation is indicated when there is full-thickness necrosis requiring amputation.

However, premature operative intervention should be avoided because many patients with superficial necrosis will recover fully if managed with conservative wound care.

Some experts advocate the use of a colloidal silver bandage with or without debridement, and others recommend twice-daily application of a petrolatum-containing ointment.

How well did you know this?
1
Not at all
2
3
4
5
Perfectly
35
Q

What follow up care is needed for snake bites?

A

Patients with upper extremity bites should be monitored for a minimum of 8 hours if there is any evidence of an envenomation.

Lower extremity envenomation warrants observation for at least 12 hours, and some providers recommend overnight hospital admission.

Admission is also recommended for patients requiring antivenom and those with evidence of systemic toxicity.

Patients should follow up with their primary care provider or a snakebite expert within 3–5 days after hospital discharge.

Routine laboratory testing is recommended for patients who were found to have hematotoxicity and those with rattlesnake envenomation because of the potential for hematologic complications.

Patients who are discharged after a pit viper envenomation should also be instructed to elevate the extremity when possible and avoid weight-bearing on the affected extremity for 1 week or longer if signs and symptoms persist.

Pain should be controlled with acetaminophen or, if necessary, opioids.

Again, NSAIDs should be avoided because of the potential hematologic effects.

How well did you know this?
1
Not at all
2
3
4
5
Perfectly
36
Q

How do coral snake bites differ from pit viper bites?

A

Coral snakes have a much more limited distribution than pit vipers and account for approximately 5% of bites from venomous snakes in the United States.

Approximately 50% of coral snake bites result in envenomation because the smaller, fixed teeth make coral snakes less efficient than pit vipers in delivering their venom.

Three species of coral snakes are found in the United States. The Sonoran coral snake, Micruroides euryxanthus, is found in Arizona and western New Mexico.

The Texas coral snake, Micrurus tener, is located in Texas, Louisiana, and Arkansas.

The Eastern coral snake, Micrurus fulvius, is confined to the Southeastern United States, as far north as North Carolina and as far west as Mississippi.

Sonoran coral snake envenomations cause superficial muscle twitching but little else, and bites do not require any specific management.

Envenomations from the Texas coral snake are associated with significant pain and, on occasion, objective neurologic toxicity, ranging from ptosis to respiratory paralysis.

Local tissue effects are confined to mild swelling and erythema. A case series of Texas coral snake bites that were reported to poison control cited a 7.3% incidence of systemic toxicity and no cases of severe envenomation.

The Eastern coral snake is responsible for the most significant toxicity from native coral snake envenomations.

Death is exceptionally rare, but progressive neurologic signs and symptoms, including respiratory paralysis, may be observed in patients who are not treated with antivenom.

Coral snake antivenom, which is different from the CroFab used to treat crotalid envenomations, should be given empirically following envenomation from Eastern coral snakes because of the high likelihood of progressive neurotoxicity.

If antivenom is unavailable or withheld, intubation and mechanical ventilation may be needed.

Antivenom is almost never needed for bites from Texas coral snakes, but pain control is important.

For both Texas and Eastern coral snake bites, patients should be observed for a minimum of 12 hours because the onset of toxicity may be delayed.

How well did you know this?
1
Not at all
2
3
4
5
Perfectly
37
Q

Which exotic snakes can cause envenomations?

A

A wide variety of exotic snakes, many of which are venomous, are maintained in zoos and private collections.

Species from which envenomations have been reported in recent years include, but are not limited to, the Gaboon viper (Bitis gabonica), monocled cobra (Naja kaouthia), snouted cobra (Naja annulifera), forest cobra (Naja melanoleuca), king cobra (Ophiophagus hannah), African bush viper (Atheris squamigera), and the white-lipped tree viper (Cryptelytrops albolabris).

A discussion of the pathophysiology, clinical features, and management of these envenomations is beyond the scope of this book.

Physicians caring for patients with these exotic envenomations are encouraged to contact their regional poison control center or snakebite expert for specific recommendations.

How well did you know this?
1
Not at all
2
3
4
5
Perfectly
38
Q

A 17-year-old male presents to the emergency department 12 hours after having his right forearm run over by a car.

His blood pressure is 135/90 mmH, and his heart rate is 110 beats per minute. Radiographs of the right forearm show no fractures. His forearm is very swollen but soft and compressible. A pressure monitor is used, which shows intra-compartmental pressure of 35 mmH. The patient does not have pain with passive extension of his fingers or wrist.
Which of the following is the next best step in the management of this patient?

Choices:
1. Immediate fasciotomy
2. Ice application and limb elevation with serial clinical compartment checks in the hospital
3. Soft compression bandage application, discharge home with instructions to apply ice and limb elevation
4. Repeat pressure measurement in 1 hour

A

Answer: 2 - Ice application and limb elevation with serial clinical compartment checks in the hospital

Explanations:
•This patient’s clinical exam is not concerning for compartment syndrome. His absolute pressure is greater than 30 mmHg, but delta pressure is greater than 30 mmHg.
• The delta pressure is the difference between diastolic pressure and compartment pressure. A delta pressure less than 30 mmHg is more indicative of compartment syndrome than an absolute pressure greater than 30 mmHg.
• The patient’s exam is not consistent with compartment syndrome.
However, it can evolve over several hours so the patient should be admitted and monitored with serial clinical exams. Ice and elevation of the forearm will provide symptomatic relief.
•An invasive pressure needle to measure compartment pressures should only be undertaken when clinical exams are not reliable.

StatPearls

How well did you know this?
1
Not at all
2
3
4
5
Perfectly
39
Q

An 8-year old child is brought to the emergency department after being struck by a car while crossing the street. He is not alert. He required immediate intubation at the scene by emergency management services. His Glasgow coma scale is 8. He appears to have a significant laceration of his scalp on the left side, but there are no skeletal fractures. The initial chest x-ray reveals a right-side pneumothorax.
A chest tube was inserted. His hematocrit is 30%, and his hemoglobin is 10.6 g/dL. Heart rate is 100 bpm, blood pressure 100/60 mmHg, and respirations 20/minute. What is the next step in management?

Choices:
1. CT of the head, thorax, and abdomen
2. Repeat chest x-ray
3. Repeat blood work
4. MRI of the head and neck

A

Answer: 1 - CT of the head, thorax, and abdomen

Explanations:
• The patient has two signs of Waddell triad and has a low hematocrit. The abdomen, especially the retroperitoneum, can be a source of significant bleeding.

• The Waddel triad includes trauma to the head, abdomen, thorax, and lower extremities with a car versus pedestrian impact. The full workup includes a whole-body CT scan, which can rapidly identify injuries to the brain (intracranial hemorrhage), chest, abdomen, and presence of any fractures. This is usually done as part of the tertiary trauma survey, which follows the secondary survey (a focused history and physical exam).

• If the patient has low hematocrit and there is no fracture, and the chest is clear, think about possible bleeding in the abdomen.

•The size of the child and the type of car can help determine injury patterns. Injury patterns will vary depending on the country and what side of the road a driver is on when a pedestrian is impacted by the vehicle. Children ages 1 to 5 years may be knocked down and dragged under the vehicle. The front bumper causes injuries to the femur, chest, abdomen, and pelvis. Children ages 3 to 11 years are prone to bumper impacts that fracture femurs and chest injuries from fenders or the hood. A child may be thrown onto the hood and hit the windshield, causing head and facial trauma. Another insult to the head occurs when the car stops abruptly, and they are thrown off the hood and onto the road striking their head again. Suspect multiple injuries with a car versus pedestrian impact. When taking a history, determine speed, the point of impact, and any safety gear worn at the time.

How well did you know this?
1
Not at all
2
3
4
5
Perfectly
40
Q

A 7-year old girl is brought to the hospital after suffering multiple injuries after being run over by a car. She has since been intubated, and her vital signs include a temperature of 37 C, pulse of 130 beats/min, respiratory rate of 20 breaths/min, and blood pressure of 90/50 mmHg. The girl has bruising involving her posterior trunk, buttocks, and abdomen. On auscultation, her heart has a regular rate and rhythm with no murmurs, and her lungs are clear bilaterally.
Her abdomen is rigid and slightly distended. Her extremities are cool and mottled, with a capillary refill time of 3 seconds. There are no other external signs of trauma. What is the most appropriate fluid management for this child’s first 24 hours of treatment?

Choices:
1. Lactated Ringer solution
2. Five percent dextrose with 0.2% sodium chloride (NaCI)
3. Five percent dextrose with 0.45% NaC1
4. Ten percent dextrose with 0.45% NaCI

A

Answer: 1 - Lactated Ringer solution

Explanations:
•Fluid management in the first 24 hours after a blunt trauma with risk of internal bleeding should include an intravenous (IV) crystalloid solution.
• Failure to adequately restore or maintain intravascular volume can adversely affect hemodynamics and lead to lactic acidosis and multiple organ failure.
•To adequately maintain intravascular volume, the choice of fluid should be isotonic, such as normal saline (0.9%) or lactated Ringer’s solution.
• If the child is in acute shock, additional boluses of isotonic fluid may be required.

StatPearls

How well did you know this?
1
Not at all
2
3
4
5
Perfectly
41
Q

A 17-year-old male patient is stabbed in the right chest at the eighth intercostal space in the midclavicular line. He was involved in a fight with his neighbor. In the emergency department, he has a blood pressure of 100/60 mmH, a respiratory rate of 22 breaths per minute, and a pulse rate of 120 beats per minute. On room air, his oxygen saturation is 94%. Auscultation reveals bilaterally equal air entry. What is the next best step in management?

Choices:
1. Resuscitate with intravenous fluids, oxygen, and abdominal computed tomography (CT) with IV contrast
2. Explore the stab wound in the emergency department
3. Diagnostic peritoneal lavage and delayed chest CT scan
4. Diagnostic laparoscopy

A

Answer: 1 - Resuscitate with intravenous fluids, oxygen, and abdominal computed tomography (CT) with IV contrast

Explanations:
•In any patient with an injury below the seventh rib, clinicians should always be cognizant of an abdominal injury. The initial step should be resuscitation. The patient is potentially unstable.
• This patient likely has a liver injury. These injuries can often be managed by interventional radiology (IR) or observation, depending on patient stability.
• Diagnostic laparoscopy is worthwhile if a CT scan or IR is not available or if the patient is unstable. It can also be converted to an open procedure if there is suspicion of significant injury.
•The abdominal contents cannot always be fully explored with a laparoscope, but the presence of blood, bile, or tissue trauma should raise suspicion for organ injury.

How well did you know this?
1
Not at all
2
3
4
5
Perfectly
42
Q

A 17-year-old male is brought to the emergency department by emergency medical services after falling from his bicycle during a mountain bike ride. The patient was helmeted. C-collar placed by EMS on arrival. He complains of back, epigastric pain and nausea.
Vitals signs are blood pressure 118/78 mmH, heart rate 101/minute, RR 18, 02 sat of 99%, Temp of 37 C. CT head and cervical spine are negative. CT of the chest, abdomen, and pelvis shows L1 chance fracture but no other injuries. Patient given pain medication and anti-emetics, however, symptoms persist. Which test is most likely going to provide the diagnosis?

Choices:
1. Deep peritoneal lavage (DPL)
2. Magnetic resonance cholangiopancreatography (MRCP)
3. Focused assessment with sonography test (FAST)
4. Liver function tests

A

Answer: 2 - Magnetic resonance cholangiopancreatography
(MRCP)
Explanations:
• Blunt abdominal trauma can lead to pancreatic trauma. A sudden localized impact to the abdomen which results in compression of the intra-abdominal organs against the vertebral column and can lead to pancreatic injuries.
• Examples of mechanisms associated with pancreatic injury include bicycle handlebar injuries in children, steering wheel injury in adult motor vehicle collision, or direct kick in an assault.
•Computerized tomography (CT) is the test of choice for hemo-dynamically stable trauma patient with blunt abdominal injury however it can have difficulty in identifying pancreatic injury.
Twenty to forty percent of pancreatic injuries appear normal within 12 hours of trauma. The accuracy of CT identifying traumatic major duct injury is reported as low as 43%. FAST, DPL, Liver function tests are not diagnostic for traumatic pancreatitis.
• MRCP has become pivotal in the evaluation of traumatic pancreatic injury. Dynamic secretin-stimulated MRCP is rapid, noninvasive, and competes with endoscopic resonance cholangiopancreatog-raphy (ERCP for accuracy. MRCP has the added advantage of being non-invasive.

StatPearls

How well did you know this?
1
Not at all
2
3
4
5
Perfectly
43
Q

A 17-year-old male patient was brought to the emergency department following a stab wound with a knife to his right chest. His initial vitals are blood pressure 95/60 mmH, pulse rate 110 beats per minute, and respiratory rate 24 breaths per minute. Respiratory sounds were diminished at the basilar half of the right hemithorax.
Following the primary chest X-ray, a right chest tube was placed, and 1700 ml of bloody drainage was evacuated. Afterward, his vitals are blood pressure of 68/50 mmg and pulse rate of 130 beats per minute.
What is the preferred next step in his management?

Choices:
1. Collect the evacuated blood and replace it with crystalloid solutions
2. Transfer the patient for thoracotomy and add anticoagulant to the evacuated blood
3. Transfer the patient for angioembolization and start the massive transfusion protocol
4. Start the massive transfusion protocol and chart the hourly bloody drainage

A

Answer: 2 - Transfer the patient for thoracotomy and add anticoagulant to the evacuated blood

Explanations:
• Although not ideal, too much anticoagulant solution is not an absolute contraindication for its use with autotransfusion blood. The addition of blood can dilute the anticoagulant solution preventing the waste of the collected blood. After the anticoagulant solution has been administered to the collected blood, it can be kept for up to 6 hours if needed.
• Triggers for massive transfusion are as follows: Systolic blood pressure (SP) 70 with penetrating torso injury, major pelvic injury, positive, focused abdominal sonography for trauma, and; BP 71-90 mmHg and heart rate > 108 with penetrating torso injury, major pelvic injury, positive, focused abdominal sonography for trauma.
• In the critically injured patient requiring large amounts of blood component therapy, a massive transfusion protocol should be followed. This approach calls for administering various components in a specific ratio during the transfusion to restore blood volume to reverse shock and correct coagulopathy. Although the optimal ratio is unknown, current evidence suggests a presumptive 1:2 red cell to plasma ratio in patients at risk for massive transfusion (10 units of packed red blood cells in 6 hours.
•Any patient with a known bleeding disorder should not be given blood with high amounts of the anticoagulant solution in collected blood. Autotransfusion is also inappropriate since the patient may benefit from specific targeted therapy.

StatPearls

How well did you know this?
1
Not at all
2
3
4
5
Perfectly
44
Q

A 7-year old girl is brought to the hospital after suffering multiple injuries after being run over by a car. She has since been intubated, and her vital signs include a temperature of 37 C, pulse of 130 beats/min, respiratory rate of 20 breaths/min, and blood pressure of 90/50 mmHg. The girl has bruising involving her posterior trunk, buttocks, and abdomen. On auscultation, her heart has a regular rate and rhythm with no murmurs, and her lungs are clear bilaterally.
Her abdomen is rigid and slightly distended. Her extremities are cool and mottled, with a capillary refill time of 3 seconds. There are no other external signs of trauma. What is the most appropriate fluid management for this child’s first 24 hours of treatment?
Choices:
1. Lactated Ringer solution
2. Five percent dextrose with 0.2% sodium chloride (NaC1)
3. Five percent dextrose with 0.45% NaC1
4. Ten percent dextrose with 0.45% NaCI

A

Answer: 1 - Lactated Ringer solution
Explanations:
• Fluid management in the first 24 hours after a blunt trauma with risk of internal bleeding should include an intravenous (IV) crystal-loid solution.
• Failure to adequately restore or maintain intravascular volume can adverselv affect hemodvnamics and lead to lactic acidosis and multiple organ failure.
• To adequately maintain intravascular volume, the choice of fluid should be isotonic, such as normal saline (0.9% or lactated Ringer’s solution.
• If the child is in acute shock, additional boluses of isotonic fluid may be required.

StatPearls

How well did you know this?
1
Not at all
2
3
4
5
Perfectly
45
Q

A 17-year-old male patient is brought to the emergency department after a motor vehicle collision. Emergency medical services found him with the steering wheel against his body. Upon arrival, the patient has a blood pressure of 134/98 mmH, a heart rate of 107 beats per minute, a respiratory rate of 18 breaths per minute, oxygen saturation of 98% on room air, and a temperature of 37 C
(98.6 F). The patient has abdominal pain. A focused assessment with sonography in trauma (FAST) is negative. A computed tomography (CT) abdomen and pelvis shows a major pancreatic contusion without clear evidence of ductal injury. What is the next step in patient management?

Choices:
1. Endoscopic retrograde cholangiopancreatography (ERCP) or magnetic resonance cholangiopancreatography (MRCP)
2. Abdominal CT angiography within the next 6 hours
3. Exploratory laparotomy and intra-operative cholangiography
4. Upper GI-endoscopy

A

Answer: 1 - Endoscopic retrograde cholangiopancreatography
(ERCP) or magnetic resonance cholangiopancreatography (MRCP)

Explanations:
• Computerized tomography (CT) is the test of choice for hemodynamically stable trauma patients with blunt abdominal injuries. However, it can have difficulty identifying a pancreatic injury. Twenty to forty percent of pancreatic injuries appear normal within 12 hours of trauma.
• Diagnostic testing may require magnetic resonance cholangiopancreatography (MRCP) or endoscopic retrograde cholangiopancreatography (ERCP).
• The American Association for the Surgery of Trauma (AAST) proposed the pancreatic Organ Injury Scale (OIS), universally accepted in grading pancreatic injury. The AAST pancreatic OIS utilizes CT findings and is applied to guide therapy in hemodynamically stable patients who suffer blunt pancreatic injury.
• The 2016 Eastern Association of the Surgery of Trauma (EAST) utilized The American Association for the Surgery of Trauma (AAST) proposed the pancreatic Organ Injury Scale (OIS) and released several recommendations for the management of traumatic pancreatic injury.

Low-grade injury is Grade I and II, which undergo preoperative or intraoperative ERCP/MRCP.

If there is no pancreatic ductal injury, then conservative management should be performed, which includes nasogastric tube placement, bowel rest, and nutritional support.

High-grade injury is defined as Grade III to V; with these injuries, they recommend conditional surgical intervention.

StatPearls

How well did you know this?
1
Not at all
2
3
4
5
Perfectly
46
Q

A 17-year-old female patient who was in a motorcycle collision comes to the emergency department. She is groaning and speaking in a low voice. She has multiple abrasions, a blood pressure of 80/50 mmH, and a respiratory rate of 22 breaths per minute. She is cold to the touch, and there is extensive subcutaneous emphysema in the right upper chest and neck. A focused assessment with sonography in trauma (FAST) exam is performed. What is the next step in management?
Choices:
1. Chest x-ray
2. Urgent chest computed tomography scan
3. Tube thoracostomy
4. Fluid resuscitation

A

Tube thoracostomy
Explanations:
• Airways should be the top priority.
• Subcutaneous emphysema with respiratory distress is a pneumothorax until proven otherwise.
• If available, FAST imaging should be done. If not, the tube is placed at the fifth intercostal and the midaxillary line.
• In the field, needle decompression can be done and may be appropriate in the hospital. In the emergency department, tube thoracostomy is preferred. An arrow catheter can be used and later converted.

StatPearls

How well did you know this?
1
Not at all
2
3
4
5
Perfectly
47
Q

A 3-year-old male weighing 14 kg (31 lbs) presents with his mother after pulling a pot of hot water onto himself from the stove. He has third-degree burns covering 60% of his body surface area.
He is admitted to the pediatric burn unit, and fluid resuscitation is initiated per the Cincinnati formula. What is the goal urine output?
Choices:
1. 1 ml/kg/hr
2. 0.5 ml/kg/hr
3.2 ml/kg/hr in the first 8 hours followed by 0.5 ml/kg/hr
4. 1 ml/kg/hr in the first 16 hours followed by 2 ml/kg/hr

A

Answer: 1 - 1 ml/kg/hr
Explanations:
• Goal urine output for children less than 30 kg is 1 ml/kg/hour, and for children more significant than 30 kg, it is 0.5 mg/kg/hour, and fluid resuscitation should be adjusted to meet but not exceed these values.
• The generally accepted cutoff for using the adult formula is 30 kg to 50 kg.
• Pediatric burn formulas are always two-figure calculations and will require initial resuscitation in addition to maintenance fluids.
• Infants will also require dextrose in addition to fluid resuscitation and maintenance fluids.

StatPearls

How well did you know this?
1
Not at all
2
3
4
5
Perfectly
48
Q

A 17-year-old male restrained driver was involved in a motor vehicle collision. In the trauma bay, his vital signs are blood pressure 120/70 mmHg, heart rate 110 beats per minute, oxygen saturation 100% on room air, and temperature 98.7 F (37 C). He is complaining of abdominal pain. On abdominal exam, no external lesions were noted. His abdomen was soft, non-distended, and tender at the left upper quadrant with no rebound or guarding. The rest of his exam was unremarkable. Urinalysis shows microscopic hematuria. A focused ultrasound was performed which was positive in the left upper quadrant view. What is the best test to further evaluate the patient?
Choices:
1. Non-contrast computed tomography (CT) scan of abdomen and pelvis
2. Intravenous urography
3. Contrast-enhanced CT scan of abdomen and pelvis with immediate and delayed imaging
4. Peritoneal lavage

A

Answer: 3 - Contrast-enhanced CT scan of abdomen and pelvis with immediate and delayed imaging
Explanations:
• If there is hemodynamic instability that warrants immediate surgical intervention, an intraoperative intravenous urography (IVU) should be performed. Doing so allows assessment of the contralatreal renal function, which can further guide the surgical approach to the affected kidnev.
• Frequently the first imaging a trauma patient receives is in the trauma bay with a focused assessment with sonography for trauma
(FAST) exam. This imaging exam is conducted to assess for intraabdominal or pericardial fluid, which if present, will be highly indicative of bleeding in the trauma patient and so assist in timely diagnosis and aid in the decision if operative intervention is warranted.
• The modality of choice in the suspected renal trauma patient is contrast-enhanced computed tomography with immediate and delayed imaging.
• A computed tomography scan delineates injury findings and severity that would be seen if surgery was performed and the contrast allows evaluation of the renal vessels, where active extravasation of blood would be supported by a blush on imaging.

StatPearls

How well did you know this?
1
Not at all
2
3
4
5
Perfectly
49
Q

A 17-year-old male patient has a stab wound medial to and at the level of the nipple. A left chest tube is placed in the emergency department and drains 400 mL of blood. The patient continues to be tachycardic and hypotensive despite a chest x-ray that shows that the chest tube is in a good position and the lung is re-expanded. What is the most appropriate next step?
Choices:
1. Transthoracic echocardiography
2. Emergency department midsternal thoracotomy
3. Emergent pericardiocentesis
4. Second left-sided chest tube

A

Answer: 3 - Emergent pericardiocentesis
Explanations:
• If this patient has a cardiac injury, pericardial blood could be decompressing into the left chest and be drained by the chest tube.
Therefore, an emergent pericardiocentesis, followed by a pericardial window, according to the results, is necessary to determine if there is a cardiac injury. An echocardiogram may be appropriate in a stable patient.
• The patient does not meet the criteria for emergency department thoracotomy. Moreover, the standard emergency department thor-acotomy is described as a left anterolateral thoracotomy to provide an appropriate life-saving transient aortic control.
• Pericardial window in the operating room is the most appropriate step in an unstable patient with a stab wound to the cardiac box.
• The chest x-ray shows that the lung is expanded and the first chest tube is in a good position. There is no indication for a second chest tube.

StatPearls

How well did you know this?
1
Not at all
2
3
4
5
Perfectly
50
Q

A 17-year-old female sustains a circumferential burn of the left forearm in a campfire accident, with additional burned areas along the left side of her body. There was no significant impact.
While assessing her total body surface area affected and evaluating her need for formal fluid resuscitation, she states that her left hand is developing “excruciating, deep, achy” pain. The initial inspection is normal.
The radial or ulnar pulses on the left wrist are not palpable. Upon touching the patient’s hand, she screams in agony, and the posterior forearm is taut to palpation. What is the preferred next step management?
Choices:
1. Measure the intracompartmental pressure, and if the pressure is less than 30 mmH, proceed with an emergent fasciotomy
2. Escharotomy is indicated before fluid resuscitation begins
3. Warm and cool intermittent compressing should be initiated after fluids are started
4. The forearm needs to be splinted in the wrist 70 degrees dorsiflexion after fluids are started, and reassessed in 3 hours

A

Answer: 2 - Escharotomy is indicated before fluid resuscitation begins
Explanations:
• Compartment syndrome is a limb-threatening complication of circumferential burns. If intracompartmental pressure (ICP) is greater than or equal to 30 mmHg or delta pressure is less than or equal to 30 mmHg, fasciotomy should be done.
• In situations of compartment syndrome, all restrictive dressings such as splints or casts should be removed immediately.
• Although occult fracture can cause compartment syndrome, the patient’s injury was not traumatic, and inflammation alone will not cause fracture.
• Escharotomy and fasciotomy should be performed before starting fluid resuscitation because the resulting edema will worsen the compartment syndrome.

StatPearls

How well did you know this?
1
Not at all
2
3
4
5
Perfectly
51
Q

A 12-year-old previously healthy boy is brought to the emergency department (ED) after he was struck on his right side by a motor vehicle while he was running across the street. His upper body was thrown forward, and he bumped his left forehead on the pavement.
He was alert and oriented when the paramedics arrived at the scene.
They immobilized his entire spine using a pediatric backboard and cervical spine collar before transport. In the ED, he is anxious but fully oriented. He reports pain in his head, abdomen, and legs. His vital signs include a temperature of 37.6 C, heart rate of 89 beats/min, respiratory rate of 19 breaths/min, blood pressure of 100/70 mmH, and pulse oximetry of 99% (room air). On physical examination, the boy’s airway is clear, he is breathing spontaneously with normal respiratory effort, and his pulses and perfusion are normal. A superficial abrasion over his left forehead is noted. His abdomen is soft and non-distended, but it is tender to palpation in the right upper quadrant. He displays no peritoneal signs. There is tenderness to palpation over his right thigh with swelling and bruising. Imaging is performed which shows a hepatic contusion and a non-displaced skull fracture on the left side. What additional finding in this patient would confirm the diagnosis of Waddell triad?

Choices:
1. Left tibia fracture
2. Right femur fracture
3. Right humerus fracture
4. Spinal injury

A

Answer: 2 - Right femur fracture

Explanations:
•Waddell triad consists of three distinct features seen in pediatric pedestrian patients with blunt force trauma, usually secondary to direct impact by a motor vehicle. They include ipsilateral femur fracture, ipsilateral intrathoracic or intraabdominal injury, and contralateral head injury.
• In this case, the patient was struck on his right side causing right side intraabdominal injury of the liver, left side head injury and would also have a right-sided femur fracture as the third sign of the triad.
• Whenever children pedestrians are involved in a motor vehicle accident, it is important not to assume that they have only one organ injury. Further investigation should be done to identify other affected organs.
• Patients who have Waddell triad should be treated as significantly injured and high risk for bleeding and shock.

StatPearls

How well did you know this?
1
Not at all
2
3
4
5
Perfectly
52
Q

A 16-year-old male was in a motorcycle accident and sustained injuries to the left side of his body. Imaging studies showed the spleen was ruptured, requiring emergent splenectomy. Can this individual survive and live normally after a splenectomy?
Choices:
1. Yes, provided precautionary vaccinations and immunizations are taken
2. No, the spleen is a vital organ
3. No, a transplant would be necessary for survival
4. Yes, there will no change in homeostasis

A

Answer: 1 - Yes, provided precautionary vaccinations and immunizations are taken
Explanations:
• The spleen is not a vital organ.
• Bone marrow, the liver, and lymph nodes provide bodily functions analogous to splenic functions.
•Splenectomy is common after of severe splenic trauma and rupture.
Splenectomy is done to avert rupture in massive splenomegaly.
•Vaccinations are given a patient at least 2 weeks before elective splenectomy. They should be given at least 2 weeks after emergent splenectomy. These include pneumococcal, Haemophilus in-fluenzae B, meningococcal, and influenza vaccines.

StatPearls

How well did you know this?
1
Not at all
2
3
4
5
Perfectly
53
Q

A 17-year-old male patient was admitted one month ago due to a motor vehicle collision. There is decreased strength in the lower extremities with normal strength in the upper extremities. He is now starting to complain of a severe headache. He also has developed intermittent hypertensive episodes where his blood pressure would reach 190/100 mmHg when he has a full bladder. At what level of spinal injury is this presentation seen?
Choices:
1. C2-C6 only
2. T3 and above
3. T6 and above
4. L1 and above

A

Answer: 3-T6 and above
Explanations:
•While patients with cervical spine injuries are at higher risk for autonomic dysreflexia, the actual risk is for any spinal lesion at or above the T6 level.
• A headache and increased blood pressure typically characterize autonomic dysreflexia.
• Treatment involves identifying and eliminating the precipitating factor, which is often a blocked foley catheter or a distended bladder.
• It rarely develops in spinal lesions at or below and T10.

StatPearls

How well did you know this?
1
Not at all
2
3
4
5
Perfectly
54
Q

Hypovolaemic shock in the paediatric population, as compared with adults, includes all of the following except:

A Children have a tendency towards delayed hypotension, followed quickly by complete cardiovascular collapse.

B Blood products should only be administered after two boluses of
5–10 mL/kg of crystalloid solution.

C Children are at increased risk of heat loss, due to an increased body surface area.

D The first-choice fluid for volume resuscitation in infants should be a crystalloid solution.

E Resuscitation may necessitate the placement of an intraosseous intravenous in children <6 years of age.

A

B

Several differences exist in the manifestation of hypovolaemic shock in the paediatric population when compared with adults.

Children have an increased physiologic reserve and show signs of hypovolaemic shock much later, with hypotension followed quickly by complete cardiovascular collapse. less obvious signs of hypovolaemia such as decreased pulse pressure (<20 mmHg), mottled skin, cool extremities, capillary refill and lethargy can be the clues to a shock state in the eyes of the experienced clinician.

Thermoregulation is an important consideration in the treatment of paediatric patients, as their increased body surface area places them at increased risk of heat loss and hypothermia.

The total estimated blood volume of a child is approximately 80 ml/kg, indicating that a 1-year-old child weighing 10 kg who has lost 200 ml of blood will actually have a 25% blood volume deficit.

Intravenous access may prove challenging during a paediatric resuscitation; an intraosseus intravenous is a simple technique applicable to children less than 6 years of age.

As in the adult, the fluid of choice for resuscitation of hypovolaemia remains crystalloid, but Advanced Trauma life Support (ATlS) guidelines suggest two boluses of 10–20 mL/kg before the administration of blood products.

SPSE 1

How well did you know this?
1
Not at all
2
3
4
5
Perfectly
55
Q

The most common aetiology of shock in the paediatric population is:

A cardiogenic
B hypovolaemic
C septic
D distributive
E obstructive.

A

B hypovolaemic

Hypovolaemic shock is by far the most common aetiology of shock in the paediatric population.

Causes include hypovolaemia from GI losses (diarrhoea), renal losses (diabetes) or haemorrhage. other less common mechanisms of shock include the following:

● Cardiogenic: a primary pump failure. Causes include myocarditis or a congenital heart defect.

● Septic: a physiologic response (systemic inflammatory response syndrome) in the presence of suspected or confirmed infection. most commonly due to bacteraemia.

● Distributive: consists of pathological and inappropriate vasodilatation, endothelial dysfunction with capillary leak, loss of vascular tone or a combination of these factors. Causes include anaphylaxis and spinal shock.

● Obstructive: produced by the impairment of venous return to the heart. Causes include cardiac tamponade or tension pneumothorax

SPSE 1

How well did you know this?
1
Not at all
2
3
4
5
Perfectly
56
Q

Which of following statements is false?

A Cardiac output is the product of stroke volume and heart rate.

B Stroke volume is dependent upon ventricular filling (preload), myocardial contractility and systemic vascular resistance (afterload).

C Oxygen delivery is the product of cardiac output and oxygen content.

D Oxygen content is determined by the total oxygen in whole blood available to tissues.

E Mixed venous oxygen tension (SvO2 ) is an accurate reflection of oxygen delivery.

A

E Mixed venous oxygen tension (SvO2 ) is an accurate reflection of oxygen delivery.

The statements in options A, B, C and D are correct.

Svo2 reflects the oxygen content in blood returning to the heart after delivery.

It is dependent upon oxygen delivery, but importantly also on oxygen consumption at the tissue level.

How well did you know this?
1
Not at all
2
3
4
5
Perfectly
57
Q

Which of the following associations is incorrect?

A Hypovolaemic shock:
↓ Preload, ↑ HR, ↑↑ SVR

B Neurogenic shock:
– Preload, ↑ HR, ↓ SVR

C Cardiogenic shock:
– Preload, ↑ HR, ↑ SVR

D Obstructive shock:
↓ Preload, ↑ HR, ↑ SVR

E Septic shock:
– Preload, ↑ HR, ↑ SVR

A

B Neurogenic shock:
– Preload, ↑ HR, ↓ SVR

Hypovolaemic shock results in a decrease in circulating blood volume and a drop in left ventricular volume (preload).

Compensatory mechanisms include tachycardia and an increase in systolic vascular resistance to maintain cardiac output.

Neurogenic (or spinal) shock is a form of distributive shock, whose primary event is loss of vascular tone. While circulatory volume (preload) remains unchanged, neurogenic shock is characterised by an inappropriate absence of tachycardia.

Cardiogenic shock is characterised by a primary failure of myocardial activity. There is no change in preload, and tachycardia and vascular constriction are compensatory mechanisms.

Obstructive shock results from a primary decrease in venous return to the heart (preload) secondary to an ‘obstruction’. Tachycardia and ↑ SVR are compensatory.

Septic shock is often described as either a ‘warm’ and ‘cold’ phase.

Answer ‘E’ accurately portrays the more commonly observed ‘cold’ phase of septic shock with primary myocardial dysfunction followed by tachycardia and increased vascular tone.

SPSE 1

How well did you know this?
1
Not at all
2
3
4
5
Perfectly
58
Q

What are the mechanisms of action of the different vasopressors and inotropes used in children?

A

(SPSE 1)

How well did you know this?
1
Not at all
2
3
4
5
Perfectly
59
Q

Which of the following is true regarding the outcomes of shock?

A Colloid resuscitation has been demonstrated to be superior to crystalloid in the management of shock in the neonate.

B A superior outcome is associated with early resuscitation in the paediatric patient with shock, regardless of aetiology.

C Volume restriction in paediatric septic shock has been associated with decreased mortality.

D Shock-related mortality is dependent on normalising and maintaining blood pressure.

E Liberal fluid resuscitation has resulted in increasing rates of acute respiratory distress syndrome (ARDS).

A

B A superior outcome is associated with early resuscitation in the paediatric patient with shock, regardless of aetiology.

No difference in outcome has been demonstrated when comparing crystalloids with colloids in paediatric shock, including the neonate.

A possible indication for colloid is in the context of a congenital heart anomaly and the concern of fluid overload.

Multiple studies in the adult and paediatric literature have demonstrated improved mortality rates with early, goal-directed therapy. This is the single greatest parameter that can be modified by healthcare providers – including emergency room physicians.

Mortality is significantly decreased when children with septic shock receive early, liberal fluid resuscitation when compared with those who are volume restricted.

Shock-related mortality is independent of maintaining or normalising the blood pressure alone, but is directly related to low cardiac index and low mixed venous oxygen saturations.

Furthermore, Carcillo and Fields (2002) did not find a difference in the occurrence of ARDS between groups with restricted fluid resuscitation compared with those who received liberal fluid resuscitation.

SPSE 1

How well did you know this?
1
Not at all
2
3
4
5
Perfectly
60
Q

Which of the following is not a well-demonstrated surrogate marker of tissue perfusion?

A serum bicarbonate
B serum pH
C serum glucose
D serum lactate
E base deficit

A

C serum glucose

Serum pH, lactate, base deficit and bicarbonate are accepted surrogate markers of tissue perfusion that correlate with the severity of shock and the adequacy of resuscitation.

Normalisation of serum bicarbonate and lactate are associated with patient recovery in paediatric septic shock.

Successful resuscitation should be accompanied by a decreasing anion gap, decreasing lactate and improving base excess.

While increases in serum glucose have been associated with an increased risk of death, hyperglycaemia indicates excessive circulating counterregulatory hormones (like catecholamine) that result in the failure of insulin to suppress hepatic gluconeogenesis.

Glucose is not a surrogate marker for tissue perfusion.

SPSE 1

How well did you know this?
1
Not at all
2
3
4
5
Perfectly
61
Q

Which of the following is true regarding relative adrenal insufficiency?

A It occurs in nearly 25% of all paediatric intensive care unit (PICU) admissions for septic shock.

B Routine administration of steroid replacement therapy is warranted in cases of hypovolaemic shock.

C Patients who increase serum cortisol levels in response to test-dose corticotropin have a 60% mortality compared with patients who do not.

D Steroid therapy should be given in septic shock if hypotension is refractory to vasopressor therapy.

E Steroids should be administered only in children with suspected or proven adrenal insufficiency.

A

E Steroids should be administered only in children with suspected or proven adrenal insufficiency.

Relative adrenal insufficiency is common in the PICU, occurring in over 75% of patients in septic shock.

Steroid replacement has no direct role in the care of hypovolaemic shock, unless an underlying steroid dependence exists.

Adrenal suppression is diagnosed by a failure to increase cortisol levels after the administration of a test dose of corticotropin.

Non-responders who fail to increase serum cortisol levels after this test dose have a 60% mortality compared with responders (normal adrenal function).

Routine treatment of paediatric patients with stress-dose steroids cannot be recommended, as stated by the Surviving Sepsis Campaign.

This publication recommends administering steroid therapy to adults in septic shock who respond poorly to vasopressors.

However, evidence in the paediatric population is lacking.

Nonetheless, in children with catecholamine-refractory shock or in whom the presence of risk factors (such as history of chronic or recent high-dose steroid treatment) is present, steroid administration should be considered.

SPSE 1

How well did you know this?
1
Not at all
2
3
4
5
Perfectly
62
Q

Changes in a child’s cardiac output is most dependent upon which of the following variables?

A end-diastolic volume
B blood pressure
C heart rate
D stroke volume
E myocardial contractility

A

C heart rate

Cardiac output = heart rate × stroke volume

Children are mostly dependent on their heart rate to increase cardiac output.

Stroke volume (the volume of blood pumped from one ventricle of the heart with each beat) cannot be altered in children to the same degree as in adults.

Factors that affect stroke volume include blood pressure (afterload), end-diastolic volume (preload) and myocardial contractility.

Blood pressure and end-diastolic volume have limited ranges while the ability to increase contractility in response to catecholamine stimulation is limited because of insufficient muscle mass and ‘stiffness’ of the young myocardium as compared with the adult heart.

SPSE 1

How well did you know this?
1
Not at all
2
3
4
5
Perfectly
63
Q

Which of the following is essential in the definition of shock?

A inadequate systemic oxygen and nutrient supply to meet metabolic demands

B the delivery of oxygen (DO2) is less than oxygen consumption (VO2 )

C hypotension refractory to volume and vasopressors

D A and B

E all of the above

A

Shock ensues when systemic oxygen and nutrient supply become acutely inadequate to meet the metabolic demands of the body’s organ systems. This can be expressed in mathematical terms, where:

Do 2 (oxygen delivery) < Vo 2 (oxygen consumption).

Hypotension is often associated with shock states, but it is not necessary.

Indeed, fatal shock can ensue in spite of a normal arterial blood pressure if metabolic demands are not being met.

SPSE 1

How well did you know this?
1
Not at all
2
3
4
5
Perfectly
64
Q

Shock in the neonatal period has several unique characteristics. All are true except:

A the development of pulmonary hypertension

B may benefit from prostaglandin infusion

C may result from maternal factors

D shock is less likely, given the increased heart rate of the newborn

E may benefit from indomethacin.

A

D shock is less likely, given the increased heart rate of the newborn

Several important differences exist in neonates as they transition from fetal to extrauterine physiology.

A patent ductus arteriosus (PDA) may contribute to or cause shock through the failure of a compensatory increase in the cardiac output. This may be secondary to myocardial immaturity or a ductal steal phenomenon, which accounts for a uniform reduction in systolic and diastolic blood pressure. This is particularly true of extremely premature infants, who benefit from efforts to close this shunt.

A PDA may close after a dose of indomethacin or ibuprofen. Alternatively, surgical closure is sometimes required.

On the other hand, several duct-dependent cardiac lesions present in shock that is refractory to fluid therapy. These neonates benefit from early prostaglandin infusion to maintain duct patency, and require a cardiac echocardiogram to confirm the diagnosis.

Neonates are also at risk for the development of pulmonary hypertension. This results in increased afterload for the right ventricle and reduced pulmonary blood flow that ultimately compromises oxygen exchange and worsens oxygen delivery.

Other causes of neonatal shock include hypovolaemia (GI losses, haemorrhage), obstructive shock (tension pneumothorax) and cardiogenic shock (arrhythmias).

Neonates are uniquely at risk for developing sepsis as a consequence of maternal infection, most commonly Group B Streptococcus. This should be considered in any newborn manifesting evidence of unexplained shock.

Neonates are less able to vary their heart rate to meet metabolic demands, placing them at an increased risk of developing shock.

SPSE 1

How well did you know this?
1
Not at all
2
3
4
5
Perfectly
65
Q

Regarding vasopressin in the treatment of paediatric septic shock, which of the following is incorrect?

A It significantly reduces 28-day mortality when compared with placebo for paediatric patients in septic shock.

B It may be useful in cases of hypotension unresponsive to conventional vasopressor therapy.

C It acts via a catecholamine-independent V1 vascular receptor.

D It has a direct antidiuretic effect on the distal renal tubule and collecting duct.

E It may reduce cardiac output by increasing afterload.

A

A

Vasopressin, otherwise know as antidiuretic hormone, is a peptide hormone with multiple effects, mediated by different receptors.

It acts through a second messenger system to increase water absorption in the distal renal tubule and collecting duct.

It also acts through a catecholamine–independent V1 receptor on vascular endothelium to promote vasoconstriction.

This increases afterload, which can lower cardiac output in the absence of compensatory changes in heart rate or contractility.

While vasopressin has been demonstrated to increase mean arterial pressure and improve urine output in patients with catecholamine-unresponsive hypotension, a recent randomised, multicentre trial (Choong et al., 2009) investigating low-dose vasopressin in the treatment of paediatric vasodilatory shock did not demonstrate any beneficial effects.

Although not statistically significant, there was a concerning trend towards increased mortality.

SPSE 1

How well did you know this?
1
Not at all
2
3
4
5
Perfectly
66
Q

A 16-year-old presents to the trauma bay after sustaining a gunshot wound to the abdomen. His pulse is 130 beats per minute, and his blood pressure is 80/50. He is anxious but conscious and orientated. A Foley catheter drains a small amount of urine output. His estimated blood loss is:

A <15%
B 15%–25%
C 25%–40%
D >40%
E not estimable given above information.

A

C

While paediatric patients tend to maintain normal vital signs in the face of hypovolaemia, this adolescent is manifesting evidence of significant blood loss.

His tachycardia and hypotension suggest blood loss greater than 25% of his circulating blood volume, but the maintenance of end-organ function (mental activity, renal function) suggests that the blood loss does not exceed 40%.

How well did you know this?
1
Not at all
2
3
4
5
Perfectly
67
Q

A 6-year-old child who weighs 30 kg presents to the hospital with fever, tachycardia (pulse = 140) and mental status changes. She has a history of urinary tract infections. Intravenous access is obtained and three successive boluses of 600 mL normal saline are administered. She is started on antibiotic therapy but continues to have delayed capillary refill and remains lethargic. After transfer to the intensive care unit, the next step in her care is:

A transfuse with 10 mL/kg of packed red blood cells

B administer hydrocortisone

C begin a milrinone infusion at 0.25 mcg/kg/min

D begin a dopamine infusion at 10 mcg/kg/min

E consider initiating extracorporeal life support (ECLS).

A

D

This child presents with fluid refractory septic shock. The next appropriate step is to begin a dopamine infusion.

The dosage should be adequate to activate adrenergic receptors and thereby provide chronotropy, inotropy and peripheral vasoconstriction.

There is no reason to suspect acute blood loss; a transfusion is not indicated at this time.

Milrinone may be useful if there is evidence of myocardial hypocontractility, but it is also not indicated at this time.

Hydrocortisone is important if the child has documented or suspected adrenal insufficiency.

While EClS has been shown to be of value in the treatment of septic shock, it should be instituted only after other treatment options (catecholamines) have been exhausted and a shock state persists.

SPSE 1

How well did you know this?
1
Not at all
2
3
4
5
Perfectly
68
Q

As compared with crystalloid resuscitation, which of the following is a documented advantage of colloid fluid resuscitation?

A decreased cost
B decreased risk of allergic reaction
C decreased volume of fluid required
D decreased risk of disease transmission
E decreased overall mortality rates

A

C

Colloid solutions effectively restore blood pressure, especially in young neonates. However, there is a concern over the potential adverse effects of using natural and synthetic colloids, which include infectious disease transmission/exposure and the potential for allergic reactions.

In a recent paediatric open-label trial, children with septic shock were randomised to receive either normal saline or gelatin polymer in saline. Both groups achieved haemodynamic stability, and mortality and organ failure rates were similar in both groups.

The colloid-resuscitated group required 40% less volume than the saline-resuscitated group.

For this reason, the use of colloids and/or hypertonic solutions may be of particular benefit in postoperative cardiac surgery patients where volume load is a concern.

SPSE 1

How well did you know this?
1
Not at all
2
3
4
5
Perfectly
69
Q

Signs of compensated shock include all of the following except:

A tachycardia
B cool extremities
C prolonged capillary refill (despite warm ambient temperature)
D weak peripheral pulses compared with central pulses
E low blood pressure

A

E

Shock progresses over a continuum of severity, from a compensated to a decompensated state.

Physiologic attempts to compensate include tachycardia and increased systemic vascular resistance (vasoconstriction) to maintain cardiac output and blood pressure.

As compensatory mechanisms fail, signs of inadequate end-organ perfusion develop.

In addition to the above, these signs include hypotension and evidence of end-organ dysfunction (mental status changes, decreased urine output, metabolic acidosis).

Hypotension is defined based on systolic blood pressure and the patient’s age:

● <60 mmHg in term neonates (0–28 days)

● <70 mmHg in infants (1 month to 12 months)

● <70 mmHg + (2 × age in years) in children 1–10 years

● <90 mmHg in children ≥10 years of age.

SPSE 1

How well did you know this?
1
Not at all
2
3
4
5
Perfectly
70
Q

What is the most frequent cause of death in children between 1 and 14 years old?

A cancer
B congenital malformation
C trauma
D infectious diseases
E heart disease

A

C

Trauma is the leading cause of death in children between ages 1 and 14 years, accounting for more than 5000 deaths per year.

The second most common cause of death varies according to the age group.

For children between 1 and 4 years of age, congenital anomalies are the second leading cause of death, whereas for children between 5 and 14 years of age, it is cancer.

For children between 0 and 1 year of age, the leading causes of death are congenital anomalies, short gestation and sudden infant death syndrome.

Trauma is the fifth leading cause of death in that age group.

SPSE 1

How well did you know this?
1
Not at all
2
3
4
5
Perfectly
71
Q

What is the most frequent cause of paediatric trauma death?

A head trauma
B thoracic trauma
C abdominal trauma
D spinal trauma
E drowning

A

A

The leading cause by far of traumatic death in children is head trauma.

It accounts for more than 80% of paediatric trauma mortality.

Overall, the incidence of traumatic brain injuries in the united States is 200 per 100 000 children.

About 5% of all these brain injuries are fatal.

It is estimated that around 3000 children die of head injury each year in the united States.

SPSE 1

How well did you know this?
1
Not at all
2
3
4
5
Perfectly
72
Q

Which trauma mechanism is the leading cause of death in children?

A fall

B motor vehicle collision – occupant

C motor vehicle collision – pedestrian

D bicycle

E drowning

A

B

Motor vehicle–related injuries account for approximately 50% of all traumatic deaths in children.

According to the Center for Disease Control and Prevention (CDC) in 2007, the most frequent mechanism of unintentional death for children aged between 1 and 14 years is motor vehicle collisions in which children are occupants.

The other leading causes of unintentional death in this age group, in order of frequency, are drowning and pedestrians hit by motor vehicles.

However, in children aged between 1 and 4 years, drowning is the leading cause of death, accounting for 28% of all unintentional injuries.

The proportion of traffic-related death increases as the age of the child increases.

Although falls remain the most common mechanism of injury in children, they are less commonly fatal.

SPSE 1

How well did you know this?
1
Not at all
2
3
4
5
Perfectly
73
Q

Which one of the following is not classically associated with nonaccidental trauma?

A subdural haematoma
B posterior rib fractures
C retinal haemorrhages
D spiral fracture of humerus
E pelvic fracture

A

E

There are four reported types of child abuse: neglect, physical abuse, sexual abuse and emotional abuse.

In 2008, the CDC estimated that 772 000 children were victims of maltreatment.

It was also estimated that 1740 children died from abuse, with 80% of these deaths being in children under 4 years of age.

Factors that should increase the suspicion of child abuse include delay in seeking medical care, vague or inconsistent history reported by caregivers, as well as specific patterns of injuries.

For example, the presence of subdural haematoma and retinal haemorrhages is essentially pathognomonic of shaken baby syndrome.

Because the chest wall of a child is very compliant, the presence of rib fractures, especially posterior rib fractures, implies significant force was exerted, more so than can be explained by simple falls.

Likewise, spiral fractures of long bones and buckethandle fractures are classically associated with non-accidental trauma.

Although pelvic fractures may occur, they are not classically associated with non-accidental trauma, except in extreme cases.

SPSE 1

How well did you know this?
1
Not at all
2
3
4
5
Perfectly
74
Q

Waddell’s triad is a pattern of injuries that occur when pedestrian children are struck by motor vehicles. What are the injuries that comprise this triad?

A head, neck and torso (chest/abdomen)

B head, torso (chest/abdomen) and lower extremities

C head, neck and upper extremities

D neck, torso (chest/abdomen) and upper extremities

E neck, torso (chest/abdomen) and lower extremities

A

B

Waddell’s triad of injuries refers to injuries to head, torso (chest/abdomen) and lower extremities.

This is a pattern of injury that is seen among pedestrians hit by cars.

The first point of impact is to the lower extremities as the bumper of the car strikes the child;

the second point of impact is to the torso (chest/abdomen) from the front of the car (bumper or bonnet);

and the third point of impact is to the head as the child is thrown to the windscreen or the ground.

Commonly, the injuries to the torso (chest/abdomen) or lower extremity are on the same side and the injury to the head is on the opposite side.

Although this triad of injuries is not always present, it emphasises the importance of mechanism of injury in predicting specific injury patterns useful in the assessment of the injured child.

SPSE 1

How well did you know this?
1
Not at all
2
3
4
5
Perfectly
75
Q

Which of the following is the most frequent cause of airway obstruction in the injured child?

A vomit
B foreign body
C tongue
D laryngospasm
E adenoid tissue and tonsils

A

C

The most frequent cause of airway obstruction in the injured child occurs when the tongue falls back and obstructs the glottis.

This can be prevented by using a simple chin-lift or jaw-thrust manoeuvre, always taking care to maintain in-line cervical traction in case of cervical spine injury in the setting of trauma.

Children with airway obstruction may also benefit from insertion of an oropharyngeal or nasopharyngeal airway.

However, if a child can tolerate such an airway adjunct, there is a need for respiratory assistance because it indicates an altered level of consciousness such that the child can no longer protect his or her airway.

Other causes of upper airway obstruction in the trauma patient include foreign bodies (i.e. broken teeth), secretions (i.e. blood) and vomit.

SPSE 1

How well did you know this?
1
Not at all
2
3
4
5
Perfectly
76
Q

Which of the following is not true regarding differences in the paediatric airway?

A A straight laryngeal blade may be preferred for intubation of young children because of an omega-shaped epiglottis.

B Intubation of the right mainstem bronchus is more frequent than in adults.

C The narrowest part of the airway is the thyroid cartilage.

D The larynx is more cephalad and anterior in children.

E The infant must be placed in a different position in order to avoid airway obstruction.

A

C

There are many differences between children and adults that must be considered during evaluation of the paediatric airway.

The tongue is a major cause of upper airway obstruction.

A straight laryngeal blade is generally preferred for endotracheal intubation because of the omega-shaped, floppy epiglottis in children, although there are those who prefer a curved blade.

The trachea is shorter, which results in more frequent right mainstem bronchus intubation.

The correct position of the endotracheal tube must be confirmed by both auscultation and chest X-ray.

During intubation, it is also important to remember that the larynx is more cephalad and anterior in children, making it more difficult to visualise the vocal cords.

Finally, the narrowest part of the airway in children is the cricoid cartilage. Because of this, uncuffed endotracheal tubes are generally preferred in children up to the age of 8 years.

A cuffed endotracheal tube can be used, but one needs to be careful to avoid overinflation of the balloon, as it can lead to tracheal injury, necrosis and subglottic stenosis.

Because the head of an infant or young child is much larger proportional to the body, and the neck is shorter, there is a propensity for the posterior pharynx to buckle anteriorly, thereby obstructing the airway and causing passive flexion of the cervical spine.

The child must be positioned such that the plane of the face is parallel to the backboard.

This may be accomplished by placing a 1-inch layer of padding beneath a child’s torso to preserve the neutral alignment of the spine.

SPSE 1

How well did you know this?
1
Not at all
2
3
4
5
Perfectly
77
Q

In children under 1 year of age, what drug is generally administered in combination with succinylcholine as part of rapid-sequence intubation?

A epinephrine
B atropine
C rocuronium
D fentanyl
E lidocaine

A

B

Rapid-sequence intubation is recommended for children who need to be intubated urgently unless they are already in cardiac arrest.

Following preoxygenation, a sedative and paralytic are administered.

Cricoid pressure is maintained to prevent aspiration of gastric contents.

The sedative agent of choice is often etomidate, which can be safely used in haemodynamically unstable and brain-injured patients.

In children, the paralytic agent of choice is often succinylcholine unless contraindicated (i.e. burns, crush injury).

For children in whom elevated intracranial pressure is suspected, premedication with lidocaine is recommended.

Furthermore, in children under 1 year of age, children younger than 5 years of age receiving succinylcholine, and in all children receiving a second dose of succinylcholine, atropine should be given to prevent bradycardia.

Atropine may decrease the incidence of bradycardia associated with direct laryngoscopy (stimulation of parasympathetic receptors in the laryngopharynx) and administration of succinylcholine (direct stimulation of cardiac muscarinic receptors).

SPSE 1

How well did you know this?
1
Not at all
2
3
4
5
Perfectly
78
Q

A 5-year-old child presents to the emergency department after being hit by a car. During the initial assessment, auscultation of the chest reveals decreased breath sounds on the right side. The trachea is also deviated to the left side. What should be done first?

A Administer 100% oxygen by a non-rebreather mask.

B Perform a needle decompression of the right chest in the second intercostal space in the mid-clavicular line.

C Insert a chest tube in the right chest at the fourth intercostal space in the anterior axillary line.

D Perform a chest X-ray immediately.

E Perform a computed tomography (CT) scan of the chest immediately.

A

B

A tension pneumothorax is a clinical diagnosis and should be addressed as soon as suspected.

The correct management is to perform a needle decompression (usually with a 14-gauge needle) in the second intercostal space in the mid-clavicular line, just above the third rib.

This can be achieved in a few seconds and the primary survey can then proceed.

After a needle decompression is done, a chest tube should be inserted in the fourth intercostal space along the anterior axillary line, above the rib to avoid the intercostal vessels.

This is usually at the level of the nipple.

There is no place for a chest CT or chest X-ray in the immediate management of a suspected tension pneumothorax.

SPSE 1

How well did you know this?
1
Not at all
2
3
4
5
Perfectly
79
Q

A child presents to the emergency department after a motor vehicle collision. He has an obvious open femur fracture that is bleeding. What is the first step in fluid resuscitation?

A transfuse 10 mL/kg of packed red blood cells (PRBCs)

B transfuse 20 mL/kg of PRBCs

C bolus 10 mL/kg of normal saline (NS)

D bolus 20 mL/kg of NS

E bolus 20 mL/kg of colloid

A

D

In the paediatric trauma patient, fluid resuscitation is begun by bolus administration of 20 ml/kg of NS or lactated Ringer’s solution (lR).

As soon as hypovolaemic shock is suspected, a type and cross-match should be sent.

If the child needs further fluid resuscitation, a second bolus of 20 ml/kg of NS or lR should be given.

If the child does not respond appropriately to crystalloid infusion, blood transfusion is begun with 10 ml/kg of PRBCs.

If cross-matched blood is available, it should be given. If not, O-negative blood should be given until crossmatched blood becomes available.

There is no place for colloid administration in the acute trauma setting.

SPSE 1

How well did you know this?
1
Not at all
2
3
4
5
Perfectly
80
Q

Which of the following is not a potential site of haemorrhage leading to hypotensive shock in children?

A intracranial
B thorax
C abdomen
D pelvis
E femur

A

A

Haemorrhagic shock is classified in four categories based on the estimated blood loss (EBl).

Class I shock is less than 15% EBL,
class II is 15%–30% EBL,
class III is 30%–40% EBL, and
class IV is more than 40% EBL.

Patients in class I shock usually show minimal findings;

patients in class II shock are mildly anxious with tachycardia, have normal blood pressure (BP) with narrow pulse pressure, and prolonged capillary refill;

patients in class III shock may have marked tachycardia, hypotension with narrowed pulse pressure, altered mental status with marked anxiety and confusion, and cool, mottled extremities;

patients in class IV shock show severe signs of hypoperfusion with loss of central and peripheral pulses, wide pulse pressure, loss of consciousness, and cold, cyanotic extremities.

Children have a unique ability to compensate for major blood loss.

Decompensation tends to be abrupt.

Tachycardia precedes hypotension which is a late, ominous sign.

Except for infants with fontanelles that can bulge, intracranial bleeding alone can never account for haemorrhagic shock.

Other sources of haemorrhage should be sought.

SPSE 1

How well did you know this?
1
Not at all
2
3
4
5
Perfectly
81
Q

A 1-year-old child presents to the emergency department after sustaining a fall from the arms of his parent. His vital signs are the following: respiratory rate 35/min, heart rate 150/min, BP 80/40. What is the correct diagnosis?

A This child is in respiratory distress and needs emergency intubation.

B This child is hypotensive and needs emergency fluid resuscitation.

C This child shows signs of intracranial bleeding and needs a head CT immediately.

D This child needs to be monitored in an intensive care unit because of his unstable condition.

E This child has normal vital signs for his age.

A

E

It is important for the trauma care provider to be familiar with normal vital signs for each paediatric age group. In children 1–10 years of age, a formula that can be used to approximate the normal systolic BP is 70 + 2 × (age in years). Normal vital signs values for different age groups are provided in Table 7.1.

How well did you know this?
1
Not at all
2
3
4
5
Perfectly
82
Q

A 5-year-old child presents to the emergency department after being involved in a high-speed motor vehicle collision. He is hypotensive and needs immediate fluid resuscitation. Nurses are unable to insert a peripheral intravenous (IV) catheter after several attempts. What should be done next?

A Try to insert a small-bore peripheral IV catheter yourself.

B Insert a central venous catheter in the femoral vein.

C Insert a peripheral venous catheter in the external jugular vein.

D Perform a saphenous vein cutdown.

E Insert an intraosseous (IO) cannula

A

E

Placement of peripheral IVs can be difficult in young children, especially in the presence of shock.

After unsuccessful attempts at inserting a peripheral IV catheter, the next step should be to obtain Io access.

The preferred sites are the proximal tibia and distal femur.

IO access should not be obtained distal to a fracture.

If there is a lower extremity or pelvis fracture, it is possible to insert an IO cannula in the proximal humerus.

IO access can be obtained in children who are conscious or unconscious.

IO access is generally safe, efficacious and expeditious.

Complications of IO infusion include osteomyelitis, compartment syndrome and iatrogenic fracture.

Other options for venous access are to insert a femoral central line (unless there is a suspicion of pelvis fracture) or perform a venous cutdown at the antecubital or saphenous sites.

If a central line is placed, it should be a large-bore introducer rather than a standard central line.

SPSE 1

How well did you know this?
1
Not at all
2
3
4
5
Perfectly
83
Q

A toddler is being evaluated in the emergency department after a motor vehicle collision. He is haemodynamically stable but his abdomen is markedly distended on physical exam. What should be done next?

A Perform an abdominal CT immediately.

B Take the patient to the operating room for an emergency exploratory laparotomy.

C Perform a diagnostic peritoneal lavage.

D Transfuse 10 mL/kg PRBCs.

E Insert a naso- or orogastric tube and re-evaluate clinically.

A

E

Infants and young children commonly develop a distended abdomen because of a stomach filled with air or gastric contents, often due to crying and screaming.

This can be erroneously mistaken for a sign of an intra-abdominal injury.

Inserting a naso- or orogastric tube should be done rapidly in the paediatric trauma management algorithm.

FAST exam (Focused Assessment with Sonography for Trauma) can also aid in the diagnosis of intra-abdominal injury that would cause abdominal distension.

Diagnostic peritoneal lavage is rarely performed in children.

SPSE 1

How well did you know this?
1
Not at all
2
3
4
5
Perfectly
84
Q

A child presents to the emergency department after being involved in a high-speed motor vehicle collision. He initially presents with a Glasgow Coma Scale (GCS) score of 12 and gradually improves to a score of 15. He is now complaining of weakness and paraesthesias in both upper extremities, although his sensory and motor exam are otherwise normal. He is still wearing a cervical collar. As part of his initial evaluation, a head and cervical spine CT were obtained, both of which were normal. What should you do?

A Discontinue the cervical collar since there is no cervical spine fracture on CT.

B Discontinue the cervical collar and perform flexion- extension X-rays.

C Repeat the cervical spine CT.

D Discontinue the cervical collar and ask for a neurology consult.

E Leave the cervical collar in place and ask for a neurosurgery consult.

A

E

Spinal cord injury without radiographic abnormality (SCIWoRA) is a phenomenon that is more commonly seen in children than in adults because of the laxity of the cervical ligaments in children.

In children younger than 8 years of age, SCIWoRA accounts for 30%–40% of all spinal cord injuries.

A thorough neurologic evaluation and examination of the cervical spine is crucial to exclude spinal injury in a child with a suspicious mechanism of injury, and to determine the need for further radiological imaging studies.

In children without a distracting injury, neurologic deficit, and who are not intoxicated, the sensitivity and negative predictive value of the clinical exam approaches 100%.

Children younger than 2 years of age may constitute an exception.

For children with midline tenderness or deformity on exam, a standard three-view X-ray of the cervical spine (crosstable lateral, anteroposterior and odontoid) should be done.

If the cervical spine is incompletely visualised on the X-ray and there are symptoms that suggest a cervical spine injury, a cervical spine CT can be done.

If this exam does not show a fracture and the child has either midline tenderness or neurologic symptoms, a cervical spine magnetic resonance imaging scan can be done to evaluate for ligamentous and spinal cord injuries.

A cervical collar should never be removed in the face of neurologic symptoms without consulting a neurosurgeon.

SPSE 1

How well did you know this?
1
Not at all
2
3
4
5
Perfectly
85
Q

What is the GCS score of a child who opens his eyes to pain, moans in response to pain, and has abnormal flexion in response to pain?

A 4
B 5
C 6
D 7
E 8

A

D There is a modified GCS to evaluate infants and children.

Table 7.2 summarises the adult, child and infant GCS.

How well did you know this?
1
Not at all
2
3
4
5
Perfectly
86
Q

Contraindications to insertion of a nasogastric tube include all of the following except:

A Battle’s sign (retroauricular ecchymosis)
B raccoon eyes (periorbital ecchymosis)
C cerebrospinal fluid (CSF) leakage from nose
D CSF leakage from ear
E epistaxis.

A

E

Basal skull fracture is a contraindication to insertion of a nasogastric tube and is suspected if there is a Battle’s sign (retroauricular ecchymosis), raccoon eyes (periorbital ecchymosis), CSF rhinorrhoea or otorrhoea, and haemotympanum.

The patients should be evaluated with a head CT.

Epistaxis alone is not a contraindication to insertion of a nasogastric tube, but severe midface fracture is.

SPSE 1

How well did you know this?
1
Not at all
2
3
4
5
Perfectly
87
Q

Which one of the following statements is true regarding skull fractures?

A Skull fractures are uncommon in young children.

B Linear fractures are less likely to be associated with underlying brain injury or haemorrhage than complex fractures.

C The vast majority of linear skull fractures in young children are caused by road traffic accidents.

D CT scan is the best imaging modality to demonstrate a linear fracture of the skull.

E Growing skull fracture is a well-recognised complication, occurring in 10% of children.

A

B

Skull fractures are common in young children and represent the most common abnormal radiographic finding.

Linear fractures in children may be associated with haemorrhage or significant underlying brain injury, but usually are not.

Multiple fractures, and complex fractures crossing venous sinuses, are usually associated with underlying brain injury or haemorrhage.

The vast majority of linear fractures in young children are caused by falls.

Plain radiograph is the best modality to identify a linear fracture; an axial slice of CT scan may occasionally miss an axially orientated fracture.

Growing skull fracture is a well-recognised complication occurring in 1% of children with skull fractures.

They occur because of underlying dural laceration.

SPSE 1

How well did you know this?
1
Not at all
2
3
4
5
Perfectly
88
Q

Which one of the following statements is true with regard to infantile acute subdural haematoma?

A It is usually due to falling from a height.

B It results in coma from the onset of the fall.

C It usually occurs in babies <6 months old.

D It usually presents with a generalised seizure.

E It is due to a tear in the branch of the middle meningeal artery.

A

D

Infantile acute subdural haematoma is due to a minor head injury, without initial loss of consciousness, and is usually due to a rupture of a bridging vein.

The most common trauma is a fall backwards from a sitting or standing position.

Patients are usually <2 years old – the age when they first begin to pull themselves up or begin to walk.

They often present with generalised seizures within minutes to an hour following the injury.

SPSE 1

How well did you know this?
1
Not at all
2
3
4
5
Perfectly
89
Q

Which one of the following statements is true with regard to cephalhaematoma?

A Its extent is limited by sutures and hence remains localised to region.

B Bleeding occurs in the loose connective tissue above the periosteum.

C Eighty percent will need evacuation as can result in calcification if not treated.

D It should be treated with aspiration rather than open drainage.

E It is most commonly seen in newborns delivered by caesarean section.

A

A

Cephalhaematoma is most commonly seen in newborns associated with parturition.

Bleeding elevates the periosteum and hence extent is limited by the sutures while a subgaleal haematoma is between the periosteum and the galea in the loose connective tissue and crosses the sutures.

Eighty percent of cephalhaematomas reabsorb over 2–3 weeks and may occasionally calcify.

They should never be aspirated because the risk of infection exceeds the risk following them up expectantly, and in the newborn removal of the blood may make them anaemic.

SPSE 1

How well did you know this?
1
Not at all
2
3
4
5
Perfectly
90
Q

Which one of the following is not a characteristic finding in shaken baby syndrome?

A retinal haemorrhages
B bilateral subdural haematomas in 80% of cases
C multiple fractures of ribs
D significant brain injury
E extensive external signs of trauma

A

E

All the statements are correct except that there may be few or no external signs of trauma. In some cases there may be finger marks on the chest.

SPSE 1

How well did you know this?
1
Not at all
2
3
4
5
Perfectly
91
Q

Which of one the following statements is true with regard to the outcome following head injury?

A The outcome of epidural haematomas overall is unfavourable.

B For children needing a craniotomy for subdural haematoma, a mortality as well as morbidity of about 30% has been described.

C Less than 10% of severely head-injured children with mass lesion die or remain severely disabled.

D Previously head-injured children who arrest or regress in their recovery are likely to have hydrocephalus.

E Post-traumatic seizures occur more frequently in younger adults than in children.

A

D

The overall outcome of extradural haematomas is favourable.

In children needing craniotomy for subdural haematoma a mortality as well as morbidity of 8% has been described.

In severely head-injured children with mass lesions, 50% either died or remained severely disabled.

If a patient arrests or regresses in their recovery following a head injury they should be scanned to rule out post-traumatic hydrocephalus, which can develop as late as 2–3 years following the initial injury.

Post-traumatic seizures are a well-recognised complication of head injury. They are more common in children and older adults.

SPSE 1

How well did you know this?
1
Not at all
2
3
4
5
Perfectly
92
Q

Which one of the following statements is true with regard to atlantoaxial rotatory subluxation?

A A history of trauma is always present.

B Cock robin head position is the characteristic deformity.

C Antibiotics along with anti-inflammatory medications will result in spontaneous correction of the deformity.

D Reduction and internal fixation as early as possible is the gold standard in its management.

E MRI scan should be done as a priority in its management.

A

B

Atlantoaxial rotatory subluxation can occur spontaneously – with rheumatoid arthritis, following minor or major trauma, or even with infection of the head or neck including upper respiratory tract infection (known as Grisel’s syndrome).

Patients are usually young and neurologic deficit is rare.

The characteristic finding is a ‘Cock robin’ head position with torticollis.

Manual reduction, either with traction or under anaesthesia, and immobilising external bracing (with a wellfitting minerva jacket or a halo ring vest) for 6 weeks to 3 months, will usually be sufficient.

In cases with upper respiratory tract infection a course of antibiotics will be necessary in addition to reduction and immobilisation.

X-ray and dynamic CT scan of the spine is usually diagnostic.

MRI may be undertaken only to assess competency of the transverse ligament.

SPSE 1

How well did you know this?
1
Not at all
2
3
4
5
Perfectly
93
Q

Which of one the following statements is true with regard to thoracic spine injuries in children?

A They are relatively common compared with cervical spine injuries.

B They are commonly avulsion-type fractures of the superior spinous processes.

C In the young child, every effort should be to treat them with closed reduction and external immobilisation.

D The majority will have purely thoracic spine injury.

E They never cause neurological deficits.

A

C

Injuries to the thoracic spine are relatively uncommon compared with cervical injuries in children (13% cases).

Hadley et al. reported only 11 out of 122 children with purely thoracic spine injuries.

These injuries are usually due to axial load, or high-velocity injuries where the flexion force is transmitted into the thoracic spine.

They are usually seen as wedge fractures or a three-column injury.

Every effort should be made, especially in the young child, to treat these with closed reduction and external bracing because operative fixation, which may need multiple segment fixation, can result in growth restriction.

If there is canal compromise there can be cord compression and a partial neurological deficit, in which case surgery is warranted.

SPSE 1

How well did you know this?
1
Not at all
2
3
4
5
Perfectly
94
Q

Which of one the following statements is true with regard to the epidemiology of spinal cord injury?

A About 12% of all paediatric trauma patients suffer some form of either spinal column or spinal cord injury.

B Cervical and lumbar spinal cord injury tends to be complete while the thoracic spinal cord injury tends to be incomplete.

C The mortality rate associated with paediatric spinal cord injury is estimated to be 2.5 times higher than that for spinal cord injury in adults.

D Survivors of spinal cord injury have improved outcome and recovery in adults than in children.

E In the paediatric group, the risk of spinal cord injury increases with younger age.

A

C

Approximately 3.4% of all paediatric trauma patients suffer some form of either spinal column or spinal cord injury.

The risk of spinal cord injury is increased with older age, more severe overall trauma, and other systemic injury like chest and abdomen.

Cervical and lumbar spinal cord injuries tend to be incomplete; thoracic spinal cord injury is more often complete.

Compared with adults the mortality associated with spinal cord injury is 2.5 times higher in children; whereas children have a better recovery and improved outcome among the survivors than adults.

SPSE 1

How well did you know this?
1
Not at all
2
3
4
5
Perfectly
95
Q

All the following are indications for elevating a depressed skull fracture in a child except:

A to reduce post-traumatic epilepsy
B to correct cranial deformity and help cosmesis
C to repair dural laceration
D to treat compound depressed fractures
E to reduce any mass effect.

A

A

Elevation of the depressed fracture neither decreases the incidence of posttraumatic epilepsy nor substantially improves associated neurologic deficits.

SPSE 1

How well did you know this?
1
Not at all
2
3
4
5
Perfectly
96
Q

Which of the following statements is not true for children with severe traumatic brain injury?

A All children with a Glasgow Coma Scale (GCS) score of 8 or less should be electively intubated and ventilated.

B They should have an urgent CT scan of the head once stabilised.

C Malignant intracranial hypertension is a common complication of severe traumatic brain injury.

D Intracranial pressure monitoring is indicated.

E Pupillary size and light reaction are the early indicators of intracranial hypertension.

A

E

All children with a GCS score of 8 or less following traumatic brain injury should be electively intubated and ventilated to prevent hypoxia and secondary brain injury.

They should be stabilised and then a CT scan of the head should be performed to look for any surgical lesion.

malignant intracranial hypertension is a common complication of severe traumatic brain injury.

It has long been recognised that clinical signs like pupillary size and light response fail as early indicators of intracranial hypertension.

Intracranial pressure monitoring is indicated as it provides a window into the global pressure status of the brain and helps manage the cerebral perfusion pressure.

SPSE 1

How well did you know this?
1
Not at all
2
3
4
5
Perfectly
97
Q

What type of thoracic traumatic injury is the most frequent in children?

A rib fractures
B flail chest
C lung contusion
D pneumothorax
E haemothorax

A

C

Overall, traumatic thoracic injuries account for only 5%–12% of trauma admissions in children.

Most thoracic injuries are the result of blunt trauma.

Rib fractures are not as frequent in children as in adults because of the greater flexibility of the thoracic cage in children.

Therefore, energy transmitted to the thorax can cause a significant lung contusion without any rib fractures.

For the same reason, flail chest is also very rare in children.

Pneumothorax and haemothorax are usually associated with rib fractures.

SPSE 1

How well did you know this?
1
Not at all
2
3
4
5
Perfectly
98
Q

What type of injury is not associated with first-rib fracture?

A clavicular fracture
B spinal cord injury
C aortic injury
D subclavian vein injury
E liver laceration

A

E

A first-rib fracture implies a significant mechanism of injury and should prompt a search for other associated injuries, including aortic injury, clavicular injury, thoracic spine and spinal cord injury, and neurovascular injury (i.e. subclavian vessels, brachial plexus).

There is currently controversy in the trauma literature as to whether or not all patients with first-rib fractures should undergo chest CT-angiography.

However, a chest CT should be done, and if there is any suggestion of aortic injury (i.e. widened mediastinum), a completion CT angiography should be done.

Liver lacerations are more often associated with lower rib fractures.

SPSE 1

How well did you know this?
1
Not at all
2
3
4
5
Perfectly
99
Q

A child is seen in the emergency department after a motor vehicle collision. He has a Glasgow Coma Scale score of 6 and is rapidly intubated in the trauma bay. The intubation is difficult and the patient vomited prior to intubation. An initial chest X-ray showed no intrathoracic injury. Twenty-four hours later, in the intensive care unit, a chest X-ray shows a unilateral basal alveolar infiltrate. What is the most plausible cause?

A lung contusion
B aspiration pneumonia
C ventilator-associated pneumonia
D acute respiratory distress sydrome (ARDS)
E haemothorax

A

B

Lung contusions are usually present on the initial chest X-ray.

Aspiration pneumonia usually becomes visible on chest X-ray within 24–48 hours.

Alveolar infiltrates may be unilateral (most commonly on the right side) or bilateral but are often in the lower lobes.

Ventilator-associated pneumonia is a bacterial pneumonia that develops at least 48 hours after intubation.

Aspiration is also a risk factor for bacterial pneumonia although there are currently no recommendations for prophylactic administration of antibiotics to prevent this complication.

ARDS is a clinical diagnosis that presents with bilateral patchy infiltrates on chest X-ray and requires a PO2 /FiO2 ratio under 200 mmHg for diagnosis.

Haemothorax does not present with alveolar infiltrates on chest X-ray.

SPSE 1

How well did you know this?
1
Not at all
2
3
4
5
Perfectly
100
Q

Which of the following is not a sign of tension pneumothorax?

A deviation of the trachea to the contralateral side
B decreased breath sounds on the ipsilateral side
C distended jugular veins
D tachycardia
E hypertension

A

E

Tension pneumothorax is a clinical, not radiological, diagnosis that should be suspected with any of these findings: deviation of the trachea to the contralateral side, decreased breath sounds on the ipsilateral side, distended jugular veins, tachycardia and hypotension.

Entrapment of air in the pleural cavity leads to a collapse of the ipsilateral lung and a shift of the mediastinum to the contralateral side that subsequently causes decrease in venous return and cardiac output.

This condition needs to be addressed immediately since cardiorespiratory collapse can ensue rapidly.

A tension pneumothorax needs to be decompressed by inserting a 14-gauge needle into the second intercostal space at the mid-clavicular line.

A chest tube must then be inserted.

SPSE 1

How well did you know this?
1
Not at all
2
3
4
5
Perfectly
101
Q

A patient comes to the emergency department after a penetrating injury to the right chest. He presents with a large open wound on the lateral aspect of the chest and has difficulty breathing. What should be done initially?

A Insert a needle in the second intercostal space.

B Insert a chest tube in the fourth intercostal space.

C Apply a completely occlusive dressing over the wound.

D Apply an occlusive dressing taped on three sides.

E Intubate the patient as soon as possible.

A

D

An open pneumothorax seriously compromises ventilation if its diameter is larger than ¾ the diameter of the trachea.

In that case, air preferentially enters the thorax through the chest wound rather than the trachea.

During inspiration, negative intrathoracic pressure is created, and air will enter the pleural cavity through this wound.

If air is allowed to enter and cannot escape because of a tissue flap, a tension pneumothorax can develop.

The initial treatment (even in the field), should be to apply an occlusive dressing taped on three sides to create a flutter-valve effect.

As the patient inhales, the dressing occludes the wound, preventing air entry, but with exhalation, the open end of the dressing allows air to escape.

The definitive treatment is to insert a chest tube and then change the dressing to a completely occlusive dressing.

Applying an occlusive dressing if one is not ready to insert a chest tube may convert an open pneumothorax to a tension pneumothorax.

Intubation needs to be considered but should not delay placement of a chest tube.

Decompressing the chest with a needle should be done only if a tension pneumothorax is suspected.

SPSE 1

How well did you know this?
1
Not at all
2
3
4
5
Perfectly
102
Q

Which finding is not suspicious for cardiac tamponade?

A decreased heart sounds

B jugular venous distension

C widened mediastinum

D decreased bilateral breath sounds

E hypotension

A

D

The classic findings of cardiac tamponade are described in Beck’s triad: decreased heart sounds, jugular venous distension and hypotension.

This is a clinical triad, but all three elements are not always present.

Moreover, it is often difficult, if not impossible, to accurately assess heart sounds in a noisy emergency department environment, especially in children.

However, tamponade should be suspected in presence of any of these findings.

Cardiac tamponade is most commonly due to a penetrating injury. As blood fills the pericardium, venous return and cardiac output are decreased, leading to the clinical triad of symptoms.

A widened mediastinum on chest X-ray should arouse suspicion, especially in the presence of a penetrating chest injury.

It can, however, be difficult to assess the size of the mediastinum on a supine anteroposterior chest X-ray.

Decreased bilateral breath sounds are more often associated with bilateral pneumothoraces, although cardiac tamponade could coexist.

SPSE 1

How well did you know this?
1
Not at all
2
3
4
5
Perfectly
103
Q

A child presents to the emergency department after sustaining a penetrating chest injury just to the left side of the sternum, at the nipple level. He is hypotensive, has decreased heart sounds, and a widened mediastinum on chest X-ray. What should be done immediately?

A Intubate the patient as soon as possible.

B Insert a needle in the second intercostal space.

C Insert a chest tube in the fourth intercostal space.

D Perform a pericardiocentesis.

E Do a chest CT urgently.

A

D

Cardiac tamponade should be a clinical diagnosis based on findings of decreased heart sounds, distended jugular veins and hypotension.

A chest X-ray may reveal a widened mediastinum but management should not be delayed to obtain a chest X-ray.

Another diagnostic tool that is being used more frequently in the emergency department is the FAST (Focused Assessment Sonography in Trauma) exam.

It takes only a few seconds and can confirm the diagnosis of haemopericardium.

As soon as tamponade is either strongly suspected or diagnosed, a pericardiocentesis should be performed by inserting a large-bore angiocatheter connected to a syringe beneath the xyphoid process at a 45° angle angulated towards the left shoulder.

Negative pressure should be applied to the syringe as the needle is inserted, all the while monitoring for signs of ventricular arrhythmias.

If ventricular arrhythmias are seen on the monitor, the needle should be slightly retracted.

Usually, blood that is withdrawn from the cardiac chambers will clot whereas blood from the pericardium will not.

As little as 15 ml of blood withdrawn from the pericardium can relieve the tamponade.

The needle is then removed, and the catheter is left in place, attached to the syringe.

Immediate surgical consultation is warranted.

Another technique for evacuation of pericardial tamponade is creation of a pericardial window, a procedure that is performed by experienced surgeons in the operating room.

SPSE 1

How well did you know this?
1
Not at all
2
3
4
5
Perfectly
104
Q

Which diagnostic study is now considered the study of choice for aortic dissection?

A chest CT
B chest CT angiography
C aortography
D transthoracic echocardiography
E transoesophageal echocardiography

A

B

Traumatic aortic dissection is rare in children but can occur with a rapid acceleration–deceleration mechanism such as in a high-speed motor vehicle collision or in a fall from great height.

The tear in the intima is usually distal to the left subclavian artery at the level of the ligamentum arteriosum.

Although aortography has been the gold standard for many years, the diagnostic study of choice for blunt aortic injury is now chest CT angiography.

The use of aortography is limited because it is a very invasive study that requires a specialised team and is also associated with complications of its own.

Chest CT angiography has excellent sensitivity and negative predictive value.

Transoesophageal echocardiography is a diagnostic test of lesser value as it can miss injuries of the ascending aorta and its branches.

SPSE 1

How well did you know this?
1
Not at all
2
3
4
5
Perfectly
105
Q

Which of the following is a finding commonly seen with commotio cordis?

A cardiac contusion
B coronary abnormality
C pre-existing arrhythmia
D valvular abnormality
E normal heart

A

E

Commotio cordis is a traumatic condition that results from sudden impact to the chest causing cessation of cardiac activity.

It is most frequently associated with sports such as baseball or hockey.

The sudden impact leads to a recalcitrant arrhythmia, usually ventricular fibrillation, that rapidly evolves into cardiovascular collapse.

This phenomenon is characterised by a sudden arrhythmia in a normal heart with no cardiac contusion or coronary injury and no underlying structural or conduction defect.

It is thought to occur more often in children because of the more compliant chest wall that allows direct transmission of energy to the heart.

Survival from commotio cordis is reported to be around 25%, mainly because the arrhythmias are often resistant to resuscitative efforts and also because the mechanism is often not initially recognised, causing delays in treatment.

Recognition of this phenomenon has led to more frequent use of mid- chest protective gear in baseball and hockey players as well as increased availability of automatic external defibrillators at organised athletic events.

SPSE 1

106
Q

After chest tube placement, a patient has persistent, large air leak noted in the Pleurovac. He also has extensive crepitus in the neck. What is the most likely cause?

A bilateral pneumothorax
B tracheobronchial tree injury
C Pleurovac malfunction
D oesophageal perforation
E diaphragmatic rupture

A

B

Tracheobronchial tree injuries are rare both in children and in adults.

They occur more commonly with penetrating than with blunt chest trauma.

Most injuries resulting from blunt trauma occur in the distal trachea or proximal bronchi.

They should be suspected in the context of bilateral or unilateral pneumothorax with persistent air leak after chest tube placement.

Hoarseness, respiratory distress, and extensive cervical or thoracic subcutaneous emphysema may be seen.

A pneumomediastinum can also be seen on chest X-ray or CT of the chest.

These injuries are highly lethal if not addressed immediately.

These children require intubation assisted with fibre-optic guidance.

Bronchoscopy should be performed to assess the airway when a tracheobronchial injury is suspected.

Occasionally, in a stable patient, a CT of the chest can help delineate the anatomy.

Treatment usually requires operative intervention.

SPSE 1

107
Q

After chest tube placement for a suspected haemothorax in a 10 kg child, 300 mL of blood is returned immediately. What should be the next step?

A insertion of another chest tube on the ipsilateral side

B insertion of a chest tube on the contralateral side

C clamping of the chest tube

D intubation

E prepare for surgical intervention.

A

E

Massive haemothorax should increase suspicion of a serious intrathoracic injury that needs to be addressed surgically.

Some volumetric guidelines for surgical intervention described in the paediatric literature include more than 15 ml/kg of initial drainage or more than 2–3 ml/kg/hr of continuous drainage.

Possible aetiologies for massive haemothorax include lacerations to intercostal vessels, lung parenchyma, heart or great vessels.

These children need to be resuscitated aggressively and brought to the operating room as soon as possible.

Patients that are haemodynamically unstable should undergo thoracotomy rather than thoracoscopy.

The approach chosen depends on the clinical suspicion of the injury.

Most authors would agree that the correct approach in an unstable child would be a left anterolateral thoracotomy.

SPSE 1

108
Q

A child has been on mechanical ventilation in an intensive care unit for 1 week following a multiple trauma. He now has a PaO2 of 65mmHg with a FiO2 of 60%, a pulmonary capillary wedge pressure of 10 mmHg, and bilateral alveolar infiltrates on chest X-ray. What is the diagnosis?

A bilateral pneumonia

B acute pulmonary oedema

C acute lung injury (ALI)

D ARDS

E bilateral pleural effusions

A

D

ARDS can occur in children with multiple trauma with or without associated thoracic trauma.

It is the result of widespread inflammation with endothelial cell damage.

Ventilation/perfusion impairment ensues with secondary hypoxaemia.

By definition, the pulmonary capillary wedge pressure is <18 mmHg, whereas it is elevated in pulmonary oedema.

The difference between ARDS and ALI is the level of hypoxaemia: PaO2 /FiO2 is <300 mmHg in ALI and is <200 mmHg in ARDS.

Both can present with bilateral patchy alveolar infiltrates.

The treatment of ARDS relies on supportive measures such as high-frequency oscillatory ventilation.

Although ARDS occurs infrequently in children, mortality is very high, reflecting the severity of the trauma.

SPSE 1

109
Q

All the following can be appropriate treatment of an empyema secondary to a haemothorax except:

A IV antibiotics only
B chest tube placement if not done initially
C fibrinolytic therapy through a pre-existing chest tube
D decortication with video-assisted thoracoscopy (VATS)
E decortication with thoracotomy.

A

A

Empyema is a collection of pus in the pleural space.

Undrained haemothorax predisposes to empyema. Therefore, in the setting of trauma, even small haemothoraces require drainage to prevent the formation of empyema.

Although empyemas may vary from simple to complex, it may be adequate to start with less invasive measures such as placement of a chest tube to drain the empyema.

The child can then be reassessed and followed with serial chest X-rays.

Fibrinolytic therapy instilled through the chest tube has also been described as an effective treatment for empyema.

If fever persists and the lung doesn’t re-expand adequately despite chest tube drainage and fibrinolytic therapy, surgery must be considered.

Surgery is also the appropriate treatment if the chest CT or ultrasound shows multiple loculations unlikely to be adequately drained with a chest tube.

The procedure can be done either by VATS or mini-thoracotomy, although most paediatric surgeons would prefer VATS as it is a less invasive procedure.

Removal of pus and fibrinous exudate that prevents the lung from re-expanding is achieved during the procedure.

A chest tube is left in place after the procedure and antibiotics are continued.

The use of antibiotics alone is not an effective treatment for a proven post-traumatic empyema.

SPSE 1

110
Q

After a child sustained a penetrating injury to the chest, unexplained fever is noted the following day. The only findings are fever and pneumomediastinum. What diagnosis should be suspected?

A necrotising pneumonia
B empyema
C lung abscess
D oesophageal perforation
E tracheobronchial injury

A

D

Oesophageal injury is rare after blunt trauma since the oesophagus is a wellprotected, posteriorly located intrathoracic organ.

Oesophageal injury is more likely after penetrating chest trauma.

It is a type of injury that can be missed initially if not thoroughly searched for.

Clinical findings suggestive of oesophageal injury include unexplained fever, pleural effusion, empyema or pneumomediastinum.

When suspected, an oesophageal injury should be evaluated with a contrast study and oesophagoscopy.

The combination of a contrast study and endoscopy increases the likelihood of identifying an injury.

The treatment will depend on the extent of the injury as well as the time since injury and may involve primary closure, debridement and drainage, or even diversion.

SPSE 1

111
Q

What would be the appropriate management in the trauma bay of a child with a suspicion of thoracic spine fracture after a motor vehicle collision?

A Leave the patient on the backboard.

B Maintain spine precautions, log-roll patient off the backboard, and keep the cervical collar in place until imaging is completed.

C Maintain spine precautions, log-roll patient off the backboard, and remove the cervical collar since there is no suspicion of cervical spine fracture.

D No spine precautions needed for the thoracic vertebrae fracture but the cervical collar needs to be kept in place until imaging is completed.

E No spine precautions are needed.

A

B

Thoracic spine injuries are rare compared with cervical spine injuries which account for 60%–80% of spine injuries.

The primary mechanisms of injury for thoracic and thoracolumbar vertebral fractures are motor vehicle collisions and falls from great heights.

Spine precautions, both for the cervical and thoracolumbar spine, need to be maintained until appropriate imaging can be done.

This includes a cervical collar for potential cervical spine injury and keeping the patient in a supine position with log-roll only for potential thoracolumbar spine injury.

Although total spine precautions must be maintained, the patient can be log-rolled off the backboard since it can rapidly lead to pressure-related complications.

Plain X-rays should be done first, but if a plausible mechanism of injury and midline focal tenderness are present, further imaging with CT scan should be done.

SPSE 1

112
Q

Which of the following is the most appropriate diagnostic study to screen for cardiac contusion after blunt chest trauma in a haemodynamically stable child?

A electrocardiogram (ECG)
B CK-MB
C troponins
D echocardiography
E all of the above

A

A

Cardiac contusion occurs as a result of blunt trauma to the chest (i.e. impact against the steering wheel in motor vehicle collisions, direct blows to the chest or crush injuries).

Initial screening includes an ECG.

The patient can then be placed on a cardiac monitor for 8 hours.

There is no need to routinely obtain labs or imaging tests.

If cardiac contusion is suspected because of arrhythmia, a FAST should be performed immediately to exclude a haemopericardium.

A formal cardiology consultation should be requested and a transthoracic echocardiography performed.

If only cardiac contusion is found, treatment is supportive and requires cardiac monitoring for approximately 24 hours if the patient remains stable.

Cardiac biomarkers (CK-mB and troponins) have no proven prognostic value, and their use is controversial.

SPSE 1

113
Q

A child sustained a penetrating injury with a knife in the seventh intercostal space on the left side of the chest along the anterior axillary line. What would be the correct management to exclude a diaphragmatic injury in a stable patient?

A CT of the chest
B CT of the chest and abdomen
C CT of the abdomen
D diagnostic laparoscopy
E none of the above

A

D

Thoracoabdominal penetrating injuries often result in diaphragmatic injuries which are difficult to diagnose.

Patients may be asymptomatic in the absence of herniation.

With a penetrating injury to the chest beneath the nipple line, diaphragmatic and intra-abdominal injury must be suspected.

CT scan of the chest or abdomen is not sensitive for detection of diaphragmatic injuries in the absence of herniation.

Similarly, FAST or diagnostic peritoneal lavage are not sensitive for detection of diaphragmatic injuries.

Diagnostic laparoscopy is the most sensitive study to exclude diaphragmatic and intra-abdominal injury as a result of penetrating thoracoabdominal trauma.

SPSE 1

114
Q

Which intra-abdominal organ is the most frequently injured in children due to blunt abdominal trauma?

A liver
B spleen
C kidney
D pancreas
E duodenum

A

B

Among the solid organs, the spleen is the most frequently injured in blunt abdominal trauma, followed by the liver and kidney.

The pancreas is injured in approximately 3% of the cases of paediatric blunt abdominal trauma.

Children are more susceptible to solid organ injury after sustaining blunt abdominal trauma because of distinct anatomical differences compared with adults.

The abdominal wall is thinner with less fat and muscular padding, resulting in more intense energy transmission and closer proximity to vital organs.

The ratio between the size of the intra-abdominal organs and the torso is higher than in adults, also making them more vulnerable to injury.

Because of smaller body mass, children are more likely to incur multiple organ injuries.

Children also have a higher incidence of upper abdominal injuries (i.e. liver and spleen) because of more flexible ribs.

Hollow viscus injury following blunt abdominal trauma is rare, occurring in only 1%–15% of patients.

The most common injury is intestinal perforation, with jejunum and ileum being most frequently injured, followed by duodenum.

SPSE 1

115
Q

What is the success rate of non-operative management of spleen and liver injury in paediatric blunt abdominal trauma?

A 20%
B 30%
C 50%
D 70%
E 90%

A

E

The success rate for non-operative management of spleen and liver injuries in paediatric blunt abdominal trauma has been reported to be as high as 90%–95%.

This differs significantly from the success rate reported in the adult literature, which is approximately 75% for blunt splenic injury.

This needs to be taken into account when caring for the injured child.

Children treated for blunt splenic trauma at adult trauma centres are nearly twice as likely to undergo splenectomy.

Non-operative management of solid organ injury in children is predicated on physiologic rather that radiological parameters.

The only absolute contraindication to non-operative management of the child with solid organ injury is haemodynamic instability unresponsive to fluid and blood resuscitation (usually >40 ml/kg of packed red blood cells) or the presence of associated injuries requiring laparotomy.

Although CT grade of injury and degree of haemoperitoneum correlate with severity of injury, they do not necessarily correlate with the need for immediate operative intervention or failure of non-operative management.

The child should be monitored closely in a setting that allows for immediate intervention should non-operative management fail.

SPSE 1

116
Q

A child sustained a blunt abdominal trauma and CT scan of the abdomen demonstrates a 4 cm parenchymal laceration with devascularisation of less than 25% of the spleen. No other injuries are identified. The child is haemodynamically stable and haemoglobin is 13. What is the most appropriate management?

A exploratory laparotomy and splenectomy

B exploratory laparotomy and partial splenectomy

C exploratory laparoscopy and splenectomy

D admission to intensive care unit (ICU) for monitoring

E admission to the floor for monitoring

A

E

In 2000, evidence-based guidelines were proposed by the American Paediatric Surgical Association Trauma Committee for the non-operative management of blunt splenic or liver injuries.

These recommendations were made based on a thorough review of the medical literature as well the retrospective analysis of more than 800 children who sustained blunt abdominal trauma and were treated conservatively.

Guidelines were proposed to address need for ICU monitoring, length of hospital stay, need for pre-discharge or post-discharge imaging, and appropriate activity restrictions.

The level of activity implied in these guidelines is normal activity for the age of the child. Return to full-contact activities is left to the discretion of the physician. Grade V injuries were excluded from this study, and patients had to meet the criteria for non-operative management.

Table 10.1 summarises the proposed guidelines.

The patient described is haemodynamically stable with an isolated grade III splenic injury according to the American Association for the Surgery of Trauma organ injury grading scale and thus can be managed non-operatively.

According to the proposed guidelines, he does not require ICU admission and can be safely admitted to the floor for monitoring.

SPSE 1

117
Q

All the following are complications of the non-operative management of blunt splenic trauma except:

A bleeding
B splenic pseudocyst
C splenic pseudoaneurysm
D overwhelming post-splenectomy sepsis (OPSI)
E missed intra-abdominal injury.

A

D

Bleeding is the most common complication of non-operative management of blunt splenic trauma.

The need for an operation in children with grade I–III splenic injuries is less than 3%.

Bleeding is more likely with higher-grade injuries and tends to occur early after injury.

Close monitoring of these children with serial abdominal exams and serial haemoglobin levels identify those patients with early bleeding.

Late bleeding can be because of the formation of a splenic pseudoaneurysm. The rate of formation of pseudoaneurysms and natural history if left untreated is unclear.

Although routine follow-up imaging is not currently recommended for non-operative management of splenic trauma, if a splenic pseudoaneurysm is identified on subsequent imaging because of symptoms, it can usually be successfully embolised.

Splenic pseudocysts are asymptomatic most of the time, and for this reason, their incidence is unknown. However, symptoms including pain and vomiting may occur with enlargement of the cyst and compression of the stomach.

If symptomatic, treatment may include simple fenestration of the cyst or partial splenectomy.

Splenic abscess is a rare complication of splenic trauma and is generally managed with antibiotics and percutaneous drainage.

Pleuropulmonary complications, including development of pleural effusions, atelectasis, pneumonia and empyema may occur in up to 50% of patients with splenic trauma.

The risk of missing an intra-abdominal injury is fairly low but does exist. For this reason, non-operative management is contraindicated in children with diffuse peritonitis.

Accordingly, a child who develops increasing abdominal tenderness with peritonitis during the observation period should undergo exploration. OPSI is a complication that occurs after splenectomy.

SPSE 1

118
Q

Following splenectomy in children, vaccines need to be administered to prevent OPSI by which of the following organisms?

A Streptococcus pneumoniae
B Neisseria meningitidis
C Haemophilus influenzae
D all of the above
E none of the above

A

D

The risk of OPSI is higher for children than adults.

The incidence is 0.23%–0.42% per year with a lifetime risk of 5% in children, compared with a 2% lifetime risk in adults.

Although the risk is low, mortality may exceed 50% when this complication occurs.

Splenectomised children are most susceptible to sepsis due to encapsulated microorganisms, including S. pneumoniae, N. meningitidis and H. influenzae.

The exact timing of vaccinations in this situation is controversial, however, it is generally recommended that children receive vaccines against these pathogens prior to discharge from the hospital to avoid problems with non- compliance.

Another area of controversy is the use of prophylactic antibiotics in asplenic children. It is generally recommended that children under the age of 5 years receive penicillin for prophylaxis.

SPSE 1

119
Q

A child involved in a motor vehicle collision is haemodynamically unstable on presentation to the emergency department and has diffuse peritonitis on abdominal exam. He is rapidly taken to the operating room for an exploratory laparotomy. An isolated, large, actively bleeding right liver laceration is found. The child remains haemodynamically unstable despite aggressive fluid and blood resuscitation. What is the appropriate management at this point?

A right hepatectomy
B hepatorrhaphy
C right hepatic artery ligation
D packing of the liver with laparotomy pads and temporary closure of the abdomen
E none of the above

A

D

The armamentarium for operative management of liver injuries is vast.

Hepatorrhaphy can be safely done for smaller lacerations. Adjuncts to hepatorrhaphy include application of sealants such as Tisseel or BioGlue and packing of the laceration with omentum.

For extensive injuries, bleeding can initially be controlled by packing the involved area with laparotomy pads.

The Pringle manoeuvre is a useful technique to temporarily occlude the hepatic hilum when there is extensive haemorrhage not controlled by other means.

A large haemostat is used to clamp the hepatoduodenal ligament thereby interrupting the flow of blood through the hepatic artery and the portal vein and thus helping to control bleeding from the liver.

Complicated procedures such as hepatic resection are generally not recommended but may be considered in experienced hands if it can be accomplished safely and expeditiously and the patient is haemodynamically stable.

Another option for uncontrollable bleeding is right or left hepatic artery ligation. This technique is fraught with complications and should be considered only when other options fail.

In the context of extensive bleeding and shock, the safest option is to expeditiously pack the abdomen to control the bleeding, perform a temporary abdominal wall closure, and continue aggressive fluid resuscitation and rewarming of the patient in the ICU.

This technique can also be combined with the postoperative angiographic embolisation prior to returning to the operating room for reassessment.

SPSE 1

120
Q

All of the following findings are associated with abdominal compartment syndrome except:

A decreased urine output
B increased respiratory peak pressures
C increased cardiac output
D decreased venous return
E splanchnic hypoperfusion.

A

C

Abdominal compartment syndrome occurs when there is increased abdominal pressure causing decreased perfusion to the body wall and intra-abdominal organs.

It has become an increasingly recognised clinical entity associated with multiple trauma, massive intra-abdominal haemorrrhage, and aggressive fluid and blood resuscitation.

Abdominal compartment syndrome has been described not only in postoperative patients but also in patients managed non-operatively.

Increased intra-abdominal pressure leads to decreased renal and splanchnic perfusion, decreased venous return and cardiac output, and respiratory failure with increased peak respiratory pressures and ventilation/perfusion mismatch.

Increased ventilator pressures, oliguria and a ‘tight’ abdomen in the multiple trauma patient should arouse suspicion for abdominal compartment syndrome.

Once suspected, it must be diagnosed and treated immediately to prevent progression to intestinal ischaemia/infarction, multiple system organ failure and death.

Indirect measurement of intra-abdominal pressure can be accomplished by measuring bladder pressures using a Foley catheter and a pressure transducer.

Decompression of the abdomen should be considered when intraabdominal pressures exceed 15–20 mmHg.

Abdominal compartment syndrome may be prevented by leaving the abdomen open in the severely injured child undergoing laparotomy and avoiding overaggressive fluid resuscitation.

Several devices can be used for temporary closure of the abdomen in children: Bogota bag, vacuum-assisted closure dressing, and Silastic sheeting among others.

SPSE 1

121
Q

All these injuries should be suspected in a child presenting with the seat-belt sign except:

A splenic laceration
B intestinal injury
C mesenteric injury
D thoracic spine fracture
E lumbar spine fracture.

A

A

The seat-belt syndrome is a pattern of injuries attributed to use of the lap seat belt in children involved in motor vehicle collisions.

It consists of the triad of abdominal wall contusion (seat-belt sign), intestinal and/or mesenteric injury and thoracic and/or lumbar spine fracture.

It has been commonly associated with a Chance fracture, which is a fracture that occurs after hyperflexion of the back over a fixed object and most commonly involves T12, L1 and L2.

Intestinal and mesenteric injuries result from deceleration, shearing and compressive forces, and present with intestinal perforation, intestinal bruising, mesenteric haematoma and/or mesenteric laceration with active bleeding.

The presence of a seat-belt sign (abdominal wall contusion caused by a seat belt) should raise suspicion for underlying intestinal injury as well as thoracolumbar spine fractures (i.e. Chance fractures), which may be accompanied by paraplegia.

CT scan may reveal bowel thickening or free fluid without solid organ injury, findings that should heighten suspicion for bowel injury.

A low threshold for surgical exploration should be maintained in this setting. A thorough clinical and radiological evaluation for lumbar or thoracic trauma should also be pursued in these patients.

Although solid organ injury such as splenic laceration can occur with the same mechanism of injury (high-impact motor vehicle collision), it is not classically associated with the seat-belt syndrome.

SPSE 1

122
Q

A child presents to the emergency department after a bicycle crash. His abdominal exam reveals an abrasion with contusion over the epigastrium and he complains of localised epigastric pain. A CT scan was done and does not show any injuries. Which injuries should be included in the differential diagnosis?

A duodenal injury
B pancreatic injury
C small-bowel injury
D all of the above
E none of the above

A

D

This abdominal wall contusion is most likely the result of the bicycle handle bar impaling the upper abdomen as the child falls from the bicycle.

Handlebar injuries are classically associated with duodenal, pancreatic, and small-bowel injuries.

In up to two-thirds of cases, duodenal injury and pancreatic injury coexist.

Duodenal, pancreatic and small-bowel injuries are not infrequently missed on the initial CT scan.

One must maintain a high index of suspicion for these injuries.

Adding oral contrast to the intravenous contrast when performing the CT scan may increase the sensitivity to detect these injuries.

Patients with localised tenderness and a normal CT scan should be monitored with serial exams.

Although clinical examination is usually reliable, repeating a CT scan at 24–48 hours in the setting of persistent symptoms may reveal evolving injuries.

Although amylase levels may initially be normal, they are usually elevated at 24 hours after the injury if a pancreatic injury is present. These children may therefore benefit from serial blood tests.

SPSE 1

123
Q

A 10-year-old child is kicked in the upper abdomen while playing soccer. He presents to the emergency department 4 days after the injury because of persistent pain and vomiting. A CT scan of the abdomen reveals a duodenal haematoma. What is the appropriate management of this patient?

A exploratory laparoscopy and drainage of haematoma

B exploratory laparotomy and drainage of the haematoma

C exploratory laparotomy and duodenal resection

D nasogastric decompression and parenteral or enteral nutrition

E none of the above

A

D

Most duodenal haematomas can be managed non-operativey in the absence of peritoneal signs on physical examination.

Children with duodenal haematomas usually present with gastric distension, vomiting and intolerance to feeds.

Conservative management consists of decompressing the stomach with a nasogastric tube and either total parenteral nutrition or enteral nutrition with nasojejunal feedings.

It may take up to 3–4 weeks for complete resolution of obstructive symptoms. Rarely, patients develop a stricture at the site of the haematoma.

When an intramural duodenal haematoma is found intraoperatively while exploring for another injury, the serosa of the duodenum overlying the haematoma can be incised to evacuate the haematoma. This should be done carefully so as not to disrupt the submucosa.

Alternatively, the haematoma can also be left in place as long as it is not a full-thickness haematoma.

It can be beneficial to pass a nasojejunal tube under direct vision to provide enteral feedings in the postoperative period.

A nasogastric tube should also be left in place.

SPSE 1

124
Q

What is the appropriate management for a child brought to the operating room 24 hours after an injury who is found to have a complex, stellate 4 cm perforation on the lateral aspect of the second part of the duodenum with massive intra-abdominal contamination?

A primary repair

B primary repair with drain left near the repair

C tube duodenostomy, pyloric exclusion, feeding jejunostomy, nasogastric tube

D Whipple’s procedure (pancreaticoduodenectomy)

E none of the above

A

C

Management of duodenal injury varies depending on type of injury (haematoma vs. laceration), location of injury, and complexity of injury.

Most duodenal haematomas can be managed non-operatively with nasogastric tube decompression and total parenteral nutrition or nasojejunal feedings.

Perforations should be managed operatively. If the perforation is small, primary closure should be performed.

Because of the often tenuous status of duodenal repairs, a closed suction drain is often left near the repair.

However, if the perforation is more complex such that primary closure is not feasible, or there is significant intra-abdominal contamination (in cases of delayed exploration), other options must be considered.

One option is primary repair, if this can be done without significantly narrowing the duodenal lumen, with pyloric exclusion to protect the repair.

If the perforation cannot be closed primarily, a tube duodenostomy may be left in place, and a pyloric exclusion and/or gastrojejunostomy performed. For both procedures, a drain should be left near the duodenum, a nasogastric tube used to decompress the stomach, and a feeding jejunostomy created to provide postoperative enteral nutritional support.

A Whipple’s procedure is not generally recommended in the acute trauma setting unless there is significant vascular injury in the pancreaticoduodenal area that precludes repair.

SPSE 1

125
Q

What is the appropriate management of a laceration involving 50% of the circumference of the descending colon in an otherwise stable child?

A primary repair
B primary repair with proximal loop colostomy
C primary repair with proximal loop ileostomy
D exteriorisation of proximal and distal ends of the colon as stomas
E Hartmann’s procedure (exteriorisation of proximal end of the colon as a stoma while leaving the distal end as a stump in the abdomen)

A

A

Both small bowel and colon injuries are managed by debridement and primary repair for smaller, less complex lacerations vs. resection and primary anastomosis for larger, more complex lacerations or complete transections.

Factors that may alter this approach include haemodynamic instability, extensive gross contamination, severe burden of injuries, or extensive blood loss with hypothermia and coagulopathy.

In such cases, a damage-control procedure is a wiser option.

The stomach is decompressed with a nasogastric tube, all injured bowel is stapled off to prevent further contamination, a temporary abdominal wall closure is performed, and the patient is taken to the ICU for further resuscitation and warming, with re-exploration in 24–48 hours once the patient is stable.

At the time of re-exploration, a decision can be made regarding primary repair, resection and reanastomosis, or resection and ostomy depending upon the clinical status of the child and the degree of contamination.

Rectal injuries are repaired primarily with or without a colostomy depending on the extent and location (intraperitoneal vs. extraperitoneal) of the injury.

SPSE 1

126
Q

What is the appropriate diagnostic study in a child who was involved in a motor vehicle collision and presents to the emergency department with a seat-belt sign and localised abdominal pain?

A FAST (Focused Assessment with Sonography for Trauma) exam

B diagnostic peritoneal lavage (DPL)

C ultrasound of the abdomen

D CT scan of the abdomen and pelvis

E diagnostic laparoscopy

A

D

CT scan of the abdomen and pelvis with intravenous contrast is the gold standard for evaluation of blunt abdominal trauma in children.

Adding oral contrast may increase the sensitivity for detection of duodenal and proximal small-bowel injuries; however, this benefit should be weighed against the risk of aspiration.

The role of the FAST exam and DPl is limited to children who are too unstable to undergo CT scan.

A FAST exam that demonstrates a large amount of free intraperitoneal fluid in a child who is haemodynamically unstable mandates operative exploration.

DPl can be useful in the case of an equivocal FAST exam in a haemodynamically unstable child.

If the DPl is positive, the child requires exploration, whereas if it is negative, other causes of bleeding should be sought, such as a pelvic fracture.

SPSE 1

127
Q

All the following are criteria for a positive DPL in blunt trauma except:

A aspiration of 1 mL of gross blood

B >100 000 red blood cells per mL

C >500 white blood cells per mL

D aspiration of bile or gastrointestinal contents

E Gram stain positive for bacteria.

A

A

DPL is rarely indicated in children; however, DPL can be helpful in situations where the patient is too unstable to undergo a CT scan of the abdomen and a FAST exam is not available or is equivocal.

It can be performed either percutaneously or using an open technique with a small incision above or below the umbilicus.

A supraumbilical approach is preferred in cases where a pelvic fracture is suspected.

The percutaneous technique is done in a Seldinger fashion, using a needle, a guide wire, a dilator, and a catheter.

For both techniques, the first step is to aspirate. If more than 10 ml of gross blood is returned upon aspiration, the exam is positive and the patient needs to go to the operating room.

The exam is also positive if bile, vegetable fibres, or gastrointestinal contents are aspirated.

If less than 10 ml of gross blood is aspirated, 10 ml/kg of normal saline should be infused in the peritoneal cavity and then aspirated and sent to the lab.

The criteria for a positive DPL are:

> 100 000 RBC/ml,
500 WBC/ml and
positive Gram stain.

DPL is a diagnostic study with excellent sensitivity, but diaphragmatic and retroperitoneal injuries may be missed.

SPSE 1

128
Q

What is the appropriate management in a child who is haemodynamically unstable after 40 mL/kg of packed red blood cells, has a Glasgow Coma Scale score of 3, and a distended abdomen with a positive FAST exam?

A emergency head CT
B emergency abdominal CT
C emergency craniotomy
D emergency laparoscopy
E emergency laparotomy with placement of an intracranial pressure monitor perioperatively

A

E

Whenever an intra-abdominal injury leading to haemorrhagic shock coexists with a traumatic brain injury (TBI), the intra-abdominal bleeding should be addressed first since secondary brain hypoperfusion can have devastating consequences on the outcome of the TBI.

A child that is haemodynamically unstable due to major intra-abdominal haemorrhage and is in a state of shock not responsive to fluid and blood resuscitation is too unstable to undergo CT scanning and needs to be transported immediately to the operating room to undergo exploratory laparotomy.

The cause of the bleeding has to be addressed as quickly as possible, and a damage control procedure may be indicated.

Neurosurgeons should be consulted while the patient is still in the emergency department so that a neurologic evaluation can be completed.

Depending on the clinical context and available resources, further management can vary from perioperative intracranial pressure monitor placement, emergency burr holes, portable head CT in the operating room, or immediate transport to CT scan after the procedure.

SPSE 1

129
Q

Which of the following is the most frequent complication of nonoperative management of blunt liver trauma?

A bile leak
B haemobilia
C abscess
D abdominal compartment syndrome
E necrotising cholecystitis

A

A

The most frequent complication of non- operative management of blunt trauma of the liver, other than bleeding, is a bile leak.

The development of fever, ileus, right upper quadrant pain, and persistently elevated liver function tests after liver trauma should arouse suspicion for liver-related complications.

An ultrasound or CT scan should be performed to evaluate for a possible biloma or abscess.

If a fluid collection is found, it can usually be drained percutaneously under ultrasound guidance. The nature of the fluid retrieved determines the diagnosis, i.e. bile with biliary leak; gross pus with abscess.

Hepatobiliary iminodiacetic acid scan is also a useful adjunct for detection of biliary leaks.

Endoscopic pancreatography (ERCP) may be both diagnostic and therapeutic. If a bile leak is identified by ERCP, a sphincterotomy may be performed to facilitate ampullary drainage, and an endoscopic stent may be placed.

Abscess may occur due to an infected biloma but can also be secondary to infected necrosis of the liver when the liver is devitalised by trauma.

Some patients will ultimately require hepatic resection to remove the necrotic part of the liver.

Gangrenous cholecystitis may occur secondary to injury to the right hepatic artery or cystic artery.

Acalculous cholecystitis may also occur in multiply-injured patients who have a prolonged length of stay in the intensive care unit, although this is not usually attributed to liver trauma.

Haemobilia is acute gastrointestinal bleeding arising from the biliary tract that may occur after major liver injury.

Patients classically present with right upper quadrant pain, jaundice, and acute gastrointestinal bleeding.

Diagnosis is made by either angiography or upper endoscopy.

These complications tend to occur in higher-grade liver injuries, such as grades IV and V.

SPSE 1

130
Q

For evaluation of a stab wound to the anterior abdominal wall of a child who has localised abdominal tenderness but is haemodynamically stable, all of the following are useful except:

A local wound exploration
B DPL
C diagnostic laparoscopy
D observation and serial physical exams
E angiography

A

E

There are several ways to approach a stab wound to the anterior abdomen.

Emergency exploration is mandated in children with diffuse peritonitis who are haemodynamically unstable.

In those who are stable with only localised pain, the stab wound may be locally explored to exclude peritoneal violation. If there is no peritoneal violation on local exploration, an intra-abdominal injury is excluded.

In patients with violation of the peritoneum, further evaluation may include DPl, CT scan or diagnostic laparoscopy. observation and serial abdominal examination alone is also an option for the haemodynamically stable patient with minimal abdominal tenderness.

The overall accuracy rate for serial examination for detection of intra-abdominal injury approaches 94% but is labour intensive and requires a thorough, experienced examiner.

Angiography is not typically utilised in the assessment of anterior abdominal wall stab wounds.

SPSE 1

131
Q

Which genitourinary organ is the most frequently injured?

A kidney
B ureter
C bladder
D urethra
E testis

A

A

The kidney is the genitourinary organ most frequently involved in both blunt and penetrating trauma, accounting for more than 50% of the genitourinary traumas.

It is thought that the kidney is involved in 10%–20% of all the paediatric blunt abdominal traumas.

More than 90% of all kidney injuries are secondary to blunt trauma.

Blunt renal injuries are associated with other intra-abdominal injuries in 42%–74% of patients.

When isolated kidney injuries occur, they are usually minor injuries.

The bladder is the second most commonly injured genitourinary organ, and injury occurs as a result of blunt abdominal trauma in up to 80% of patients.

Bladder injuries are associated with pelvic fractures in 75%–95% of blunt abdominal trauma.

Ureteral injuries are rare and occur in less than 1% of blunt abdominal trauma and 4% of penetrating abdominal trauma.

Urethral injuries are different in children than in adults because of the fact that the prostate is less developed and does not protect the posterior urethra in children.

Posterior urethral trauma is associated with pelvic fractures whereas anterior urethral trauma is associated with straddle injuries.

Testicular injuries are rare in children although scrotal haematoma is more frequent.

SPSE 1

132
Q

Children are more vulnerable to renal trauma for all of the following reasons except:

A kidneys are proportionally smaller in children

B kidneys are positioned lower in the abdomen in children

C Gerota’s fascia is less developed in children

D children have weaker abdominal wall musculature

E children can have persistent fetal lobulations

A

A

There are many anatomical differences between children and adults that render the kidney more susceptible to injury in children.

Kidneys are proportionally larger in children relative to the size of the body and are positioned lower in the abdomen and therefore not as protected by the rib cage.

Furthermore, since the rib cage is not completely ossified, it does not offer as much protection as in adults.

Because children have weaker abdominal wall musculature, there is less perirenal fat to offer additional protection.

Likewise, Gerota’s fascia and the renal capsule are also not as thick as in adults and make the kidney more vulnerable to lacerations, non-confined bleeding and urinary extravasation.

Since many paediatric kidneys retain their fetal lobulations, lower pole amputation is more likely.

Renal trauma is also more prevalent in children with underlying congenital anomalies.

The reported incidence of pre-existing renal disease in children sustaining renal trauma varies between 1% and 23%. Associated anomalies may include hydronephrosis (commonly secondary to ureteropelvic junction obstruction), abnormal kidney position (horseshoe kidney), abnormal kidney consistency (polycystic kidney disease, urinary reflux), and tumour (Wilms’s tumour).

SPSE 1

133
Q

Which sports-related injury has been most commonly associated with renal trauma in children?

A hockey
B football
C baseball
D martial arts
E bicycle collisions

A

E

Although the most common mechanism of blunt renal trauma in children is motor vehicle collisions, the most common sports-related injuries are from bicycle crashes.

Bicycle crashes are four times as frequent as contact sports in paediatric blunt renal trauma. All-terrain vehicles have also been recognised as an important mechanism of injury for paediatric kidneys.

This has led to some questioning of the advice about safe activities that should be provided to children with kidney abnormalities or after renal trauma.

SPSE 1

134
Q

What should be done when blood is seen at the urethral meatus in a child who sustained blunt trauma?

A Insert a Foley catheter with caution.

B Insert a suprapubic cystostomy tube.

C Insert a nephrostomy tube.

D Perform a retrograde urethrography.

E Perform a CT of the abdomen and pelvis with intravenous (IV) contrast.

A

D

Signs and symptoms suggestive of urethral injury include blood at the meatus, gross haematuria, scrotal haematoma, and inability to void or the sensation of voiding without passing urine.

Another commonly reported sign in the adult literature is the presence of a high-riding prostate, which can be difficult to assess in children.

If any of these signs and symptoms are present, urethral trauma should be suspected and investigated.

Insertion of a urinary catheter should not be attempted because of the risk of creating a false passage with the catheter and aggravating the injury.

Insertion of a suprapubic cystostomy or nephrostomy tube is not recommended in the acute setting prior to any other studies.

The imaging modality of choice is retrograde urethrography. If a urethral injury is confirmed, a urologist should be consulted to insert a Foley catheter.

A cystogram is then completed to exclude an associated bladder injury.

Although most patients with blunt abdominal trauma will require a CT scan to exclude intra-abdominal injury, CT is not a very sensitive study for detection of urethral injuries.

SPSE 1

135
Q

What is the diagnostic study of choice for investigation of haematuria in children?

A intravenous pyelogram (IVP)
B cystogram
C ultrasound
D CT of the abdomen and pelvis with IV contrast
E CT of the abdomen and pelvis without IV contrast

A

D

The management of haematuria in children is an area of controversy.

All agree that children with gross haematuria require further investigation.

However, the degree of microscopic haematuria that mandates further investigation has not been clearly defined in the literature.

Some recommend further investigation for microscopic haematuria >5 red blood cells (RBCs) per high-power field (HPF), while others recommend further investigation for >20 RBCs/HPF, >50 RBCs/ HPF, or >100 RBCs/HPF.

In the presence of any signs of shock, any degree of microscopic haematuria should be investigated. One of the reasons why some experts recommend performing diagnostic studies for any level of haematuria in children is the fact that it may be associated with an underlying renal abnormality such as ureteropelvic junction obstruction or Wilms’s tumour.

Historically, IVP has been the diagnostic study of choice. This has now been replaced by CT of the abdomen and pelvis with IV contrast.

An early-phase contrast bolus will detect arterial injuries to kidneys, a delayed phase (>80 seconds) will detect venous and renal parenchymal injuries, while an excretory phase (2–10 minutes) will detect injuries to ureters and bladder.

CT is not sensitive for diagnosis of bladder injury if the bladder is not fully distended.

If a bladder injury is strongly suspected, a CT or conventional cystogram should be performed.

Ultrasound has decreased sensitivity compared with CT, especially for renal parenchymal injuries.

SPSE 1

136
Q

Which one of the following is the appropriate initial management of urinary extravasation?

A observation
B ureteral stent
C percutaneous drainage
D all of the above
E none of the above

A

D

Urinary extravasation can occur with grade IV and V renal injuries according to the organ injury grading scale of the American Association for the Surgery of Trauma (AAST).

In the past, urinary extravasation was an indication for surgical exploration whereas now most of these injuries are managed non-operatively.

Most urinomas will resolve spontaneously without any intervention.

An enlarging or infected urinoma can manifest itself with an expanding flank mass, pain, fever, or ileus.

If an enlarging urinoma is suspected, an ultrasound or CT of the abdomen and pelvis with IV contrast should be performed. This can still be managed nonoperatively with percutaneous drainage and/or ureteral stenting by cystoscopy.

The use of routine antibiotics for urinary extravasation is controversial with some authors recommending its use only when ureteral stents are used.

If an infected urinoma is suspected, it should be treated with antibiotics and percutaneous drainage, with or without ureteral stents.

SPSE 1

137
Q

What is the most important criterion to consider when deciding on operative vs. non-operative management of renal trauma?

A large perirenal haematoma
B extensive urinary extravasation
C parenchymal laceration extending through the renal cortex, medulla and collecting system
D renal artery thrombosis
E haemodynamic instability

A

The only absolute indication for operative intervention for renal trauma is haemodynamic instability unresponsive to fluid and/or blood resuscitation.

Even complex grade IV and V renal injuries may be successfully managed non-operatively as long as the patient remains haemodynamically stable.

Ongoing bleeding and increasing transfusion requirements are indications for operative management.

Radiographic signs on CT indicative of ongoing renal bleeding include an expanding or uncontained retroperitoneal haematoma or complete avulsion of the main renal artery or vein with extravasation.

Whereas some of these radiographic findings may be managed non-operatively with angiography and selective embolisation in the stable paediatric patient, surgery is mandated in the unstable paediatric patient.

During surgical exploration, if the kidney appears salvageable, a conservative approach should be attempted with renorraphy or partial nephrectomy.

If the kidney appears unsalvageable or the patient is too unstable to undergo conservative management, a nephrectomy should be performed.

SPSE 1

138
Q

What is the most common complication after non-operative management of renal trauma?

A bleeding
B urinoma
C perinephric abscess
D hydronephrosis
E hypertension

A

B

The most common complication of non-operative management of renal injuries is development of urinoma secondary to urine extravasation.

It has been reported that approximately two-thirds of urinomas in children will resolve spontaneously.

Accordingly, it is appropriate to observe a small non-infected urinoma.

Percutaneous drainage and/or ureteral stents are indicated for enlarging and/or infected urinomas.

Delayed bleeding is rare in renal trauma.

Perinephric abscesses can be associated with infected urinomas or perinephric haematomas.

Most can be treated by CT-guided percutaneous drainage and antibiotics while some loculated abscesses will eventually require operative drainage.

Hypertension is thought to occur in less than 5% of the renal traumas managed non-operatively.

Blood pressure should be monitored annually after renal trauma. late complications of renal trauma are rare and include hydronephrosis, arteriovenous fistula, arterial pseudoaneurysm, pyelonephritis and calculus formation.

SPSE 1

139
Q

What is the appropriate management of a renovascular injury diagnosed in a stable patient who presents to the emergency room 2 hours after a blunt abdominal trauma?

A observation

B surgical exploration with attempted revascularisation if the kidney appears salvageable

C surgical exploration and nephrectomy

D angiography and endovascular stent

E none of the above

A

B

Renovascular injuries are rare in children.

The most frequent mechanism of renovascular injury in blunt trauma is rapid deceleration that results in stretching of the renal vasculature with disruption of the arterial intima and subsequent arterial thrombosis.

The proposed treatment algorithm is based on the duration of warm ischaemia. The time of warm ischaemia for which surgical revascularisation is more likely to be successful in blunt renal trauma is less than 4–6 hours.

The chances of successful revascularisation are greater with penetrating trauma than with blunt trauma.

In blunt trauma, renovascular injuries are associated with severe disruption of the renal parenchyma that can make the kidney unsalvageable.

If there is significant parenchymal disruption, nephrectomy would be more appropriate than attempts at revascularisation.

When the diagnosis is made more than 4–6 hours after injury, observation may be an appropriate treatment if the patient is haemodynamically stable.

Others recommend angiography and endovascular stenting in this setting.

Revascularisation should always be attempted in patients with bilateral kidney injuries or a solitary kidney.

SPSE 1

140
Q

Which one of these is the most appropriate for the management of a distal ureteral injury?

A observation
B ureteral stent
C psoas hitch and/or Boari flap
D transureteroureterostomy
E ureteral ligation

A

C

The management of ureteral trauma depends on the location of injury and the haemodynamic stability of the patient.

In a haemodynamically unstable patient, it can be appropriate to ligate the ureter, insert a tube nephrostomy, and plan a delayed intervention.

All devitalised tissue must be debrided and this sometimes results in a length deficit in the ureter.

In distal ureteral injury, primary repair can usually be achieved using a psoas hitch and/or a Boari flap.

The psoas hitch consists of fixing the bladder on the psoas muscle to allow a tension-free anastomosis.

The Boari flap uses a tubularised flap of the bladder to increase the length of the distal ureter.

These primary repairs are often done over ureteral stents.

Another option to increase ureteral length is to mobilise the kidney to add up to 3–5 cm of length.

For more proximal injuries, a spatulated end-to-end anastomosis can be performed or a transureteroureterostomy if a large deficit is present.

Other more complicated options include renal auto-transplantation and ileal interposition.

SPSE 1

141
Q

What is the most appropriate management of extraperitoneal bladder rupture?

A observation
B Foley catheter insertion
C suprapubic cystostomy
D nephrostomy tube
E operative repair

A

B

The most appropriate management of extraperitoneal rupture noted on cystogram is bladder drainage with a Foley catheter.

Surgical repair of these injuries entails manipulation of an extraperitoneal haematoma which increases the risk of bleeding as well as converting a closed pelvic fracture into an open fracture thereby increasing the risk of infection.

According to the literature, approximately 90% of bladder injuries will heal in 10 days and the remainder within 3 weeks with only transurethral catheter drainage.

The literature doesn’t support the routine use of a contrast study before removing the urinary catheter, although it is commonly performed.

SPSE 1

142
Q

What is the most appropriate management of intraperitoneal bladder rupture?

A observation
B Foley catheter
C suprapubic cystostomy
D nephrostomy tube
E operative repair

A

E

Intraperitoneal bladder rupture requires surgical exploration.

Transurethral drainage of the bladder alone is not sufficient since protracted extravasation of urine into the peritoneal cavity can lead to serious metabolic and septic complications.

Furthermore, many of these injuries are associated with other intra-abdominal injuries that require exploration.

The bladder is more likely to be injured if it is distended at the time of injury.

The most common area of injury is at the dome of the bladder.

At the time of repair, the ureteral orifices inside the bladder must be visualised through the defect to ensure their patency.

The defect can then be repaired with absorbable sutures in two layers.

A closed-suction drain can be left in place over the repair.

A Foley catheter is generally left in place for 5–10 days.

Some authors recommend performing a contrast study prior to removal of the closed-suction drain and Foley catheter. Antibiotics are not required.

SPSE 1

143
Q

What is the most appropriate management of partial disruption of the posterior urethra following blunt trauma?

A observation
B Foley catheter
C suprapubic cystotomy
D nephrostomy tube
E operative repair

A

B

Urethral injuries are classified into five categories according to the AAST: contusion,
stretching,
partial disruption,
complete disruption of less than 2cm
and more than 2 cm.

Urethral injuries are most commonly due to pelvic fractures.

The posterior urethra is more prone to injuries in children as it is not protected by the prostate.

Urethral injuries are diagnosed by retrograde urethrography.

Low-grade injuries can be treated with observation alone.

If the child is unable to void, a urinary catheter can be inserted carefully by a urologist.

Most children with partial disruption of the urethra can be managed with a urinary catheter only.

Higher grade injuries can be managed with early surgical repair or delayed surgical repair after insertion of a suprapubic cystostomy tube.

The advantage of delayed repair is avoiding entering a pelvic haematoma with increased risk of bleeding.

SPSE 1

144
Q

Peritoneal lavage is considered positive if lavage effluent contains following except:

A. Blood.

B. Bacteria/faeces.

C. Bile.

D. Amylase.

E. Lipase.

A

E Presence of lipase is not a criteria for labelling positive peritoneal lavage.

MCQs/Syed

145
Q

In the management of penetrating abdominal trauma (stab wound), which one is not correct.

A. Stab wound does not need local wound exploration.

B. Indication of laparotomy includes unstable patient, unexplained blood loss and signs of peritonitis.

C. Positive peritoneal lavage needs laparotomy.

D. Negative peritoneal lavage with signs of peritonitis needs laparotomy.

E. If intact posterior fascia, patient can be discharged.

A

A Stab wound needs local wound exploration.

MCQs/Syed

146
Q

After four blood transfusions, the following should all be administered except:

A. Fresh frozen plasma.

B. Platelets.

C. Magnesium.

D. Calcium.

E. Sodium bicarbonate.

A

C There is no indication of magnesium administration after four blood transfusions until it is low

MCQ/Syed

147
Q

In the management of diaphragmatic injury due to blunt abdominal trauma, which of the following is false?

A. Transthoracic approach is preferable in acute cases.

B. Two layered closure is preferable.

C. Non-absorbable suture is preferable.

D. In stable patient GIT series and ultrasound is helpful.

E. X-rays show shifting of mediastinum to opposite side.

A

A Transabdominal approach is usually selected in acute cases to permit assessment of possible intraabdominal injury. However, in late cases, when adhesion may be present, a thoracic incision is preferable.

MCQ/Syed

148
Q

Regarding gastric perforation in trauma, all are correct except:

A. It is more common in children than in adults.

B. Peritoneal lavage is indicated in comatose patient.

C. Rupture usually occurs in on the greater curvature.

D. Defect is best closed in two layers.

E. Percutaneous gastrostomy is considered if extensive contamination has occurred or other injury is present.

A

E Stamm gastrostomy is constructed if extensive contamination has occurred or other injuries are present.

MCQ/Syed

149
Q

In traumatic intestinal perforation, all of the following are false except:

A. Mobile portion of intestine is more prone to perforation.

B. Perforation mostly occurs on mesenteric border.

C. CT scan is non-diagnostic.

D. Best repair is by simple closure.

E. Physiological stability may necessitate exteriorization.

A

D

Fixed portion of intestine is more prone to perforation.

Perforation mostly occurs on antimesenteric border.

CT scan shows peritoneal fluid in the absence of solid organ injury and intestinal wall enhancement.

Best repair is by simple closure or segmental resection with end-to-end anastomosis if the injury is extensive.

In rare situations, physiological instability may necessitate exteriorization.

MCQ/Syed

150
Q

Regarding splenic trauma, which of the following is false?

A. Unique radial segmental blood supply permits resection and repair of both large and small portion of spleen.

B. Plain radiograph shows medial gastric displacement.

C. Decision of laparotomy based on extent of injury as shown on CT.

D. There are many techniques of splenorrhaphy.

E. During laparotomy, unreasonable prolongation of surgery in patient with multiple trauma is one of the indications of splenectomy.

A

C

Decision of laparotomy based on physical condition, not the extent of injury as shown on CT.

Different splenorrhaphy techniques include:

  1. Vertical mattress sutures
  2. Horizontal mattress sutures using Teflon pledgets to prevent tearing
  3. Figure of eight sutures
  4. Simple interrupted sutures with omental incorporation
  5. Long-interrupted sutures
  6. Chromic ladder sling to appropriate parenchyma, without insertion of needle

MCQ/Syed

151
Q

Regarding post splenectomy treatment, all of the following are correct except:

A. Vaccination against H-Influenza.

B. Vaccination against meningococcus.

C. Vaccination against streptococcus.

D. Prophylactic antibiotic with penicillin.

E. Medical Alert tag.

A

C

Third organism against which vaccination required is with pneumococcus not streptococcus.

Syed/MCQ

152
Q

Regarding pancreatic and duodenal injury, all of the following are false except:

A. Pancreas and duodenum are retroperitoneal, so fully protected.

B. Blunt trauma is more common cause than penetrating trauma.

C. X-rays show air around left kidney.

D. Serum amylase and lipase levels are low.

E. Positive peritoneal lavage for bile and amylase is specific for pancreaticoduodenal injury.

A

B

Blunt trauma is more common cause than penetrating trauma.

Compression occurs against vertebral column. Pancreas and duodenum are retroperitoneal structures so relatively protected.

X-rays shows air around right kidney.

Serum amylase and lipase levels are high.

Positive peritoneal lavage for bile and amylase is nonspecific for pancreaticoduodenal injury, as it is also positive in other intestinal perforations.

Syed/MCQ

153
Q

Regarding hepatic trauma, which of the following is not true?

A. Penetrating trauma is less common than blunt.

B. CT scan helps in diagnosis.

C. Non-operative treatment depends upon physiological stability.

D. Non-expending haematoma resolve spontaneously.

E. All above statements are false.

A

E

All mentioned statements A, B, C and D are true

Syed/MCQ

154
Q

Management of diffuse parenchymal bleeding in hepatic trauma include all except:

A. Cauterization.

B. Manual pressure.

C. Direct ligation of vessel.

D. Placement of radivac drain.

E. General management of patient for haemodynamic stability.

A

D

Penrose drain, not the radivac drain, is used for drainage of bile or blood if intrahepatic ligation is not entirely effective

Syed/MCQ

155
Q

Management of complication, after acute management of liver injury, all are true except:

A. Monthly observation for stable haematoma.

B. Roux-en-Y hepaticojejunostomy for bile duct strictures.

C. External drainage of haemobilia (bleeding in biliary tree), if arteriogram shows aneurysm.

D. External drainage of hepatic abscess.

E. Cholecystectomy for gallbladder injury.

A

C

Embolization is the treatment of haemobilia if arteriogram shows aneurysm.

Syed/MCQ

156
Q

Grade IV hepatic injury is labelled with which of the following features?

A. Laceration 1 cm deep.

B. Bi-lobar tissue maceration and devascularisation.

C. Laceration 3–10 cm deep.

D. Laceration more than 10 cm deep.

E. Laceration 1–3 cm deep.

A

D

Grade IV hepatic injury is labelled when laceration is more than 10 cm deep.

Grade I is less than 1 cm deep,

grade II is 1–3 cm deep,

grade III is 3-10 cm deep,

grade IV more than 10 cm deep laceration and

grade V is bilobar tissue maceration and devascularisation.

Syed/MCQ

*By AAST grading, IV is 25-75% disruption or juxtavenous hepatic injuries

157
Q

Regarding classification of pancreatic injury, which of the following statements is false?

A. Class I injury is contusion and laceration without ductal injury.

B. Class II injury is distal transection or parenchymal injury with probable ductal injury.

C. Proximal transection or parenchymal injury with probable ductal injury.

D. Combined pancreatic and duodenal injury.

E. All of the above are false statements.

A

E

All of the above statements are true.

Syed/MCQ

158
Q

Regarding urethral injury all of the following are false except:

A. Posterior urethral injury is most commonly caused by penetrating trauma.

B. Straddle injury causes anterior urethral trauma.

C. Blood at urethral meatus is less common finding.

D. Urethral contusion is considered as grade II urethral injury.

E. Partial disruption is considered as grade II urethral injury.

A

B

Blunt trauma with disruption of bony pelvis accounts for most of posterior urethral injuries.

Straddle injuries are an important cause of anterior urethral injury. Blood at urethral meatus is a very common finding.

Grades of urethral injury are contusion (grade I),
stretch injury (grade II),
partial disruption (grade III) and
complete disruption (grade IV).

Syed/MCQ

159
Q

Regarding management of urethral injuries, which one is true?

A. Pass foley catheter in complex anterior urethral injuries.

B. Delayed primary realignment causes more bleeding.

C. Delayed primary realignment is contraindicated in boys with disruption of bladder neck.

D. Secondary reconstruction is done after one month.

E. Secondary reconstruction has more incidence of impotency and incontinence.

A

C

Complex urethral injury is managed by suprapubic catheter and later on urethral stricture may develop, requiring urethroplasty.

Delayed primary repair realignment causes less bleeding.

Delayed primary realignment is contraindicated in boys with complete disruption of bladder neck.

Secondary realignment is done after 6–9 months. Secondary reconstruction has fewer incidences of impotence and incontinence.

Syed/MCQ

160
Q

Regarding long-term sequelae of urethral injury, all of the following are false except:

A. Impotence is more common than urethral stricture.

B. Urinary incontinence is more common than impotence.

C. Bladder neck mechanism is not important to avoid retrograde ejaculation.

D. Impotence is caused by injury to nerve to erigentes.

E. Urethral stricture is not the direct consequence of trauma.

A

D

Long-term sequel of urethral injury, urethral stricture is most common (about 50 percent), impotence about 30 percent, and incontinence is about 8 percent.

Bladder neck mechanism is important to avoid retrograde ejaculation.

Impotence is caused by nerve to erigentes.

Urethral stricture by direct consequence of trauma may be from delayed reconstruction of urethral injury.

Syed/MCQ

161
Q

On the organ-injury-severity scale for the kidney, which grade would you label a laceration more than one centimetre parenchymal deep in the renal cortex without collecting system rupture or urinary extravasation?

A. Grade I.

B. Grade II.

C. Grade III.

D. Grade IV.

E. Grade V.

A

C

Grade I: Contusion. Microscopic or gross haematuria; urological studies are normal.

Haematoma—subcapsular, non-expanding without parenchymal laceration.

Grade II: Haematoma. Nonexpanding perirenal haematoma confined to renal retroperitoneum.

Laceration—less than 1 cm parenchymal depth of renal cortex without urinary extravasation.

Grade III: Laceration. More than 1 cm parenchymal depth of renal cortex without collecting system rupture or urinary extravasation.

Grade IV: Laceration. Parenchymal laceration extending through renal cortex, medulla and collecting system.

Vascular—Main renal artery or vein injury without contained haemorrhage. Grade V: Laceration. Completely shattered kidney.

Vascular—avulsion of renal hilum, devascularizing the kidney.

Syed/MCQ

162
Q

In which of the following situation can we diagnose grade IV ureteral injury.

A. Avulsion with more than 2 cm devascularisation.

B. Contusion or haematoma.

C. Less than 50 percent transection.

D. More than 50per cent transection.

E. All of the above are false.

A

E

Grade IV injury is labelled when complete transection with less than 2 cm devascularisation.

Syed/MCQ

163
Q

You can label grade IV urethral injury in case of the following feature.

A. Complete transection with more than 2 cm urethral separation.

B. Partial disruption.

C. Contusion.

D. Complete disruption with less than 2 cm urethral separation.

E. Stretch injury.

A

D

Grade IV urethral injury is complete disruption with less than 2 cm urethral separation.

Grade I is contusion,
grade II is stretch injury,
grade III is partial disruption and
grade V is complete disruption with more than 2 cm urethral separation.

Syed/MCQ

164
Q

Regarding pseudopancreatic cyst, which of the following is true?

A. It is not a consequence of inflammatory process.

B. It is not a consequence of direct trauma to duct system.

C. Manifests within 3–4 weeks of injury.

D. Treatment is medical within first week.

E. Mostly resolves spontaneously and does not require drainage.

A

C

Pseudopancreatic cysts manifest within 3–4 weeks of injury.

Pseudocyst is a consequence of inflammatory process or direct trauma to duct system.

Treatment is medical in first 3–6 weeks.

Most of these persists and not resolved spontaneously and requires drainage.

Syed/MCQs

165
Q

In the management of burns, all of the following are false except:

A. Fifty percent have about a fifty per cent survival rate.

B. Parkland formula is 2 ml/kg/ percent of burned surface are.

C. Fifty per cent of Parkland formula is given in the first 6 hours.

D. In a one-year-old child, head constitutes 19 percent.

E. In a four-year-old child, one lower limb constitutes 18 percent.

A

D

In one-year-old child, head constitutes 19 percent.

By advancement in medical science 50 percent of burn has about 98 percent survival.

Parkland formula is 4 ml/kg/percent of burned surface area in addition to maintenance fluid.

In four-year-old child, one lower limb constitutes 15 percent of body surface area.

Fifty percent of Parkland formula fluid should be given in first 8 hours.

Syed/MCQ

166
Q

In relation to topical antimicrobials used in burns, all are false except:

A. Silver sulphadiazine is a painful application.

B. Mefenide acetate does not penetrate eschar.

C. Silver nitrate penetrates eschar.

D. Povidone iodine is a painful application.

E. Gentamicin is narrow spectrum.

A

D

Povidone iodine is a painful application, so needs good analgesic and monitor of thyroid function.

Silver sulphadiazine is a painless application.

Mefenide acetate penetrates eschar while silver nitrates does not penetrate eschar.

Gentamicin is a broad spectrum antibiotic.

Syed/MCQ

167
Q

Regarding management of infection in burn, all are true except:

A. Purpose of topical antibiotic is to keep bacteria below 100 organism/gram of tissue.

B. If the level of bacteria is 105/gram of tissue, a change in the antibiotic is warranted.

C. Bacteria invasion is suspected when bacterial count is 1015/gram of tissue.

D. Hypothermia is a feature of septicaemia.

E. In burn patients, infection may be in wound, lung and urine or at cannula site.

A

C

Bacterial invasion is suspected when bacterial count is more than 107 organism/gram of tissue.

Features of septicaemia include hypothermia, hyperthermia, paralytic ileus, altered mental status, tachycardia, unexplained acidosis, thrombocytopenia, leucocytosis or hyperglycaemia.

Syed/MCQ

168
Q

In burn patient, which of the following is false?

A. In 40 percent of total burn surface area (TBSA), there is a 50–100 percent increase in BMR (basal metabolic rate).

B. Increase in catecholamine is cause of necrotising enterocolitis.

C. Prolonged growth arrest noted in first year.

D. A decrease in growth hormones and IGF-I strongly suggests that administration of human growth hormones would be advantageous.

E. All of the above are false.

A

E

All statements are true. Increases in catecholamine causes mesenteric vasoconstriction, which leads to decrease in mucosal integrity that in turn causes bacterial translocation resulting in necrotizing enterocolitis.

Syed/MCQ

169
Q

Characteristics of full-thickness skin burn include all except:

A. All viable epithelium are destroyed.

B. Wound is often pink with bullae.

C. There is thrombosed superficial vein.

D. There is no sensation.

E. It requires skin grafting.

A

B

Pink wound with bulla is features of partial thickness burn.

Full-thickness skin burn is usually dry.

Syed/MCQ

170
Q

Acid burn causes all of the following except:

A. Saponification of fat.

B. Electrolyte abnormalities.

C. Renal failure.

D. Intravascular haemolysis

E. RDS (respiratory distress)

A

A

Saponification of fat is a feature of alkali burn.

Syed/MCQ

171
Q

Regarding high-voltage electric current, all of the following are true except:

A. Deep-tissue destruction is produced by heat generated by electric resistance of tissues.

B. Bone injury is common.

C. If facial compartmental pressure is increased, do eschorotomy to prevent limb.

D. Haemochromogen is present in urine.

E. Delayed complication includes neurological disorder, cataracts and paralysis.

A

C

If fascial compartmental pressure is increased, do fasciotomy to save limb.

Syed/MCQ

172
Q

Regarding inhalation injury in burn patient, all of the following are true except:

A. Increase in vasoactive amines causes pulmonary oedema.

B. We label carbon monoxide poisoning if carboxy haemoglobin increases to 3 percent.

C. Twenty percent acetylcysteine nebuliser solution is used.

D. Nebulise with heparin (5000–10,000 U) with 3 ml of normal saline every 4 hours.

E. High flow of humidified oxygen is required

A

B

In carbon monoxide poisoning, carboxyhaemoglobin rises more than 10 percent.

Syed/MCQ

173
Q

Inhalation injury in burn has mortality of approximately.

A. 20 percent.

B. 30 percent.

C. 40 percent.

D. 50 percent.

E. 60 percent.

A

C

40 percent

Syed/MCQ

174
Q

Regarding thoracic injury, the following are immediate life-threatening injuries except:

A. Pulmonary contusion.

B. Airway obstruction.

C. Tension pneumothorax.

D. Massive haemothorax.

E. Cardiac tamponade.

A

A

Pulmonary contusion is potentially life-threatening, not immediately lifethreatening.

Other immediate life-threatening injuries include myocardial contusions, aortic disruption, ruptured diaphragm and tracheobronchial disruption.

Other immediate life-threatening injuries include open pneumothorax and flail chest.

Syed/MCQ

175
Q

Which one is the commonest thoracic injury?

A. Pneumothorax.

B. Lung contusion.

C. Haemothorax.

D. Rib fracture.

E. Ruptured diaphragm.

A

B

Lung contusion is the commonest thoracic injury (50 percent).

Other incidences are:
rib fracture 30 per cent,
pneumothorax 24 percent, and
haemothorax 9 percent.

Rare thoracic injuries include diaphragmatic rupture, aortic, bronchial, and oesophageal injuries, lung laceration, cardiac contusion and cardiac laceration.

Syed/MCQ

176
Q

All of the following are indication for emergency thoracotomy except:

A. Penetrating wound of heart and great vessels.

B. Massive or continuous intrathoracic bleed.

C. Tension pneumothorax.

D. Aortogram showing aorta or major branch injury.

E. Cardiac tamponade.

A

C

Chest intubation is required for tension pneumothorax.

Open pneumothorax with major chest wall defect needs immediate thoracotomy.

Other possible indications of immediate thoracotomy include oesophageal perforation, diaphragmatic rupture, and massive or continuous air leak, indicating injury to major airway.

Syed/MCQ

177
Q

Regarding chest injury, which of the following signs is not correlated with the condition?

A. Noisy breathing results from foreign material blood, vomitus or mucus in the airway.

B. Tracheal deviation implies massive haemothorax.

C. Hoarseness indicates tension pneumothorax.

D. Surgical emphysema suggests tracheal or bronchial laceration or oesophageal perforation.

E. Increased JVP, decreased blood pressure and increased pulses paradoxes ≥10 mm Hg, implies cardiac tamponade.

A

C

Hoarseness indicates direct laryngeal or tracheal injury.

Tracheal deviation implies tension pneumothorax and massive haemothorax.

Syed/MCQ

178
Q

The following is ECMO exclusion criteria except:

A. Grade II intraventricular haemorrhage.

B. Weight less than 2 kg.

C. Severe chromosomal abnormalities.

D. Reversible cardiovascular failure.

E. Cyanotic heart disease.

A

D

Reversible cardiovascular failure is one of the inclusion criteria for ECMO, not the exclusion criteria.

Syed/MCQ

179
Q

Regarding child abuse, which of the following is false?

A. Fractures at various stages of healing is seen in shaken baby syndrome.

B. Abdominal injury is uncommon in abused child.

C. Long bone fracture is more common than skull fracture.

D. Burn in abused children, most victims are less than two years.

E. In the absence of major trauma or pre-existing disease, rib fractures in infants must be considered as evidence of no accidental injury.

A

A

Fractures at various stages of healing are seen in battered baby syndrome.

In shaken baby syndrome, the baby is shaken violently while being held by the torso, which causes acceleration and deceleration injury of the head, neck and trunk.

Acute brain injury occurs, such as subdural and subarachnoid bleeding with cerebral contusion, with no external evidence of head haematoma.

Syed/MCQ

180
Q

Which one is false about birth injuries?

A. In Erb’s palsy, there is lack of shoulder motion.

B. In Klumpke’s palsy, there is lack of wrist movement.

C. Horner’s syndrome is associated with Klumpke’s paralysis.

D. Phrenic nerve injury is associated with Erb’s palsy.

E. Among fractures, humerus fracture is most common in birth injuries.

A

E

Among fractures, clavicular fracture is the most common in birth injuries.

Syed/MCQ

181
Q

All of these are complications of urethral trauma except:

A impotence
B retrograde ejaculation
C incontinence
D stricture
E Peyronie’s disease.

A

E

The long-term sequelae of urethral trauma can be devastating and may include incontinence and urethral strictures, as well as impotence and retrograde ejaculation in boys, and urethrovaginal fistula and vaginal stenosis in girls.

Some of these complications may be a direct consequence of the trauma itself or may be related to surgical attempts at repair.

Delays in diagnosis of urethral injuries in girls occur frequently and can lead to other complications such as sepsis and necrotising fasciitis.

Peyronie’s disease is a curvature in the penis that is secondary to an area of fibrosis in the tunica albuginea. The cause of this disease is uncertain but it is thought to be due to vascular trauma or injury to the penis. It is not secondary to urethral injury.

SPSE 1

182
Q

What is the most common mechanism of injury to the penis in children?

A straddle injuries
B zipper injuries
C blunt injuries
D penetrating injuries
E iatrogenic injuries

A

E

The most common mechanism of injury to the penis is iatrogenic and most often occurs during circumcision.

Complications of circumcision include penile amputation, urethral fistulisation, laceration of the glans, and inaccurate removal of the foreskin that can later lead to phimosis.

Penile injuries from penetrating and blunt trauma are rare in children.

Zipper entrapment of the penis is a well-described mechanism of injury in children. It can often be managed in the emergency room with sedation, but general anaesthesia may be necessary.

SPSE 1

183
Q

What is the most appropriate management of haematocele in children?

A observation
B anti-inflammatory medication
C percutaneous drainage
D surgical exploration
E none of the above

A

D

Ultrasound is very useful for evaluation of scrotal and testicular injuries. It can differentiate between scrotal haematoma, hydroceles, haematoceles, testicular rupture and infarction.

Patients with haematoceles should be considered for surgical exploration to evacuate blood from the tunica vaginalis because this approach reduces morbidity and hastens recovery.

Testicular rupture and infraction are surgical emergencies. Testicular rupture should be managed with debridement and primary closure.

SPSE 1

184
Q

What is the most appropriate management in a 4-year-old girl who was found to have a vaginal laceration after a motor vehicle collision?

A observation
B antibiotics
C repair of laceration in the emergency room
D exam under general anaesthesia
E none of the above

A

D
Vaginal lacerations associated with blunt trauma are particularly worrisome for associated injuries. Attempts at exam and repair should not be attempted in the emergency department.

These patients should be brought to the operating room in order to perform a thorough exam under general anaesthesia.

Speculum examination and vaginoscopy can then be performed, as well as urethroscopy and proctoscopy to exclude associated injuries that can have devastating consequences if diagnosis is delayed.

Depending on the extent, vaginal lacerations can be simply observed or repaired in a single layer with absorbable sutures.

There is no need for antibiotics.

Vaginal lacerations are usually associated with straddle injuries and pelvic fractures when found after blunt trauma.

The possibility of abuse has to be kept in mind if the mechanism of injury is not consistent with the findings.

SPSE 1

185
Q

Children’s fractures differ from adults because the musculoskeletal system in a child differs from that of an adult in all the following ways except:

A presence of growth plate
B higher collagen-to-bone ratio in children
C thicker and stronger periosteal sleeve
D ligaments in children are relatively stronger than bone as compared with adult
E decreased ratio of cartilage to bone.

A

E

The increased ratio of cartilage to bone in children improves resilience but makes evaluation by radiography more difficult, as the size of the articular fragment is often underestimated.

The most obvious difference between the bones of a child and an adult is the presence of a growth plate. The growth plate facilitates remodelling that corrects residual angulation over time but when injured can lead to asymmetrical growth causing angular deformity.

The higher collagen-to-bone ratio reduces the tensile strength of bone thus reducing the tendency of fractures to propagate with less fracture comminution.

Thick periosteum permits less fracture displacement and also when intact, aids in fracture reduction during manipulation. It is also more metabolically active which explains the exuberant callus formation, quicker healing and increased potential for remodelling.

Avulsion injury patterns in childhood are more common because ligaments are relatively stronger than bone.

SPSE 1

186
Q

Fracture remodelling in children depends on:

A the number of years of growth remaining

B the proximity of fracture to a rapidly growing growth plate

C the magnitude of original angular deformity

D the plane of angulation relative to adjacent joints

E all of the above.

A

E all of the above.

Remodelling at the fracture site occurs by reorientation of the growth plate with improvement in overall alignment of the limb as well as bone resorption on the convexity and deposition on the concavity of the fracture.

It may continue for 5–6 years after fracture as long as growth occurs during the process of remodelling.

Fractures in the plane of joint motion and near rapidly growing physes have the greatest capacity to remodel.

For instance remodelling in a fracture of the proximal humerus is far superior than a fracture of the distal humerus, as the proximal humeral growth plate contributes to 80% of the growth of the humerus.

Fractures with the smallest degrees of malunion are more likely to remodel completely.

Remodelling of rotational deformity is less predictable than angular remodelling.

SPSE 1

187
Q

Regarding physeal (growth plate) injury, which of the following is not true?

A Physeal bridging and altered growth occurs in 1% of physeal injuries.

B It occurs more commonly in Salter–Harris type 3, 4 and 5 injuries.

C For best results anatomical reduction is essential in Salter–Harris type 3 and 4 injuries.

D Growth need not be monitored when physeal injury is suspected.

E Imaging physeal bars may be done best with CT or MRI scans.

A

D

Growth needs to be monitored to detect physeal bridges that develop following injury and lead to either angular deformity or limb length discrepancy or both, all of which occur over time.

They happen most commonly following Salter–Harris type 3, 4 and 5 injuries.

These bony bridges are best imaged using coronal and sagittal reformats of 1 mm CT scans or MRI studies.

MRI scans provide more soft tissue information but may be difficult to interpret.

Anatomical reduction of Salter–Harris type 3 and 4 injuries is essential for improving outcomes.

Open reduction and internal fixation that does not traverse the physis is best.

If fixation is necessary across growth plates then a small, smooth K wire must be used.

SPSE 1

188
Q

The principles of fracture fixation in children include all except:

A supplementation with cast using minimal fixation techniques

B external fixation in open fractures

C avoid crossing the physis (growth plate)

D use of ‘flexible fixation’ options in long bones

E rigid intramedullary fixation.

A

E

Cast immobilisation is used to supplement minimal internal fixation in most instances.

Growth plates can be crossed with smooth K wires only.

Flexible intramedullary fixation is ideal for transverse long-bone fractures with lack of comminution or obliquity.

External fixation is used in long-bone fractures with severe soft tissue injuries.

Rigid intramuscular fixation exposes child to risk of physeal damage or avascular necrosis (especially in the proximal femur) and is not used in children with open physes.

SPSE 1

189
Q

Open fractures in children differ from open fractures in adults in all the following ways except:

A soft tissue healing is much more rapid and complete

B devitalised bone that is not contaminated can be left in place and will incorporate

C periosteum will generate new bone when lost

D delayed or non-union is more common in children

E external fixators may be left in place till union.

A

D

Delayed or non-union is uncommon in paediatric fractures because of the overall enhanced healing potential.

The soft tissue healing potential is far superior to adults and limb salvage is usually feasible in the large majority of cases.

External fixation may result in being the only form of treatment utilised to achieve union and bone loss is not as big a problem as it would be in an adult with a similar injury.

The management of open fractures otherwise is similar to an adult in terms of using antibiotic prophylaxis, doing a thorough debridement and fracture stabilisation in order to promote soft tissue healing.

SPSE 1

190
Q

Fractures that have a high complication rate if inadequately treated include all except:

A supracondylar fractures
B lateral condylar fractures
C distal radial buckle fractures
D femoral neck fractures
E distal femoral physeal fractures.

A

C

Buckle fractures of the distal radius are easily treated with immobilisation in a short arm cast or splint for 3 weeks.

Improperly treated supracondylar fractures can develop malunion (cubitus varus) and ischaemic contracture secondary to undiagnosed compartment syndrome.

lateral condylar fractures have the potential for displacement and non-union.

Distal femoral metaphyseal fractures often develop growth arrest and angular deformity.

Femoral neck fractures can develop avascular necrosis of the femoral head.

SPSE 1

191
Q

Which of the following statements is untrue about proximal femoral fractures in the paediatric age group?

A As in the elderly, paediatric proximal femoral fractures occur following low energy trauma.

B On presentation a child has a typically flexed, abducted and externally rotated posture.

C They have a high complication rate due to unique vascular anatomy of the femoral head.

D In cases of intracapsular femoral neck fracture, urgent anatomical reduction is critical to restore blood flow to the femoral head.

E The most frequent and devastating complication of hip fractures is osteonecrosis of the femoral head.

A

A

Hip fractures in children occur nearly always as a result of high- energy trauma, such as motor vehicle accident or a fall from height, placing children with these injuries at risk for multiple injuries.

In the elderly population with osteoporotic bone, most proximal femoral fractures occur following low energy trauma.

Clinical presentation of affected extremity is typically that of flexion, abduction and external rotation at the hip.

The femoral head is essentially an end organ supplied exclusively by branches from the medial circumflex femoral artery, which puts it at risk of osteonecrosis if blood supply is disrupted by the trauma.

urgent, anatomical reduction is critical to restore/maintain blood supply to the femoral head in intracapsular fractures in order to warrant a good outcome.

Extracapsular fractures have a more favourable prognosis because the blood supply is usually preserved but can have complications like varus malunion.

SPSE 1

192
Q

Not true regarding Paediatric femoral shaft fracture:

A that occurs in children younger than 1 year should be viewed with suspicion for non-accidental injury.

B is usually treated with a Pavlik harness in infants and spica casting in toddlers.

C when casted, is usually immobilised at 90 degrees flexion at the hip and knee with approximately 30 degrees of abduction and 20 degrees of external rotation and may be preceded by a few days of traction.

D plating and external fixation are never used as a method of fixation.

E is usually treated by flexible femoral nailing in preschool children.

A

D

External fixation is usually used to manage open femoral fractures, or in patients with head or vascular injuries.

The benefits of external fixation include avoidance of long incisions, exposure of the fracture site and significant blood loss.

Plate fixation, though done less frequently, is used in atypical fracture patterns.

A technique called submuscular bridge plating is being utilised in some centres to treat extensively comminuted, length unstable fractures.

Infants with femoral fractures can be managed in a Pavlik harness and spica casting is the method of choice in preschoolers 1–5 years of age.

Flexible nailing is a form of fixation utilised in average-build school-going children with greatest success in transverse fracture patterns.

Bigger adolescents with femoral fractures are treated with lateral entry femoral nails.

These nails avoid the piriform fossa and reduce the chance of disrupting the blood supply of the femoral head, which can lead to avascular necrosis.

SPSE 1

193
Q

Regarding injuries in and around the paediatric knee, which of the following statements is false?

A Physeal injuries of the distal femur do not carry much risk of growth disturbance.

B Proximal tibial physeal disruptions should be carefully evaluated for neurovascular injuries and compartment syndrome.

C Tibial tubercle avulsion can lead to genu recurvatum.

D Conservative treatment is the mainstay of patellar dislocations.

E Surgical reconstruction provides superior results if the child desires to continue an active lifestyle following anterior cruciate ligament (ACL) injury.

A

A

Physeal injuries of the distal femur carry a significant risk of growth disturbance because of its complex undulating shape, which decreases the odds of a clean cleavage plane when injured thus increasing the risk of focal damage to the physis.

The neurovascular structures about the knee are at some risk with any injury in that region but this risk is greatest for injury to the proximal tibia because of tethering of the popliteal artery at its trifurcation, to the peroneal nerve around the proximal fibula and to the tibial nerve at the proximal interosseous membrane.

Genu recurvatum can develop rarely in tibial tubercle fractures that occur in patients younger than 11 years because of disruption of the anterior aspect of the proximal tibial growth plate.

Conservative treatment is the mainstay of patellar dislocations and includes strengthening of the vastus medialis obliquus, hamstring stretching, proprioceptive training and patellar bracing or taping.

Intra-articular reconstruction using bone tunnels may be performed on boys that have achieved skeletal maturity or on girls that are at least 14 years of age.

In younger children with ACL injuries controversy exists regarding treatment for fear of disrupting adjacent growth plates during surgery.

SPSE 1

194
Q

Which of the following is not true regarding paediatric ankle fractures?

A Most commonly occur in patients 8–15 years of age.

B The non-uniform closure of the distal tibial physis results in transitional fracture patterns like the triplane fracture and Tillaux’s fracture.

C Diagnostic imaging should only include CT scans.

D If the fracture does not involve the joint surface it can usually be treated with closed reduction and casting + minimal fixation.

E One of the most common ankle injuries in children is the isolated Salter–Harris 1 fracture of the distal fibula, which is often misdiagnosed as an ankle sprain.

A

C

Initial diagnostic imaging should always consist of antereoposterior, lateral and mortise radiographs of the affected ankle.

Accessory ossification centres should not be confused with fractures.

CT scanning is recommended when plain radiographs show intra-articular fractures with questionable displacement, as CT allows for more accurate estimation of articular displacement and surgical planning.

The closure of the distal tibial physis starts centrally and then proceeds in a medial to posterior direction with the anterolateral portion of the physis being last to close.

The unfused areas of this physis represent areas of relative weakness and are prone to fracture.

Salter–Harris 1 fractures of the distal fibula can be misdiagnosed with ankle sprains and are treated with 3–4 weeks of cast immobilisation.

SPSE 1

195
Q

Which of the following statements about a humerus fracture is not true?

A The proximal humerus fracture has tremendous potential for remodelling and malunions are rare.

B The proximal humerus fracture can be treated with a sling and a swathe.

C An entrapped biceps tendon in the fracture site is an indication for open reduction and internal fixation of the proximal humerus.

D Humeral shaft fractures in children are always treated with open reduction and internal fixation.

E Almost all associated radial nerve injuries in a paediatric patient can be treated conservatively with observation.

A

D

Humeral shaft fractures in children are rarely treated with open reduction.

Almost all associated radial nerve injuries in paediatric patients can be treated conservatively with observation, and full recovery should be expected.

In the event of non-recovery within 3–4 months, electrodiagnostic tests and surgical exploration are warranted.

The proximal humeral physis contributes 80% of the growth of the humerus as a result of which there is good remodelling and malunions are rare.

Most fractures of the proximal humerus are treated conservatively with a sling and swathe, which serves to provide comfort.

Interposition of an entrapped biceps tendon is a clear indication for open reduction and internal fixation.

SPSE 1

196
Q

Regarding fractures of the paediatric elbow which of the following statements is untrue?

A Knowledge of the sequence, timing and appearance of secondary ossification centres around the elbow is essential for making the correct diagnosis.

B Most displaced fractures of the lateral condyle can be treated conservatively.

C A high incidence of medial epicondylar fractures (60%) has been reported in association with elbow dislocations.

D Radial neck fractures with malalignment of greater than 30 degrees should be reduced.

E The most common nerve injured in paediatric supracondylar fractures is the anterior interosseous branch of the median nerve.

A

B

All displaced lateral condyle fractures require open reduction and internal fixation as they involve an articular surface.

Care is taken to avoid disruption of the blood supply of the trochlea by avoiding posterior dissection.

Careful anterior exposure is performed of the joint and fracture to ensure adequate reduction of articular surface.

Knowledge of the sequence, timing and appearance of secondary ossification centres around the elbow is essential in making the correct diagnosis.

The order of appearance may be abbreviated by CRIToE, which stands for capitellum, radial head, medial (internal) epicondyle, trochlea, olecranon and lateral (external) epicondyle.

A high incidence of medial epicondylar fractures (60%) has been reported in association with elbow dislocations.

A major concern following this injury is incarceration of the medial epicondyle in the joint.

Radial neck fractures with a malalignment of greater than 30 degrees should be reduced using closed or percutaneous means.

Open reduction in these situations has a high rate of osteonecrosis and non-union.

The anterior interosseous branch of the median nerve is the most common nerve injured in paediatric supracondylar fractures.

median nerve function is assessed by testing thenar opposition for intrinsic innervation, and the anterior interosseous nerve is evaluated by assessing flexor pollicis longus and flexor digitorum profundus to the index finger.

most displaced paediatric supracondylar fractures are treated by closed reduction under radiographic control followed by stabilisation with two to three lateral-entry divergent K wires and long arm casting.

medial pins are avoided for fear of iatrogenic ulnar nerve injury.

SPSE 1

197
Q

Which of the following is untrue for paediatric forearm fractures?

A Diaphyseal fractures of the forearm can be divided into three categories according to pattern of injury: plastic deformation, incomplete or greenstick, and bicortical or complete fractures.

B Reducing and stabilising the ulnar fracture is the key to maintaining radial head reduction in Monteggia’s fractures.

C Indications for surgical treatment in bicortical forearm fractures include irreducible or unstable fractures, floating elbow injuries.

D Distal radial fractures are rare in children.

E Most distal radial physeal fractures are Salter–Harris type 2 fractures that are treated with closed reduction and above-elbow cast mobilisation.

A

D

Between 75% and 84% of all forearm fractures in children involve the distal radius, of which 15%–20% involve the distal radial physis.

Diaphyseal fractures of the forearm can be divided into three categories based on pattern of injury.

Clinical and radiographic manifestations of plastic deformation may be subtle and at times, if left uncorrected, may result in limitation of forearm rotation.

Greenstick fractures account for approximately 50% of diaphyseal fractures and typically occur in younger children.

Greenstick fractures are treated with closed reduction followed by a well-moulded cast.

Bicortical fractures occur in older children and have a higher predisposition for instability.

Indications for surgical treatment in bicortical forearm fractures include irreducible or unstable fractures and floating elbow injuries.

Stabilisation may be achieved with a plate and screw construct or intramedullary fixation.

Intramedullary fixation usually requires additional postoperative cast immobilisation as it does not provide rotational stability.

SPSE 1

198
Q

Which of the following is true regarding injuries of the paediatric spine?

A Special care needs to be during immobilisation and transport of the paediatric patient, keeping in mind certain anatomical differences in the paediatric upper body from adult patients.

B SCIWORA (spinal cord injury without radiological abnormality) is more common in the paediatric population than the adult population.

C MRI is the test of choice for determining and defining spinal cord injury.

D Spondylolysis is believed to be related to repetitive microtrauma.

E All of the above.

A

E

Compared with adults, the head of a child is proportionately larger. As a result, immobilisation on a standard adult backboard will lead to flexion of a child’s neck and potentially worsen a cervical spine injury.

Hence an infant or toddler with possible neck injury should be transported with a pad under the shoulders in order to maintain the neck in a neutral position.

SCIWoRA is characterised by presence of a spinal cord injury in a patient with normal radiographic studies.

The cervical spine is most commonly involved and neurologic injuries may be complete or incomplete.

MRI can usually delineate cord changes such as swelling, contusion or infarction.

Spondylolysis, which is a stress fracture of the pars interarticularis, is not caused by a single traumatic event but believed to be related to repetitive microtrauma.

It usually occurs in adolescents who are highly active.

Bracing, physical therapy focused on hamstring stretching and abdominal strengthening are the mainstays of treatment.

SPSE 1

199
Q

The treatment of severe snakebite envenomation includes all except which one of the following?

A The initial dose of snake antivenin in children is half the required dose for adults.

B Aggressive intravenous hydration should follow the administration of antivenin.

C Extremity fasciotomy is rarely necessary.

D The polyvalent form of antivenin is more allergenic than the ovine Fab antivenin.

E Alkalinise the urine with bicarbonate and administer mannitol.

A

A

Mild snakebite envenomation manifests with localised oedema and absence of systemic symptoms.

These patients can be discharged from the hospital after an observation period of 24 hours.

The oedema in moderate envenomation is more pronounced; there may be localised blisters and the patient experiences non-specific systemic effects such as nausea, vomiting or tachycardia.

Signs of severe envenomation include rapid progression of local and systemic signs, hypotension, petechiae, abnormal laboratory values, coagulopathy, seizures and death.

It is important to transport the victim immediately to a specialised centre where the antivenin is administered without delay.

Snake antivenin should be administered in cases of moderate or severe envenomation, not mild.

Because the blood volume in children is small in relation to the venom concentration, the dose of antivenin in paediatric patients is usually double the required dose for adults, not half.

Immediately after the administration of antivenin, aggressive resuscitation with intravenous fluids should be instituted.

Rarely wound debridement and limb fasciotomy are needed, but the affected limb should be examined constantly in search of signs of compartment syndrome.

Crotalide polyvalent antivenin is frequently associated with allergic reactions compared with the new ovine Fab antivenin, which is safer in children.

In cases of moderate envenomation, an abnormal coagulation value without bleeding is common. Coagulopathy after severe snake envenomation usually presents with active bleeding. In this case, ICu admission and aggressive management of coagulopathy should be implemented.

Similarly, extensive rhabdomyolysis puts the patient at risk for renal failure. In this case, the urine should be alkalinised with intravenous bicarbonate and mannitol.

SPSE 1

200
Q

Which one of the following statements regarding spider bites is correct?

A The brown recluse spider is identified by its eight paired eyes and a violin-shaped cephalothorax.

B The management of mild spider envenomation requires the use of antivenin.

C Systemic signs after black widow and funnel web spider bites include autonomic and neuromuscular hyperactivity.

D The neurotoxin released by Loxosceles reclusa (brown recluse spider) causes severe cutaneous necrosis.

E Dapsone is the first-line treatment for brown recluse bite.

A

C

The two most common species of spider responsible for arachnidism in the united States are the black widow (Lactrodectus spp.) and brown recluse (Loxosceles reclusa) spiders.

Characteristically, the brown recluse spider has six paired eyes (most spiders have eight).

The funnel web (Atrax spp.) spider is a common venomous spider in Australia.

mild to moderate symptoms of arachnidism are treated with supportive measures only.

Systemic signs indicate severe envenomation and require prompt use of antivenin in children; black widow and funnel web spiders produce a potent neurotoxin responsible for autonomic hyperactivity, muscle rigidity, spasms and occasionally respiratory arrest.

localised cutaneous necrosis is often found after brown recluse spider bite and is caused by local enzymatic destruction secondary to cytotoxic venom, not a neurotoxin.

This should be treated with limb elevation, ice compresses and local wound care.

Healing takes several weeks and rarely requires extensive debridement.

There is no strong evidence to support the use of Dapsone in humans although it is frequently administered.

Rarely, loxoscelism presents with systemic toxicity such as hypotension, renal failure, seizures or even coma.

SPSE 1

201
Q

Which one of the following statements is not true regarding scorpion sting?

A Severe scorpion envenomation is more common in children than adults.

B Scorpion antivenin is contraindicated in children.

C The syndrome of severe envenomation includes neuromotor hyper-activity, cardiorespiratory compromise and visual abnormalities.

D Neuromotor hyperactivity may be treated in the ICU with high doses of benzodiazepines.

E The administration of scorpion antivenin can successfully reduce
the symptoms of severe envenomation.

A

B

Centruroides is the only species of scorpion responsible for severe envenomation in the united States.

mild envenomation causes only local burning pain and resolves spontaneously within a few hours.

However, this is not the case in children who usually have a serious clinical syndrome consisting of neuromotor hyperactivity, visual abnormalities, and respiratory compromise due to excessive secretions, bronchospasm and pulmonary oedema.

Traditionally, and before the availability of the antivenin, these patients were treated in the ICU with high doses of benzodiazepines and ventilatory support.

The prompt administration of scorpion antivenin obviates the need for such extreme measures.

This is not contraindicated in children.

SPSE 1

202
Q

Which one of the following statements is true regarding dog bites?

A The majority of dog bites in children occur in the upper extremity.

B Routine use of systemic antibiotics is recommended to treat any type of dog bite.

C Primary closure of bite wounds should only be attempted in deep lacerations involving the face.

D Staphylococcus is the most common pathogen responsible for delayed wound infection after dog bites.

E Prophylaxis against rabies should be started within the first week if the dog’s vaccination certificate cannot be obtained.

A

D

Dog bites are common in children under 10 years of age.

The head and neck area is affected almost 50% of the time, followed by the upper and lower extremities with 28% and 18%, respectively.

Superficial bite wounds should be treated with local care and the patient discharged from the emergency department with no antibiotics.

For deep lacerations, admission to the surgical floor, administration of systemic antibiotics and early surgical intervention is recommended.

Copious irrigation, debridement of devitalised tissue and closure of all deep lacerations is the standard treatment.

Although Pasteurella multocida is found in the oral flora of dogs and is associated with very early infections, Staphylococcus spp. is for the most part responsible for wound infections occurring 24 hours after the bite.

If the dog’s certificate of vaccination against rabies cannot be obtained within 24 hours, the patient should be placed on a rabies vaccine schedule. Tetanus status should be investigated.

SPSE 1

203
Q

Which one of the following statements is not true regarding the epidemiology of paediatric burns?

A Burn injuries constitute the second leading cause of accidental death of children in the United States.

B Most scald burn injuries occur in children younger than 4 years of age.

C Areas commonly affected by scald burns include the face and hands.

D A stocking-glove distribution of a burn indicates possible child abuse.

E Contact with hot liquids at temperatures of 100°F for more than 3 seconds can cause severe thermal burns in children

A

E

Thermal injuries can be caused by flame, by contact with hot surfaces or by scalding due to hot fluids.

Over 500 children per year die from severe burns in the united States making it the second leading cause of accidental death.

Spillage of hot food or beverages is the commonest cause of scald burns in children under 4 years of age.

The areas usually affected are the hands and face.

Child abuse should be suspected when the burn is localised in the perineum, has a stocking-glove distribution or when burn care has been neglected.

For a thermal burn to occur, the contact with the offending agent should be for at least for 3 seconds at a temperature greater than 120°F (49°C).

Therefore most homes in the United States have the hot water temperature set at less than 110°F (43°C).

SPSE 1

204
Q

Which one of the following statements is correct regarding skin and burn physiology?

A The total surface area of skin in a newborn is approximately 0.5 m^2 .

B According to the Lund and Browder chart, the surface area of skin represented by the head and neck in infants is approximately 25%.

C A neurovascular plexus separates the epidermis from the papillary dermis.

D Full thickness burns re-epithelialise from retained appendages in the reticular dermis.

E The zone of stasis in burned tissue has impaired blood flow secondary to an increase in local inflammatory mediators.

A

E

The skin exerts an essential role in body thermoregulation and fluid homeostasis, and is the main barrier against opportunistic organisms.

The total surface area of skin in a newborn is 0.25 m^2.

Unlike adults, the head represents approximately 18% of the body surface area in children under 4 years of age according to the Lund and Browder chart. This chart estimates the surface area in children more precisely than the classic Rule of Nines.

The depth of the burn is classified according to the affected skin layer.

Superficial or first-degree burns affect only the epidermis, are erythematous, oedematous and resolve in about a week without scarring.

Partial thickness or second-degree burns are divided into superficial and deep if the papillary or reticular dermis respectively, is affected.

Because a rich neurovascular plexus separates the papillary from the reticular dermis, a superficial partial-thickness burn is exquisitely painful, with blanching erythema and often with blisters.

It re-epithelialises in 2 weeks from residual epidermal appendages without scarring.

A deep partial-thickness burn penetrates into the reticular dermis, clinically presents with non-blanching erythema and is often insensate. It usually requires skin grafting.

A full-thickness burn involves all layers of the skin into the subcutaneous tissue. Its clinical appearance is dry and leathery and is insensate because of the complete destruction of all nerve terminals. This burn will not re-epithelialise and will require a skin graft.

Some recognise a fourth category in which the burn penetrates into the muscle, tendon or bone.

The centre of the burn is called the zone of coagulation and represents the necrotic eschar. Immediately around this area, the zone of stasis represents all the borderline viable tissue that can be rescued with appropriate management.

Intense local release of inflammatory mediators decreases the blood flow to this area. Finally, the hyperaemic zone is the viable tissue surrounding the burn.

SPSE 1

205
Q

Which one of the following statements regarding initial resuscitation after burn injuries is not correct?

A Intubation and ventilator support should be instituted in cases of extensive burns and burns in closed spaces.

B The preferred initial resuscitation fluid in children is 0.9% normal saline solution.

C The estimation of fluid requirements should consider partial- and full-thickness burns only.

D The fluid requirement in children with burns equals the sum of the fluid estimated by the Parkland formula plus the maintenance requirement.

E Urine output of 1 mL/kg/hr accurately reflects appropriate fluid resuscitation.

A

B

The ABC of resuscitation should be applied to all victims of burns, and endotracheal intubation should be considered not only in patients subjected to inhalation injury but also in those with extensive burns.

Signs of inhalation injury include singed nasal hair and carbonaceous sputum.

Following the management of the airway, large-bore IV lines should be obtained, preferably but not necessarily away from the area burned.

Anticipate the need for central lines to avoid displacement of short IV catheters when massive oedema is expected.

lactated Ringer’s solution (lR) is perhaps the best initial resuscitation fluid, not only because of its low sodium concentration but also because of its buffering effect on metabolic acidosis.

Estimates of resuscitation fluid may be achieved by various methods and are based on the affected body surface (TBSA), weight and only begun for partial and full-thickness burns, not superficial.

In addition, most clinicians agree that resuscitation should be started only when the TBSA burned is more than 15% in children. The Parkland formula is the most used and is applied for the first 24 hours as follows:

4 ml × TBSA(%) × weight(kg).

Half the estimated amount should be administered within the first 8 hours after the event (not the first time seen in the ER) and the other half in the remaining 16 hours.

For children under 4 years old, the maintenance fluid requirement and dextrose should be added to the calculated amount, as D5 LR.

After the first 24 hours, when capillary permeability has stabilised, the addition of albumin may be beneficial.

The end point of resuscitation is the appropriate perfusion of end organs. A good indicator of a good resuscitation is a urine output of at least 1 ml/kg/hr.

SPSE 1

206
Q

Which one of the following statements is true regarding topical agents for burns?

A Silver sulfadiazine penetrates the burn eschar efficiently but causes hyponatraemia.

B Silver sulfadiazide is effective against Pseudomonas.

C The main side effect of Sulfamylon is leucopenia.

D Silver nitrate is an alternative topical agent for facial burns.

E Antibiotic ointments are recommended for facial burns and small
partial-thickness burns.

A

E

The initial management of most partial- and full-thickness burns consists of topical preparations.

These agents prevent, or at least minimise, the bacterial colonisation of the wound and provide a favourable environment for wound healing or in preparation for skin grafting.

Superficial or first-degree burns do not require any special treatment. Aloe vera ointments and protection from the sun are sufficient measures to promote healing.

For partial-thickness burns, excision of large blisters and debridement of the eschar under sedation is necessary for proper burn care.

Small blisters are left alone because they act as a natural barrier against microbial invasion. once exposed, the dermis or deeper tissues should be covered with various preparations.

Silver sulfadiazide is an excellent agent, but does not penetrate the eschar and sometimes produces reversible leucopenia. It has poor coverage against Pseudomonas.

mafenide acetate or Sulfamylon penetrates the eschar and protects against Pseudomonas, but its administration is painful and causes metabolic acidosis due to carbonic anhydrase inhibition.

Silver nitrate has a broad coverage and its application is painless. Its major side effects are electrolyte abnormalities such as hyponatraemia and silver staining of the skin. This side effect limits its use on the face.

For small partial-thickness burns or burns on the face, antibiotic ointment is recommended.

SPSE 1

207
Q

Which one of the following statements concerning burn dressings is not correct?

A The ideal wound dressing promotes spontaneous epithelialisation, acts as a scaffold, provides moisture and is durable.

B Site, extent and depth of the wound determines the type of dressing selection.

C Membrane dressings are biological films that promote epithelialisation after adherence to the burn bed.

D Decellularised dermis serves as a scaffold for epithelial cell migration in full-thickness burns.

E Porcine skin can be used as a temporary skin substitute when large burned surfaces need coverage.

A

C

The first step in the management of partial- or full-thickness burns is the application of an appropriate dressing.

This should not only provide coverage and prevent fluid losses but also act as a barrier against infection, serve as a scaffold for re-epithelialisation, and provide moisture and durability, thus avoiding frequent and painful dressing changes.

The type of dressing depends largely on the depth and extent of the burn.

For most superficial partial-thickness burns, topical creams such as silver sulfadiazide followed by the application of membrane dressings is enough.

membrane dressings are synthetic films that adhere to the burned skin, serving as a scaffold. Silver-impregnated films are an example and they do not have to be changed at all because they separate off after complete epithelialisation has occurred.

Silver components should be avoided on the face because they cause staining.

Decellularised dermis is a biologic agent that serves the same purpose and its use is appropriate in cases of full-thickness burns.

often, large burns require extensive coverage that can only be achieved through the use of temporary substitutes such as porcine skin. meshing the skin graft is a simple method to expand the graft and accomplish coverage, but should be avoided on the face, neck or hands because of poor cosmetic results.

SPSE 1

208
Q

Which of the following statements is true regarding skin grafting?

A Excision and skin grafting should be delayed for at least 2 weeks after the inflammatory response has subsided.

B Blood loss from excision of the eschar is prominent during the first 48 hours.

C Partial thickness autografts are less prone to wound contraction and their ‘take’ is always 100%.

D Full-thickness allografts are prone to excessive contraction.

E The donor site of full-thickness grafts may be closed primarily.

A

E

It is important to remove burned devitalised tissue to cease the immunosuppressive and inflammatory response.

Several studies demonstrated that delayed excision and grafting expose the patient to risk of wound infection, sepsis and multiorgan failure.

After the first 48 hours, the burn has already started the process of angiogenesis, this may be responsible for substantial bleeding during the delayed excision of the eschar.

The most common technique of excision is the tangential method, using a sharp blade or dermatome. The excision should be continued until viable tissue is observed or sometimes to the fascial plane.

Skin grafts are named according to their origin. A graft from the same patient is an autograft; it as an isograft if it comes from a twin, an allograft if it comes from another human and a xenograft if it comes from another species.

According to the layers of skin included in the graft, it is classified as split-thickness skin graft (STSG) if it includes the epidermis and dermis, and full-thickness skin graft (FTSG) if it includes all layers of the skin.

The STSG donor sites should be covered with a temporary dressing such as petroleum gauze.

These dressings will slough off the wound after epithelialisation occurs.

FTSG take is excellent and it undergoes less contraction than STSG, which is paramount when a graft is used in hands or joints.

Even more, the colour matching is optimal in cosmetic areas like the face.

For FTSG, primary closure or STSG should be done.

SPSE 1

209
Q

Which one of the following statements is incorrect regarding the hypermetabolic response to burns?

A Between 20% and 40% of the total caloric intake should be derived from proteins.

B Intravenous hyperalimentation is currently recommended to boost the immune system.

C Enteral feeding prevents intestinal mucosal atrophy and decreases the level of catabolic hormones

D Formulas to calculate the caloric requirements in burned children are based on burned body surface.

E Hyperglycaemia in burn patients is caused by hepatic insulin resistance.

A

B

The paediatric patient is extremely vulnerable to malnutrition after burns.

The small proportion of fat and muscle in children compared with adults makes them prone to protein-caloric malnutrition. Hence it is important to provide a balanced nutrition consisting of protein-derived calories of about 20%–40% and carbohydrate-derived calories of 40%–70%.

Several catabolic hormones such as catecholamines and cortisol are released as a primary response and their action can persist for months. These, along with the potent inflammatory response cause physiologic stress and increased metabolic demands.

Intravenous hyperalimentation, once thought to be useful in burned patients, is now avoided because of its deleterious effects on the immune system and the gut.

The enteral route is preferred because it not only achieves the nutritional goals but also prevents intestinal mucosal atrophy, translocation and indirectly decreases the levels of catabolic hormones.

The estimation of the caloric requirements can be obtained from the Curreri formula as follows:

25 kcal/kg + 40 kcal x %TBSA.

Appropriate enteral nutrition should be instituted as soon as possible either by the oral route or through feeding tubes. Delays can result in malnutrition, organ dysfunction and delayed wound healing.

Hyperglycaemia in the burned patient results from hepatic insulin resistance and the persistent effect of catabolic hormones.

SPSE 1

210
Q

Which one of the following statements is correct regarding inhalation injury?

A Severe symptoms of inhalation injury include cough, wheezing and shortness of breath.

B Pulmonary injury can be ruled out if the initial chest X-ray (CXR) and arterial blood gas (ABG) are normal.

C Oxygen therapy should be started when carboxyhaemoglobin levels exceed 20%.

D The half-life of carboxyhaemoglobin is approximately 2 hours.

E Lactic acidosis, high mixed-venous saturations and unresponsiveness to oxygen therapy are indicators of cyanide toxicity.

A

E

The clinical presentation of inhalation injury ranges from mild respiratory symptoms such as cough, shortness of breath and wheezing to a more-severe clinical picture with confusion, coma and cardiopulmonary arrest.

Smoke inhalation injury affects four different levels:
(1) upper airway, causing swelling and ulceration of the mucosa;
(2) tracheobronchial, causing oedema and formation of ‘airway casts’ leading to bronchial obstruction;
(3) alveolar, causing alveolar oedema, loss of surfactant and excessive inflammatory infiltrate; and
(4) systemic, due to hypoxaemia and the absorption of toxins such as carbon monoxide (CO) and cyanide.

A baseline CXR and ABG should always be obtained and repeated in 12–24 hours because pulmonary injury is often underestimated by initial radiographs.

Supplemental oxygen at 100% by non-rebreather mask should be instituted early in the management of smoke inhalation injury and not delayed by waiting on CO levels.

If appropriate, intubate the patient in anticipation of worsening airway oedema.

Perform an early bronchoscopy to determine the extent of injury and to clear the bronchi from soot, accumulated mucus and airway casts.

Administer bronchodilators and institute aggressive pulmonary toilet.

The role of steroids in the management of inhalation injury is controversial.

CO has 200-fold affinity for haemoglobin compared with oxygen and shifts the dissociation curve left, making oxygen less available. Symptoms secondary to elevated carboxyhaemoglobin are usually present when the level is above 10%.

The administration of 100% oxygen decreases the half-life from 6 hours to 60 minutes. Hyperbaric oxygen at 2–3 atm reduces even more the half-life of CO to 15 minutes, but its availability is limited.

Cyanide blocks oxidative phosphorylation by inhibition of the cytochrome oxidase, resulting in increased lactic acid levels, metabolic acidosis and high mixed-venous saturation. It is released from the partial combustion of plastics.

Treatment of cyanide toxicity should include a combination of oxygen with sodium thiosulfate or amyl nitrite.

Hydroxycobalamin (vitamin B12 ) creates cyanide compounds that facilitate its excretion by the kidneys.

SPSE 1

211
Q

Which one of the following statements regarding caustic dermal injuries is not correct?

A Irrigation of alkali burns should be performed for at least 12 hours.

B High-pressure burn irrigation is preferred to low-pressure irrigation.

C Exothermic acid–base reactions limit the use of neutralising agents.

D The corneal pH after irrigation should be at least 7.3 to prevent permanent damage.

E Intravenous magnesium is as effective as intra-arterial calcium infusion for hydrofluoric acid burns.

A

B

Caustic agents include acids and alkalis. They are found in multiple detergents, housecleaning and plumbing agents.

Acids produce coagulation necrosis and their penetration is limited by the formation of a thick eschar.

Sulphuric acid is the most common acid associated with chemical burns and typically produces a black eschar.

once identified, an acid burn should be irrigated for at least 2–3 hours.

Alkalis produce liquefaction necrosis that may persist for several hours, so it is important to irrigate the affected tissues for at least 12 hours.

After the contaminated clothing is removed, low-pressure irrigation should be started.

Tap water is preferable to neutralising agents because the latter could generate an exothermic acid–base reaction that would produce further tissue damage.

ocular chemical burns should be irrigated from 30 to 60 minutes and the pH of the cornea measured. Any level below pH 7.3 requires that the irrigation should be continued. ophthalmology consultation should be obtained immediately.

Fluoride ions released by hydrofluoric acid avidly penetrate tissues and cause severe burns with uncontrollable pain.

For small and superficial burns with hydrofluoric acid, the application of 2.5% calcium gluconate gel is sufficient.

For deep burns or burns on the hand, it is necessary to inject calcium gluconate either subcutaneously or intra-arterially.

Calcium sequesters fluoride ions.

Recent studies have shown that intravenous administration of magnesium is as effective as, and less complicated than, the administration of intra-arterial calcium.

SPSE 1

212
Q

Which one of the statements below is not correct regarding electrical injuries?

A High voltage is defined as greater than 1000 V.

B The entry and exit wounds caused by alternating current are usually full thickness.

C Breaks in the skin lower its resistance to electrical current.

D The electrical path in the direction of one hand to another can be associated with ventricular fibrillation.

E An asymptomatic patient with a normal initial ECG does not require continuous monitoring.

A

B

Electrical injuries are common in children. The current and voltage are directly related to the degree of injury caused by electricity.

Alternating current of low voltage (< 1000 V), commonly found in homes, deliver an electrical current that generates a prolonged muscle tetany and cardiac conduction abnormalities.

Ventricular fibrillation is the most common arrhythmia.

The entry and exit electrical injury wounds are usually small and full thickness. They are not usually associated with alternating current, rather, are the result of direct-current or high-voltage electricity and may indicate serious injury to internal organs.

Impedance is defined as resistance to electrical current flow. The resistance is low in tissues such as muscles, nerves and vessels and high in the skin, bone and fat. So, the skin offers significant protection against electrical injuries. This resistance decreases when the skin is wet or damaged.

An electric current path from a hand to a hand or hand to foot, travels through the chest and may cause cardiac arrhythmias such as ventricular fibrillation.

All patients with electrical burns should have a workup that includes an ECG, complete blood cell count, electrolytes, creatine phosphokinase level and urinalysis.

A normal initial ECG in a patient with a low-voltage electrical injury, no history of seizures or loss of consciousness and a normal urinalysis can be discharged after a few hours of observation in the emergency room.

Patients with severe injuries, ECG abnormalities, loss of consciousness during the event, myoglobinuria or high-voltage injuries must be hospitalised.

SPSE 1

213
Q

Which of the following are not at increased risk of child abuse or neglect?

A families living in areas of community violence
B children born prematurely
C children with learning disability
D families with religious faith participation
E children exposed to domestic violence

A

D

Several risk factors have been identified for child abuse and neglect. These risk factors fall into the categories of child factors, parental factors and environmental factors.

more recently researchers and policy makers have identified protective factors against abuse.

The most significant risk factors are having a parent aged younger than 21 years, having a history of mental illness or depression, or living with a violent adult.

Religious faith participation has been found to be one of several social/environmental protective factors.

SPSE 1

214
Q

Which of the following is not true?

A Parental responsibility may be acquired through the court by a residence order.

B Both parents have parental responsibility if they were married at the time of the child’s birth.

C Both parents have parental responsibility if they were cohabiting
at the time of the child’s birth.

D A father may acquire parental responsibility by marrying the
mother.

E Parental responsibility can be exercised by one person independently.

A

C

Parental responsibility – the Children Act 1989 introduced the concept of parental responsibility.

Parental responsibility can be held by one or more people at a time. It is important in cases of abuse or for consenting to medical treatment.

Fathers have responsibility only if they were married to the mother at the time of birth, they marry the mother later, they obtain a court order, they have a formal agreement with the mother, or are present at the registration of the birth and are recorded as such.

SPSE 1

215
Q

Bruising to which of the following areas is likely to be accidental?

A neck
B ear
C buttocks
D hip
E genitalia

A

D

Bruising is a common injury from physical child abuse and from accidental causes.

Bruises in certain areas are predictive of abuse.

Any bruise in an area that is protected should be given a high index of suspicion (neck, ears, torso, genitalia), areas over bones are more likely to be accidental (limbs, spine, hips).

SPSE 1

216
Q

In a mobile child who is currently well, with multiple bruises to her legs and buttocks and a slightly swollen left ankle, which of the following is most likely?

A meningococcal sepsis
B von Willebrand’s deficiency
C idiopathic thrombocytopenic purpura (ITP)
D Henoch–Schönlein’s purpura (HSP)
E Ehlers–Danlos’s syndrome

A

A ? Should be D?

Meningococcal sepsis causing multiple bruises is usually seen in a child who is very unwell, peripherally shut down, drowsy and who may be pyrexial.

Von Willebrand’s deficiency usually presents with prolonged bleeding after a surgical/dental procedure or because of a family history.

This picture could be seen in ITP but the bruising is usually all over the body rather than confined to the lower limbs.

HSP commonly presents with petechiae and purpura and often with inflamed joints.

Ehlers–Danlos’s can present with easy bruising but this is rarely seen, and would not normally be seen acutely.

217
Q

Which of the following is least likely to be mistaken for child abuse?

A capillary haemangioma
B Mongolian blue spot
C café au lait macules
D varicella zoster infection
E sucking blisters

A

D

Capillary haemangiomas, though often present at birth, tend to darken and become more visible after a few weeks/months and can therefore be mistaken for new unexplained bruises.

mongolian blue spots look very similar to bruises over the spine and therefore if not known about could cause confusion.

Café au lait macules also develop over time and are often on the buttocks and thighs.

Varicella zoster is usually easily recognised because it is so common.

Sucking blisters are small blisters that can be mistaken for burns but are always in an area that a baby can get into their mouth to be able to suck.

SPSE 1

218
Q

A full blood count and coagulation screen will miss which of the following?

A haemophilia
B platelet function defect
C idiopathic thrombocytopenic purpura
D warfarin ingestion
E disseminated intravascular coagulation

A

B

Haemophilia can be diagnosed with a very raised activated partial thromboplastin time (aPTT).

Blood count in ITP will show very low platelet count.

Warfarin ingestion will show a raised prothrombin time (PT) and INR (interenational normalised ratio).

Disseminated intravascular coagulation will show raised aPTT, PT and fibrinogen is likely to be low.

In a platelet function defect the platelet count will be normal and therefore platelet function testing is needed if suspected.

SPSE 1

219
Q

Which of these investigations would not usually be required in a 9-month-old with unexplained bruising to the abdomen?

A skeletal survey
B vitamin D levels
C CT head
D CT abdomen
E ophthalmological examination

A

B

Bruising to the cheek in a non-mobile child would be indicative of nonaccidental injury.

In a child under 2 years a skeletal survey, CT head and ophthalmology examination are indicated.

With bruising to the abdomen a CT abdomen is important to look for internal injuries.

Vitamin D levels would not be required unless these investigations show fractures or bony abnormalities.

SPSE 1

220
Q

Which of the following fractures has a high specificity for child abuse?

A long bone diaphyseal
B rib
C long bone metaphyseal
D scapula
E spine

A

A

Diaphyseal fractures are four times more common than metaphyseal fractures.

Metaphyseal fractures can be caused by gripping, twisting, pulling and possibly shaking. They are often not accompanied by swelling and bruising, and may not develop callus.

SPSE 1

221
Q

Rib fractures are best detected:

A by chest X-ray (CXR) following acute presentation of injury
B as part of skeletal survey
C by delayed CXR
D clinically with bruising
E following detection of visceral injuries.

A

C

Rib fractures may not be easy to see on an acute CXR, or on skeletal survey if the injury has been recent, therefore a delayed CXR (2 weeks later) is advised.

SPSE 1

222
Q

Indicators of sexual abuse include:

A recurrent UTIs in a toddler
B constipation in a 10-year-old requiring regular laxatives to ensure regular bowel movements
C a 7-year-old girl wanting to dress like her mum
D rectal prolapse in a toddler
E wrist-cutting in a 14-year-old girl.

A

E

Recurrent UTIs are common in children who aren’t yet potty-trained, particularly if there is constipation.

Constipation itself is very common in all ages of children but if it is intractable or more often there is encopresis then all forms of abuse should be considered.

Young girls commonly wish to mimic their mother’s behaviour and appearance.

Rectal prolapse is common in children under 5 and is usually secondary to constipation but can be associated with cystic fibrosis and Hirschprung’s disease.

Self-harm is an important presentation of sexual abuse but may be hard to elucidate.

SPSE 1

223
Q

A 13-year-old girl is referred with abdominal pain and is found to be pregnant. She discloses that her Mum’s partner is likely to be the father of the child. She states she doesn’t want her mother to know. What should you do?

A Tell her that what she tells you is confidential.

B Refer to the obstetric unit and expect that they will look into the relationship.

C Contact social services without telling her.

D Advise her that her safety is a concern and in this case you must break her confidence to inform social services.

E Talk to her mother and mother’s partner about these allegations.

A

D

Confidentiality is the responsibility of any doctor who suspects abuse. Although the patient’s relationship may be consensual, there is a high likelihood the girl has been manipulated and the age difference is likely to be big.

As she is only 13, a sexual relationship is illegal.

SPSE 1

224
Q

A 3-year-old boy with quadriplegic cerebral palsy is brought to your outpatient clinic. He had a percutaneous endoscopic gastrostomy (PEG) tube inserted by you last year because of unsafe swallowing. He has gained very little weight over the last year and appears very quiet. His PEG site is very inflamed and excoriated. When you examine him his nappy is soiled and he has an extensive excoriated nappy rash. There is also an area of skin breakdown over his back. Mum tells you she saw the GP recently but he did not do anything. When you check the notes he has failed to attend his neurodisability clinic appointments. The best course of action would be:

A referral to social services

B ask Mum to take him back to the GP about the areas of skin breakdown

C discuss his case with other professionals involved

D admit him for observation of his feeding

E copy a clinic letter to the neurodisability consultant he sees.

A

A

This child is showing many signs of neglect. He is at increased risk of abuse because of his disability.

A referral to social services is indicated as a child in need. It would also be useful to follow this up with discussions with other professionals involved in his case.

A multi-agency assessment would be indicated.

options C and E would also be important but not the most important.

SPSE 1

225
Q

An 18-month-old boy was transferred to a neurosurgical unit with severe head trauma. The history given was that he had climbed onto a chest of drawers and fallen onto a wooden floor. Initial investigations at the referring hospital showed a mildly reduced haemoglobin, normal urea and electrolytes and raised transaminases (AST 250, ALT 458). Bilirubin level was normal. What should your next step be?

A exclude abdominal injury on clinical examination

B repeat transaminases and if falling, observe

C CT abdomen

D laparotomy

E clinical observations

A

C

Occult liver injuries occur as a result of physical abuse.

Raised transaminases are a good indicator of abdominal injury.

The result should be followed up by CT abdomen.

The history given is unlikely to have caused such severe head trauma, therefore abuse is likely.

SPSE 1

226
Q

Which of the following should be treated urgently with vitamin K and fresh frozen plasma?

A a large subaponeurotic haemorrhage
B a chignon
C a cephalhaematoma
D a caput succedaneum
E a laceration to the scalp from forcep use

A

A

A subaponeurotic haemorrhage is a haemorrhage between the aponeurosis and periosteum and is therefore not limited by the suture lines.

It is rare but is associated with vacuum and instrumental deliveries.

A chignon is the area where the vacuum was applied in a vacuum delivery.

A cephalhaematoma may follow an instrumental or normal delivery but is differentiated from the subaponeurotic haemorrhage because the haemorrhage does not cross suture lines.

A caput is a serosanguineous subcutaneous effusion over the presenting part in a vaginal delivery. It does cross the suture lines but resolves quickly.

Lacerations are not uncommon and may require a Steri-Strip, but extremely rarely any further treatment.

SPSE 1

227
Q

Which of the following is not suggestive of an Erb’s palsy?

A weak hand movements

B biceps paralysis

C grasp reflex present

D Moro reflex is asymmetric

E over 70% recover completely without intervention

A

A

In an Erb’s palsy (C5, C6) the arm is limp from paralysis of deltoid, biceps, brachioradialis and long wrist extensors but finger movements and grasp reflex are preserved.

SPSE 1

228
Q

In hypoxic ischaemic encephalopathy, which is the most appropriate intervention?

A controlled hypothermia
B anti-epileptic medication
C hyperventilation to reduce PCO2
D fluid resuscitation
E nasogastric feeding

A

A

Controlled hypothermia has been shown to improve outcome in the mild to moderate encephalopathy groups.

It is now used in tertiary neonatal units under strict criteria.

SPSE 1

229
Q

A 6-month-old is referred to your outpatient clinic with undescended testes. When you examine him you notice he has dirt under his nails and a severe nappy rash. What should you do?

A Treat him the same as any other referral.

B Contact his health visitor.

C Ask Mum why he is dirty.

D Contact the local safeguarding board to find out if they have information on the child.

E Record in your notes that he appears dirty.

A

B

Dirt under the nails of a non-mobile child could signify neglect.

Nappy rash is common but a caring parent would have sought advice for it.

Contacting the health visitor would enable further monitoring and then lead onto social services referral if necessary.

SPSE 1

230
Q

A 7-year-old boy is brought into the emergency department with a supracondylar fracture of his right arm. This is his third fracture over 2 years. He says he jumped off a wall and slipped. What should you do?

A Contact social services.
B Contact local safeguarding board.
C Ask a paediatrician to see him.
D Treat his fracture and then allow home.
E Treat his fracture and then ask a paediatrician to see him.

A

D

It is not uncommon for mobile normal children aged 4–7 years to have several fractures.

The sites are commonly radius/ulna and tibia.

SPSE 1

231
Q

A 3-year-old child is referred to the child development centre for a developmental assessment. He is found to be microcephalic and an MRI shows an underdeveloped brain. Which of the following could not be the cause?

A antenatal infection

B lack of stimulation during early life

C neonatal stroke

D meningitis at age 2½

E a chromosomal disorder

A

D

Antenatal ‘TORCH’ infections are well-known causes of microcephaly and developmental delay.

They include toxoplasmosis, rubella, cytomegalovirus and herpes.

lack of stimulation is a well-recognised cause of developmental delay and microcephaly. The orphans in Romanian orphanages (Rutter, 1998) are a clear example of this.

Neonatal stroke if severe could cause profound long-term problems, and many chromosomal disorders may be found at investigation for microcephaly.

A more recent meningitis may well cause arrest of development and brain growth but would be unlikely to cause an underdeveloped brain.

SPSE 1

232
Q

The United Nations convention on the Rights of the Child does not state that:

A governments must do all they can to fulfil the rights of every child

B a child with a disability has a right to live a full and decent life

C every child has the right to privacy

D every child has the right to relax, play, and join in a wide range of cultural and artistic activities

E everyone under the age of 16 has all the rights in the convention.

A

E

Article 4 of the Convention on the Rights of the Child states that governments must do all they can to fulfil the rights of every child.

Article 23 states that a child with a disability has a right to live a full and decent life.

Article 16 states that every child has the right to privacy.

Article 31 states that every child has the right to relax, play and join in a wide range of cultural and artistic activities, and

Article 1 states that everyone under the age of 18 has all the rights in the convention.

SPSE 1

233
Q

A 33-month-old girl is brought to clinic for follow-up. These are her growth charts. Which of the options is most likely?

A The child is PEG fed.

B She has been in foster care from age 18 months to 2½ years.

C She suffers from a malabsorption syndrome.

D She is fed by parenteral nutrition.

E She is on long-term steroid treatment.

A

B

The child’s growth charts show the ‘saw-tooth’ appearance of non-organic faltering growth.

The height was on the 50th centile at birth but then steadily dropped down.

She went into foster care at 18 months and her weight quickly improved. Her height then caught up.

As she went back to the care of her parents she stopped gaining weight as well.

SPSE 1

234
Q

A 5-year-old child sustains a deep laceration to his right leg from a boat propeller. The limb is neurologically intact. The foot is cool to the touch and painful. During wound exploration, there are long segments of injured superficial femoral artery and vein. What would be your plan of treatment?

A wound closure, anticoagulation and observation

B repair of the superficial femoral artery with a prosthetic graft

C repair of both the superficial femoral artery and vein using saphenous vein

D repair of the superficial femoral artery using a saphenous vein graft/vein ligation

E leg amputation if vessels too small to repair

A

D

In general, paediatric vascular trauma is managed similarly to adult vascular trauma and the expected long-term results are equivalent to repairs in adults.

At 5 years of age, the femoral vessels are large enough to surgically repair with the anticipation of having a good outcome if there are no significant associated orthopaedic or neurologic injuries.

The saphenous vein is generally a large enough conduit in this patient age group and a prosthetic graft is not needed or beneficial.

Repair of the superficial femoral artery alone will result in a viable limb. The vein should only be repaired if it is a reasonable undertaking and the patient is stable.

SPSE 1

235
Q

What is the triad of hemobilia in trauma?

A

Hematemesis
Abdominal pain
Jaundice

236
Q

What is the main predictor of injury pattern?

A

Injury mechanism is the main predictor of injury pattern.

The body regions most frequently injured in major childhood trauma are the lower extremities, head and neck, and abdomen.

In minor childhood injury, soft tissue and upper extremity injuries predominate.

Motor vehicle versus pedestrian trauma may result in the Waddell triad of injuries to the head, torso, and lower extremity (pelvis, femur, or tibia).

Motor vehicle accidents may cause head, face, and neck injuries in unrestrained passengers.

Cervical spine injuries, bowel disruption or hematoma, and Chance fractures occur in restrained passengers.

Bicycle trauma results in head injury in unhelmeted riders as well as upper extremity and upper abdominal injuries, the latter the result of contact with the handlebar.

Direct impact from a bicycle handlebar may be predictive of the need for operation.

H&A

237
Q

In the evaluation of blunt abdominal trauma, what are the criteria for omitting abdominal computed tomography?

A

Shock index (maximum heart rate divided by minimum systolic blood pressure [normal <0.9]) and the pediatric adjusted shock index (SIPA) have a moderate ability to identify children with intra-abdominal injury needing transfusion or other intervention.

For SIPA, the cut-off values by age are

> 1.22 (ages 4–6)
1.0 (ages 7–12)
0.9 (13 years and older)

In a recent study, SIPA was able to identify 93% of children with a grade 3 or higher liver or spleen injury requiring transfusion within the first 24 hours, but was not as sensitive for more minor injuries.

Computed tomography (CT) remains the diagnostic study of choice in evaluating blunt abdominal trauma. Recent studies by the Pediatric Emergency Care Applied Network (PECARN) suggest a selective approach to imaging is safe.

For children with a Glasgow Coma Scale (GCS) score of 15, physical examination findings of abdominal pain alone had a 79% sensitivity for the identification of significant intra-abdominal injury, but the sensitivity decreased rapidly for children with a lower GCS.

In fact, even for children with a GCS of 14, half of the children with intra-abdominal injuries did not have abdominal pain on examination.

The PECARN network suggests CT may be safely omitted if seven criteria are met (Table 16.1).

In another recent study, children with a nonmotorized mechanism of injury, normal GCS, and normal age-adjusted shock index were unlikely to have a solid organ injury that required intervention. In contrast, in another review, the physical examination finding of abdominal wall bruising was quite significant, with 19% of these children having an intra-abdominal injury, and 11% having a bowel injury.

Abdominal wall bruising appears to warrant serial abdominal examinations regardless of the decision to image.

Many centers use liver transaminases to determine the need for CT scan in patients without other indications for imaging. Thresholds of >100 are often used to guide the use of CT scanning.

In a recent retrospective study, aspartate aminotransferase/alanine aminotransferase (AST/ALT) threshold of <400/<200 had a negative predictive value of 96% in predicting the presence of grade III or higher liver injuries.

For children who need abdominal CT, intravenous contrast alone is adequate without the need for oral contrast.

H&A

238
Q

What are the ATOMAC guidelines for suspected liver or spleen injury in children?

A
239
Q

Risk factors for failure of nonoperative management of liver and spleen injuries?

A
240
Q

Operative management principles of liver and spleen bleeding?

A

OPERATIVE MANAGEMENT OF LIVER AND SPLEEN BLEEDING

For splenic hemorrhage, the abdomen is initially packed in all four quadrants.

Packs are removed from one quadrant at a time, typically leaving the quadrant(s) most likely to contain the source of bleeding until last.

Definitive treatment options for splenic bleeding include packing and waiting, angiography with packing in place, splenorrhaphy, or splenectomy.

As NOM has become more successful, fewer than 400 children per year in the United States undergo splenectomy for trauma.

Operative management of liver bleeding can be challenging. The Western Trauma Association guideline for operative management of adult blunt hepatic trauma is useful.

In this stepwise approach, the abdomen is packed.

If the bleeding stops, the patient undergoes angiography and ICU monitoring.

If packing fails to stop the hemorrhage, a Pringle maneuver is performed.

If this controls bleeding, the bleeding is most likely from a hepatic artery or a portal vein branch. These are treated with selective vessel ligation and omental packing.

However, if the bleeding continues, the patient most likely has an inferior vena cava (IVC) injury and vascular isolation will be necessary for control.

A median sternotomy may be needed, and the IVC injury is approached after vascular isolation.

H&A

241
Q

What is an indication to terminate nonoperative management of blunt liver and spleen trauma?

A

For several years, consensus evidence has suggested 40 mL/kg of blood products during the first 24 hours is a breakpoint at which NOM is less likely to be successful.

Recent pediatric data from the U.S. military has confirmed 40 mL/kg is a predictor of early and late mortality in children.

Additionally, the need for early transfusion, systolic blood pressure less than 50 mmHg, or a recurrent episode of hypotension suggests a higher risk of mortality.

Early recurrent hypotension after blood transfusion in a pediatric trauma patient believed to be bleeding from a solid organ injury indicates failure of NOM and requires operation or another intervention such as embolization.

H&A

242
Q

What is the role of angioembolization in pediatric blunt liver and spleen injury?

A

Successful control of bleeding with angioembolization for children and adolescents with solid organ injury has been reported in several series and is included in the algorithmic management of traumatic injuries in children.

Contrast extravasation is seen in approximately 5–15% of children with splenic injury.

The majority of children with contrast extravasation, however, do not need angioembolization, and the current role for angioembolization appears to be limited to children who are otherwise failing NOM.

In cases of hemobilia (upper gastrointestinal bleeding due to liver injury with bleeding into the bile ducts), angioembolization has been described with good success.

Use of angioembolization in managing pseudoaneurysms of the liver or spleen is controversial and probably not necessary for most splenic pseudoaneurysms.

H&A

243
Q

What is the role of ERCP in pediatric blunt liver and spleen trauma?

A

Endoscopic retrograde cholangiopancreaticography (ERCP) after liver trauma appears to have a role in identifying and treating children with major bile duct injuries.

Stenting across the injured duct may be possible in selective cases.

Additionally, other therapeutic interventions include sphincterotomy and stenting of the ampulla to decrease the biliary tract pressure, even if the injury itself cannot be stented.

These procedures are often done as an adjunct to percutaneous drainage of a biloma secondary to a traumatic bile duct injury.

H&A

244
Q

AAST grading for Liver injury?

A

.

245
Q

AAST grading for splenic injury?

A
246
Q

AAST grading for renal injury?

A

Severity is assessed according to the depth of renal parenchymal damage and involvement of the urinary collecting system and renal vessels.

Grade I: Conservative management
- Subcapsular hematoma or contusion, without laceration

Grade II: Conservative management, under close observation
- Superficial laceration ≤1 cm depth not involving the collecting system (no evidence of urine extravasation)
- Perirenal hematoma confined within the perirenal fascia

Grade III: Conservative management under close observation; may be managed surgically if undergoing laparotomy for other abdominal injuries

  • Laceration >1 cm not involving the collecting system (no evidence of urine extravasation)
  • Vascular injury or active bleeding confined within the perirenal fascia

Grade IV: If unstable, consider surgical management, especially if undergoing laparotomy for other abdominal injuries
- Laceration involving the collecting system with urinary extravasation
laceration of the renal pelvis and/or complete ureteropelvic disruption
vascular injury to segmental renal artery or vein
- Segmental infarctions without associated active bleeding (i.e. due to vessel thrombosis)
- Active bleeding extending beyond the perirenal fascia (i.e. into the retroperitoneum or peritoneum)

Grade V: If unstable, consider surgical management
- Shattered kidney
- Avulsion of renal hilum or laceration of the main renal artery or vein:
devascularisation of a kidney due to hilar injury
devascularised kidney with active bleeding

Additional points:

Advance one grade for multiple injuries up to grade III.

“Vascular injury” (i.e. pseudoaneurysm or AV fistula) - appears as a focal collection of vascular contrast which decreases in attenuation on delayed images

“Active bleeding” - focal or diffuse collection of vascular contrast which increases in size or attenuation on a delayed phase
Imaging technique

The AAST guidelines recommend dual arterial/portal venous phase imaging for evaluation of a vascular injury of liver, spleen, or kidney.

If there are imaging or clinical findings suggesting collecting system injury (e.g. hematuria or blood at the meatus), additional delayed excretory phase images should be obtained after 5-15 minutes of delay to evaluate for urine extravasation.

Urinary contrast is usually hyperdense and readily distinguished from hemorrhagic vascular contrast.

Note that multiphase scanning should not preempt emergent management in the setting of hemodynamic instability or other life-threatening injuries.

Researchgate (based from AAST)

The most popular scheme for grading renal injury is the renal injury scale of the AAST. Although devised for adults, the scale is also used in children.

The majority of blunt renal injuries, up to 75%, are classified as grade I. This category includes patients with contusions, nonexpanding subcapsular hematomas without parenchymal lacerations, a limited perinephric hematoma, or a small subsegmental infarct.

The category is also used for flank trauma resulting in microscopic or gross hematuria in the presence of normal imaging.

The higher grades, IV and V, involve vascular and collecting system injuries.

A grade V injury implies avulsion or thrombosis of the main renal artery or vein with a devascularized kidney.

It must be remembered that the injury grading system is somewhat qualitative, and some injuries cannot be easily placed into one of the five categories.

In these cases, it is probably safer from a management point of view to upgrade, rather than downgrade, the injury.

Sherif

247
Q

Management of renal injury in children?

A

Renal injuries are less common than injuries to the liver or spleen, representing only about one-third as many admissions.

The initial management of blunt renal injury is similar to management of liver and spleen injuries, with >97% of cases managed nonoperatively.

One exception, however, is that bed rest after blunt renal injury is not required. Omission of mandatory bed rest for stable patients with blunt renal injury appears to result in a shortened hospitalization without an increased risk of bleeding or readmission.

Although NOM of renal injury grades I through III has been well accepted for many years, management of highgrade injuries without operation is now also the norm.

In one series reporting operative intervention in children with low-grade renal injuries, the reason for operation was generally for management of other injuries, or for children found to have congenital anomalies.

For grade IV and V renal injuries, surgeons have adopted NOM for hemodynamically stable patients with successful outcomes.

In a meta-analysis assessing NOM in grade IV renal injuries, 73% of patients were managed with NOM without intervention.

At least partial renal preservation was possible in 95% of these patients.

LeeVan et al. reported successful NOM for grade IV and V renal injuries in 80–100% of cases.

Risk factors for failure of NOM include injuries involving the renal collecting system, large perinephric hematomas, urinomas greater than 4 cm, lacerations to the anteromedial or medial portion of the kidney, the presence of dissociated renal fragments, and interpolar extravasation.

Transcatheter arterial embolization has been advocated as a first step in the management of active renal bleeding to maximally preserve renal parenchyma and function.

The primary indication for operative management is persistent hemodynamic instability.

Adjunctive procedures such as stenting, percutaneous drainage, and angioembolization may assist in avoiding laparotomy.

Late hypertension is uncommon, but occurs transiently in 6% of high-grade (≥III) injuries.

A small number of these patients require long-term antihypertensive treatment.

RENAL ARTERY THROMBOSIS

Renal artery thrombosis is a rare event in the pediatric population. The most common cause is a deceleration stretch or sheer injury to the arterial intima, followed by impaired blood flow and subsequent thrombus.

CT scan with intravenous contrast demonstrates nonopacification of the kidney.

The arterial thrombus and subsequent warm renal ischemia cause damage quickly.

Success after open surgical repair of the artery is low, even when revascularization is successful and blood flow is reestablished as early as 5 hours after injury.

In this situation, NOM is generally advocated.

Catheter-based arterial thrombolysis and stenting has been described.

Benefits of this approach include avoiding a laparotomy and faster reestablishment of blood flow, even in cases with delayed treatment.

Prior to initiation of this therapy, consideration should be given to the need for and ability to provide anticoagulation after stent placement.

Suction and aspiration of thrombus has been also described.

H&A

The management of blunt renal trauma has significantly shifted to a nonoperative approach, similar to that used for other solid organ injuries. Grade of injury is not an indication for operative exploration, and a number of recent publications and systematic reviews have shown excellent renal salvage rates with nonoperative management of grade IV and V injuries.

In addition, the availability of less-invasive techniques, such as angioembolization and endourologic ureteral stenting, has provided additional options for patients who fail initial nonoperative management, such as described in the index case.

Grade I–III injuries are considered minor. For these injuries, admission to a basic hospital bed is advised for at least 24 hours.

Bed rest until resolution of gross hematuria has been employed in almost all studies of nonoperative management, but the practice has not been scrutinized prospectively.

The treatment of a high-grade injury has been discussed in the index case.

The indication for placement of a Foley catheter should be close monitoring of urine output in cases of hemodynamic instability or associated head injury, and not hematuria or the renal injury itself.

Antibiotics should not be used prophylactically, even if there is proven injury to the collecting system.

A repeat CT scan may be performed in 48 hours if there is worsening of clinical status.

An increasing hematoma may be an indication for diagnostic and therapeutic angiography.

A urinary leak will often seal within a few days without any intervention. However, increased urinary extravasation is best approached by placement of a double-J ureteral stent.

Children who sustain an injury to the collecting system along with a major vascular injury appear to have a higher chance of requiring urologic intervention.

Persistent hemorrhage resulting in hemodynamic instability despite transfusion of packed red blood cells and correction of coagulopathy is one of the few indications for operative exploration.

The transfusion threshold before invasive intervention is required varies from two units of packed red blood cells to 50% of blood volume in different studies and should be established a priori by each trauma service.

When operation is necessary, a partial or total nephrectomy is most frequently performed.

Some have advocated a trial of angioembolization before operative intervention is undertaken, even in the presence of transfusion requirements. It is reasonable, therefore, to consider the need for transfusion as a trigger for angiography.

Although a nonoperative approach is now the standard for all renal injury grades, grading is still useful as it may predict the probability of an invasive procedure (including angioembolization), which increases from 0% in grades I and II to 67% in grade V.

A particular controversy has existed regarding the completely devascularized kidney, as manifested by failure to take up arterial contrast after blunt trauma.

The mechanism is usually a shear injury to the intima of the renal artery, resulting in complete thrombosis.

Some have advocated attempts at endovascular or operative revascularization if the injury is less than 6 hours old.

However, renal salvage rates in this injury are typically quite poor.

If there is a normal functioning contralateral kidney, nonintervention is a completely reasonable approach practiced in many trauma centers.

Patients have to be followed carefully for the development of hypertension.

In many, the affected kidney simply atrophies without blood pressure consequences.

Penetrating renal injury, due to gunshot or stab wounds, often involves other organs. Upon abdominal exploration, vascular or urologic repair, partial nephrectomy, or total nephrectomy should be considered, depending on the injury, associated injuries, and the hemodynamic status of the patient.

If a total nephrectomy is contemplated, an on-table intravenous program is advised to confirm normal function of the contralateral kidney and patency of the contralateral ureter.

Patients who sustain renal trauma, particularly grades IV and V, should be followed for variable periods, depending on the nature of the injury and the interventions required.

Chronic renal failure secondary to renal trauma is exceedingly uncommon. However, sequelae may include urinoma, perinephric abscess, decreased renal function, pseudoaneurysms, and hypertension.

The incidence of impaired renal function increases with injury grade and is seen mostly with grade IV and V injuries.

Urinomas may resolve spontaneously or respond to ureteral stenting.

Large urinomas and perinephric abscesses may require percutaneous drainage.

Hypertension probably has a much lower incidence than once thought, currently estimated to occur in less than 5% of injured patients.

Nevertheless, late diagnosis in pediatric patients may ensue, and this may have a deleterious effect on residual kidney function.

Kidney perfusion and function can be assessed by Doppler ultrasound and DMSA scan, respectively.

Patients who manifest abnormalities after high-grade injuries should be followed until adulthood and transferred to adult nephrologists if necessary.

Sherif

248
Q

Hallmark symptom of renal trauma?

A

The hallmark symptom of renal trauma is hematuria, defined as more than 5 red blood cells per high-power field on a urine specimen.

Microscopic hematuria is defined as less than 50 red blood cells per high-power field.

However, it must be remembered that very severe renal injuries, such as renal pedicle avulsions or complete renal artery occlusions, may not present with hematuria.

Thus, the absence of gross or microscopic hematuria does not rule out a renal injury.

Similarly, the presence of hematuria does not confirm a renal injury, since the source of the hematuria may be the ureter or bladder.

Patients with high-grade renal injuries may also present with hypotension due to significant hemorrhage.

In fact, persistent hemodynamic instability despite blood transfusions is currently one of the few indications for operative treatment of renal injuries.

Sherif

249
Q

What is an adequate CT scan evaluation of renal trauma?

A

Adequate CT-scan evaluation of the kidney should include:

1) an arterial phase (within 30 seconds of injection),

2) a nephrogram phase (80–90 seconds after injection), and

3) a delayed phase (approximately 10 minutes after injection).

The latter is particularly important to evaluate for urinary extravasation.

The accumulation of extravasated contrast in the delayed phase denotes a urinary leak, since extravasated contrast within the vascular system will wash out 10 minutes after injection.

The combination of imaging in the three phases will allow for the definition of the type of injury. These include contusions, infarctions, subcapsular hematomas, perinephric/retroperitoneal hematomas, lacerations, pseudoaneurysms, arterial extravasations, urinary extravasations, shattered kidneys, or any combination.

Images from the three CT scan phases in a 13-year-old boy with a grade IV blunt left renal injury from a bicycle handle bar are shown in Figure 65.5. The vascular pedicle is well visualized in the arterial phase and is intact. The extent of the laceration to the kidney is best seen in the nephrogenic phase, whereas the pyelogenic (delayed) phase rules out a urinary leak. The patient was successfully managed with a nonoperative approach.

Sherif

250
Q

AAST grading of pancreatic injury?

A
251
Q

Management of blunt pancreatic injuries in children?

A

The most important aspects of grading pancreatic injuries are the location of the injury and the status of the main pancreatic duct.

Management of children with a high-grade pancreatic injury is controversial.

AAST grade I and II pancreatic injuries are best managed nonoperatively.

Patients with a transected main pancreatic duct (grade III) have fewer complications with distal pancreatectomy, and laparoscopic distal pancreatectomy after trauma also has been found to be beneficial if diagnosed shortly after the injury.

The 2014 multi-institution retrospective study done by the PATCH group found fewer complications, fewer pseudocysts, less time on parenteral nutrition, fewer interventions, and a shorter hospital length of stay with an operative approach.

On the other hand, some studies have shown successful NOM for ductal injuries.

A recent multicenter study for NOM of high-grade pancreatic injuries demonstrated success in avoiding operation in 86 patients, but only a fourth of the patients had confirmation of ductal injury by ERCP or magnetic resonance cholangiopancreatography (MRCP), and the study was limited to only successful cases of NOM.

Moreover, no effort was made to distinguish between partial injury of the main duct or complete transection.

In contrast to NOM for splenic injury, however, NOM of pancreatic injury does not save the pancreas.

Studies have shown that NOM in patients with pancreatic duct injury results in loss of the pancreas distal to the ductal injury, most likely due to pancreatic enzyme leak.

For injuries involving less than 50% of the pancreas, this does not appear to result in endocrine or exocrine dysfunction. However, those patients who lose more than 50% function often have abnormal glucose tolerance.

For more proximal injuries, operative reconstruction of the distal pancreas with a Roux-en-Y pancreaticojejunostomy provides ductal drainage to the gastrointestinal tract, allowing pancreatic salvage and preserving both endocrine and exocrine function.

A simpler reconstruction with a pancreaticogastrostomy also has been described, and large adult series now show good outcomes in patients undergoing central pancreatectomies for both traumatic and nontraumatic disease.

Successful endoscopic management of a pancreatic injury has been reported and may have advantages in injuries to the pancreatic head.

Failures, however, are not uncommon, and pancreatic preservation does not appear likely if the duct is completely transected.

The ideal use appears to be for injuries in which the pancreatic stent can completely cross the injured duct, rather than cases treated with sphincterotomy and stenting of the ampulla.

H&A

The controversy in managing pediatric pancreatic trauma has essentially revolved around grade III injuries, those that involve major ductal disruption at the body or neck to the left of the superior mesenteric vein.

Different authors and institutions have touted the advantages of both operative and nonoperative management, and the pediatric trauma surgeon must understand the arguments for and against operative intervention. Recent results from the National Trauma Data Bank, appearing to favor nonoperative management, did not stratify patients by pancreatic injury grade.

A major advantage of early intervention for pancreatic transection is operating in a relatively clean field, free of the inflammatory findings that develop after ductal disruption.

Figure 64.2 shows the surgical field in an 8-year-old patient who underwent an open distal pancreatectomy for transection. The final result of a spleen-preserving distal pancreatectomy with ligation of the pancreatic branches of the splenic vein and control of the pancreatic edge with prolene sutures is shown. One can appreciate the absence of hemorrhage and saponification. This has prompted many to approach this lesion by minimally invasive techniques, such as shown in the index case. Multi-institutional experiences have recently been published, confirming the efficacy of this approach.

Those who support nonoperative treatment point to the high success rate of this approach and avoidance of operative risks, as well as possible future pancreatic insufficiency.

While most transections can, in fact, be managed nonoperatively, this typically comes at the cost of a longer hospital stay, longer duration of parenteral nutrition, increased need for early and late interventional procedures such as ERCP and percutaneous pseudocyst drainage, and overall increases in complication rates.

A recent multi-institutional study did not demonstrate any benefit to early ERCP in patients managed nonoperatively.

ERCP appeared most useful in diagnosis of ductal injury and management of late complications such as stricture and fistula.

The purported advantage of pancreatic parenchymal preservation is challenged by recent reports showing atrophy of the distal pancreas in a large number of patients treated nonoperatively.

One must also remember that, unlike adults, children are likely to need general anesthesia for most invasive procedures.

There have been no randomized controlled studies of operative versus nonoperative treatment in children. However, recent multi-institutional retrospective series strongly favor distal pancreatectomy for complete transection.

Lower grades of pancreatic trauma are typically treated nonoperatively.

ERCP or operative procedures, such as debridement and drainage, are used very selectively in cases with early deterioration or late pseuodocyst formation.

Sherif

252
Q

Management of blunt gastric injuries?

A

Most traumatic injuries to the stomach are due to penetrating trauma.

In blunt trauma, the most common mechanism is a high-pressure rupture of a full stomach.

The gastric rupture typically occurs along the greater curvature of the stomach, and nonlinear lacerations are common.

Treatment involves abdominal washout, debridement of devitalized stomach, and primary repair.

Laparoscopic repair also has been described.

H&A

253
Q

Management of blunt duodenal injuries?

A

Duodenal injuries are relatively infrequent.

In children younger than 4 years of age, this injury is strongly associated with child abuse.

Blunt force trauma to the abdomen by the assailant often injures the relatively fixed second portion of the duodenum, although other areas may be damaged.

In older children, a duodenal injury is typically caused by a concentrated transfer of injury to the duodenum, often where it crosses the spine.

Motor vehicle collision is the second most common mechanism after child abuse.

AAST duodenal injury grades are shown in Table 16.4.

Patients with an intramural duodenal hematoma often can be managed with nutritional support with a reasonable expectation of resolution of the hematoma at a median of 9 days.

In cases of duodenal perforation, operative repair is needed, even when diagnosed late.

For nondestructive injuries, primary repair of the duodenum with or without a drain is reasonable.

Options for reconstruction of the destroyed duodenum, however, may require complicated repair strategies.

H&A

254
Q

Management of blunt small bowel injuries?

A

Injuries to the small intestine are generally repaired primarily in penetrating trauma, but may require resection and anastomosis following blunt injury or cases in which the mesentery has been torn.

In rare cases of avulsion of the superior mesenteric artery, urgent vascular reconstruction based on training and available resources may be successful, but these injuries are often lethal.

H&A

255
Q

Management of blunt colonic injuries?

A

COLONIC INJURY

Management of colonic injuries has evolved in the last few decades.

In hemodynamically stable patients, debridement or resection of the damaged tissue followed by primary repair can be performed without fecal diversion.

In unstable patients, treatment should be individualized.

Damage control with surgical staplers, bowel discontinuity, and early delayed repair has been advocated in adult centers.

H&A

256
Q

Management of rectal injuries?

A

A rectal injury may occur through a variety of mechanisms, including sexual abuse, straddle injuries, watercraft injury, and accidental impalement.

Personal watercraft injuries (e.g., Jet Skis, WaveRunners, or Sea-Doos) may involve hydrostatic force injury as the propulsive force enters the rectum, causing perforation.

Rarely, pelvic fractures may secondarily perforate the rectum.

Rectal examination alone is unreliable to exclude these injuries.

Abdominopelvic CT with or without anoscopy, proctoscopy, vaginoscopy, or cystoscopy may be helpful.

Rectal injuries above the peritoneal reflection are treated the same as colonic injuries, and repair without diversion is reasonable.

Below the peritoneal reflection, management depends on the degree of injury and concurrent injuries.

The Eastern Association for the Surgery of Trauma (EAST) guidelines conditionally recommend diversion, as do some pediatric surgeons.

The data, however, are limited. For injuries less than 50% of the bowel circumference, primary transanal repair may be possible without diversion.

In cases with greater than 50% of the circumference involved, or significant destruction of the surrounding tissues, primary repair and diversion is advocated.

Females with combined rectal (above the dentate line) and vaginal injuries are at a high risk of developing a rectovaginal fistula regardless of the repair, but early diversion may help with definitive repair of recurrent fistulas.

Most cases with uncomplicated isolated rectal injuries undergoing early repair retain fecal continence.

H&A

257
Q

Management of blunt abdominal aortic trauma?

A

Blunt abdominal aortic injury is an uncommon event, often associated with severe deceleration injuries.

Patterns of injury associated with blunt abdominal aortic injury can include lap belt bruising, disruption of the abdominal wall fascia, Chance fractures of the lumbar spine, bowel injury, and/or disruption of the cauda equina.

Endovascular management and stenting of the aorta and iliac vessels in children have been reported, but primary repair of the aortic injury is still performed in many centers.

The primary advantages of endovascular stenting include potentially avoiding a laparotomy or avoiding contamination of the vascular repair in patients with a concurrent bowel injury.

Longterm follow-up studies in children, however, are not available.

H&A

258
Q

Management of blunt diaphragmatic injury?

A

Rupture of the diaphragm due to blunt trauma occurs infrequently but is typically associated with a severe and rapid increase in intra-abdominal pressure.

Blunt injury to the diaphragm is frequently diagnosed late, and even delayed presentations are not uncommon.

Repair may be done at laparotomy for other injuries, but a primary laparoscopic or thoracoscopic repair are both feasible.

Thoracoscopic repair may have advantages in children with delayed presentations, including better access to adhesions that have developed in the chest and a better view of the diaphragm.

The abdominal approach is preferred in the acute setting to exclude associated intra-abdominal injury.

H&A

259
Q

Complications of splenic trauma?

A

Late abdominal pain after NOM of blunt splenic injury is common and was seen in up to 15% of patients in one study.

When reimaging for pain is performed, most studies show only a healing spleen.

When imaging demonstrates other abnormalities, pseudocysts and pseudoaneurysms are occasionally identified.

Splenic Pseudoaneurysm
Splenic pseudoaneurysms are vascular complications that result from injury to the vessel wall.

In contrast to a true aneurysmal dilation, all layers of the vessel wall are not involved in the dilated pseudoaneurysm.

An injury to the vessel is typically contained by the surrounding tissue, which compresses the associated parenchyma.

Recent studies have found that splenic pseudoaneurysms after blunt trauma are common, with as many as 17% of patients developing a pseudoaneurysm.

In a letter to the editor, Letton et al. challenged the clinical significance of this finding.

In another study, Kristoffersen and Mooney found no clinically significant vascular complications in long-term follow-up of a large series of patients with splenic injury.

Over an 18-year series, Safavi identified 10 pseudoaneurysms among 362 children with splenic injuries.

These were more common following high-grade traumatic injuries, and spontaneous thrombosis was common.

Two were electively embolized, and one splenic artery pseudoaneurysm bled.

Routine or increased use of reimaging is associated with higher rates of identification, often without clinical significance.

In another study identifying asymptomatic splenic artery pseudoaneurysms, 89% underwent spontaneous thrombosis.

The APSA guidelines do not recommend routine reimaging, and there is not strong evidence for routine reimaging after splenic trauma.

For symptomatic pseudoaneurysms, angioembolization may be beneficial and was successful in over 85% of cases in one study.

Splenic Pseudocysts
Post-traumatic pseudocysts are typically unilocular cysts with a nonepithelial lining.

Although some studies suggest these are nontraumatic in origin, enough cases after blunt trauma exist to suggest a portion of these are post-traumatic.

In contrast to pseudoaneurysms, these cysts typically have no flow within the lesion and do not undergo spontaneous resolution.

Percutaneous drainage alone has a high failure rate, but success with ethanol sclerotherapy has been described.

Laparoscopic partial splenectomy or marsupialization are common treatments.

In cases in which these options are not feasible, splenectomy can be considered.

Overwhelming Postsplenectomy Infection
Overwhelming postsplenectomy infection is an uncommon, but significant complication of splenectomy.

Causative organisms include Streptococcus pneumoniae, Haemophilus influenzae type B, and Neisseria meningitidis.

The incidence is difficult to determine, but it is probably around 1% after splenectomy.

Early symptoms may include fatigue, colored skin, weight loss, abdominal pain, diarrhea, constipation, nausea, and headache.

A rapid downhill clinical course can quickly lead to coma and death within 24–48 hours due to shock, hypoglycemia, acidosis, electrolyte abnormalities, respiratory distress, and disseminated intravascular coagulation.

Vaccinations after splenectomy should be given after postoperative day 14 to optimize the patient’s immune response.

Vaccination currently consists of pneumococcal vaccine, meningococcal vaccine, and H. influenzae serotype b (Hib) vaccines.

Antibiotic prophylaxis for children also may be indicated.

Current recommendations can be found at https://www.cdc.gov/vaccines/ schedules/.

In some countries, for children who are asplenic, the parent or guardian is advised to keep a full course of antibiotics at home in case of illness.

Antibiotics should be initiated at the first signs of infection, such as fever, chills, sore throat, or cough.

Families should then seek medical attention for further testing at the onset of any illness.

Vaccinations should be updated every 3–6 years throughout the patient’s life, depending on the specific vaccine.

H&A

260
Q

Complications of liver injuries?

A

Biloma

Biloma is a collection of bile within the abdominal cavity.

Bile leak after blunt hepatic trauma is uncommon, occurring in only 2% of liver injuries in one retrospective study.

Typical symptoms include increasing abdominal pain and an ileus in conjunction with an elevated bilirubin and liver function tests following blunt abdominal injury.

Most bilomas can be managed with image-guided percutaneous drainage.

Cultures should be obtained at the time of drainage, because infected bile collections can occur.

The addition of bile duct stenting via ERCP with or without sphincterotomy to decrease the biliary pressure has been described.

In patients who require an operation for another indication, laparoscopic or open drainage with abdominal washout also can be beneficial.

Antibiotics are needed in patients with an infected bile collection.

Major Bile Duct Injury

Very few pediatric series report major bile duct injury after liver injury, but one adult series found significant bile duct injury in 1.4% of injured patients.

This study suggested ECRP may be beneficial in treatment as well as diagnosis, and a small study also showed efficacy in children.

Many major bile duct injuries in the past were probably empirically treated bilomas or repaired with hepatorrhaphy without specific identification of the ductal injury by MRCP or ERCP.

Open operative management is rarely required.

Similar to biloma resulting from minor ductal injuries, treatment involves imaged-guided percutaneous drainage of the biloma, endoscopic stenting across the duct (if possible), and sphincterotomy of the ampulla.

Hepatic Artery Pseudoaneurysm
Hepatic artery pseudoaneurysms are rare and usually associated with high-grade injuries (grade IV or higher).

Unfortunately, pseudoaneurysms following hepatic injury may be less likely to undergo spontaneous thrombosis and may also be at a higher risk of bleeding.

Limited data are available to make recommendations for these lesions, but angioembolization has been reported for these lesions once they are identified.

H&A

261
Q

Complications of kidney injuries?

A

Renal Artery Pseudoaneurysms

Traumatic pseudoaneurysm of the main renal artery is uncommon, but can occur in high-grade blunt and penetrating renal trauma.

Presenting symptoms include hematuria, hypertension, or abdominal pain.

Angiography with embolization of the pseudoaneurysm is first-line therapy.

Surgery is typically reserved for patients who are hemodynamically unstable from a rupture or if angioembolization is unsuccessful.

Urinoma

High-grade renal injuries often include damage to the collecting system and can be associated with a collection of urine near the kidney, referred to as a urinoma.

A persistent or increasing urinoma may be concerning for ureteropelvic junction disruption.

Although a symptomatic urinoma occurs in approximately 15–20% of patients with a highgrade renal injury, over three-fourths of these patients can be managed with percutaneous drainage of the urinoma and cystoscopic placement of a ureteral stent.

Urinomas are often associated with bleeding from the renal parenchyma, and subsequent hematuria is common.

Operative intervention for a urinoma due to trauma, however, is rare in the absence of congenital urinary abnormalities.

Pyeloplasty may occasionally be required.

Hypertension

Hypertension after blunt renal injury is uncommon in children.

A Page kidney is renal hypertension from external compression of the organ. This appears to be an uncommon event.

In studies by Graziano et al. and Wu et al., no patients developed hypertension in early follow-up.

The risk of long-term complications appears to be correlated with an increased grade of injury, but hypertension appears to be infrequent and relatively easily controlled.

H&A

262
Q

Guidelines of follow up care of blunt organ injury patients?

A

ACTIVITY RESTRICTION

The 2000 APSA guidelines recommended activity restriction equal to the grade of injury + 2 in weeks.

The study by Kristoffersen and Mooney showed that a significant number of patients do not follow these activity restrictions, and no convincing evidence is available to support or refute this recommendation, which continues to be in general use.

The APSA recommendations also allow injured children to return to school after hospital discharge, provided they do not participate in gym class, sports, activities with wheels, or any activities in which both feet leave the ground at the same time.

REIMAGING AFTER LIVER OR SPLEEN INJURY

According to the APSA guidelines, no routine imaging is required for blunt liver or spleen injury.

For children with symptoms of dizziness, lightheadedness, jaundice, increasing abdominal or shoulder pain, reimaging with US is recommended.

One paper suggested high-grade liver injuries may benefit from reimaging prior to discharge due to the risk of bleeding but a subsequent large study could find no benefit to reimaging children with blunt liver or spleen injury.

REIMAGING AFTER RENAL INJURY

Unlike liver and spleen injuries, reimaging after high-grade renal injury is commonly used to quantify renal function and evaluate healing.

Technetium-99m dimercaptosuccinic acid scan (DMSA) in the early postinjury period and at long-term follow-up can provide an estimate of renal function, but this study does not generally lead to a change in therapy.

The study is commonly performed, although no strong recommendation can be made for its utility.

Similarly, some centers perform follow-up US on renal injuries involving the collecting system.

These may provide utility if attempting to evaluate for an underlying congenital anomaly.

There does not appear to be a strong consensus on the type of imaging follow-up. US, CT, and MRI have all been described.

Postinjury imaging is typically done around the 3-month period in centers that perform routine renal imaging.

H&A